You are on page 1of 223

PHARMACY

EXAMINATION SECTION

TEST 1
DIRECTIONS: Each question or incomplete statement is followed by several
suggested answers or completions. Select the one that BEST answers the question
or completes the statement. PRINT THE LETTER OF THE CORRECT ANSWER IN THE
SPACE AT THE RIGHT.
Questions 1-5.
DIRECTIONS: Questions 1 to 5 refer to the prescription below.
Rx:
Ichthammol 1.2 grams
Resorcinol 2.0 grams
Zinc oxide 10.0 grams
Lime water 60.0 ml.
Olive oil 60.0 ml.
Mft.
Sig: Apply to affected area as directed
1. Which of the following is the BEST order of mixing the

prescription?

A. Triturate the powders with the ichthammol, add this to the lime water, and q.s.
with olive oil
B. Dissolve the resorcinol in the lime water, add the zinc oxide and ichthammol, and
q.s. with the olive oil
1. C Add the lime water to the olive oil; then add the powders and the ichthammol
D. Add the ichthammol, zinc oxide, and resorcinol to the olive oil; then q.s. with the
lime water
E. Dissolve the ichthammol and resorcinol in the olive oil, dissolve the zinc oxide in
the lime water; then
triturate the two portions together
2. The prescription above is BEST prepared
A. in a prescription bottle
B. in a mortar
C. in a graduated cylinder
D. on an ointment slab
E. by triturating with a rubber spatula
1. C

2. B

3. Lime water contains which of the following agents?

A. Aluminum acetate
B. Acetic acid
C. Ferrous chloride
D. Calcium hydroxide
E. Lime juice
4. TRUE statements about materials in the prescription inelude which of
the following?
I. Resorcinol is soluble in water.
II. Zinc oxide is soluble in water.
III. Olive oil is miscible with water.
The CORRECT answer is:
A. I only
B. II only
D. I and II
E. I, II, and III

C. III only

5. The prescription above is BEST described as a(n)


A. emulsion
B. magma
C. solution
D. gel
E. trituration
3. D

4. A

5. A

Questions 6-8.
DIRECTIONS: For Questions 6-8 below, ONE or MORE of the responses given is
correct. Decide which of the responses is (are) correct.
Mark

A
B
C
D
E

if 1, 2, and 3 are correct;


if only 1 and 2 are correct;
if only 2 and 3 are correct;
if only 1 is correct;
if only 3 is correct.

The graph above illustrates the time course of the mean plasma theophylline
(administered as the ethylenadiamine salt) concentra tion when 0.5 gram is
administered
(A) intravenously;
(B) as a retention enema; and
(C) as an oral tablet, in a cross-over-designed experiment involving 30 adult males.

6. TRUE statements regarding the oral administration (curve C) of


theophylline include which of the following?

1. It exhibits the same time of peak concentration as the enema.


2. The lag time of absorption may be due to the dissolution characteristics of the
theophylline in the gut.
3. It is physiologically equivalent to the intravenous injection.
7. TRUE statements regarding the intravenous administration (curve A) of
theophylline include which of the following?
1. The distribution of theophylline to tissue is faster than its excretion from the body.
2. The elimination t1 of theophylline is approximately 3.5 hr.
3. The intravenous profile indicates a two-compartment open model system.
8. From the graph above, it can be concluded that
1. the degree of bioavailability of theophylline is
affected by route of
administration
2. theophylline is excreted from the body by zero-order kinetics
3. all modes of administration reach the same peak plasma concentration
9. A physician calls and wants to know the name of a hypnotic that will
LEAST interfere with the control of his patient who is on Coumadin. The
pharmacist should suggest
A. Dalmane
B. Doriden
C. Nembutal
D. Noctec
E. Tuinal
10. Tony, a known alcoholic who has cirrhosis of the liver, is brought into
the hospital with delirium tremens. On admission, he is quite agitated.
The resident on duty should select which of the following as the
appropriate sedative for Tony?
A. Meperidine
B. Pentobarbital
C. Paraldehyde
D. Glutethimide
E. Secobarbital
11. A patient presents a prescription that requires the
extemporaneous
preparation of a buffered eyedrop. Which of the following is the source for
information on the correct buffering of the eyedrop?
A. FACTS AND COMPARISONS B. THE MERCK MANUAL
C. PHARMINDEX
D. REMINGTON'S PHARMACEUTICAL SCIENCES
E. PHYSICIANS' DESK REFERENCE
6. B

7. A

8. D

9.A

10. C

11. D

12. A physician asks for information on an extemporaneously


compounded prescription used for the treatment of warts. Which of the
following is the BEST reference for immediate information?
A. AMERICAN HOSPITAL FORMULARY SERVICE
B. THE MERCK MANUAL
C. UNITED STATES PHARMACOPEIA
D. PHYSICIANS' DESK REFERENCE
E. FACTS AND COMPARISONS
Questions 13-14
and
15-17.
DIRECTIONS:
For Questions 13-14 and 15-17 below, ONE or MORE of the responses given
is(are)correct.Decidewhich of the responses is (are) correct.
Mark

A
B
C
D
E

if
if
if
if
if

only
only
only
only

1, 2, and 3
1 and 2
2 and 3
1
3

are correct;
are correct;
are correct;
is correct;
is correct.

The stability of an erythromycin salt from a nonenteric coated tablet was evaluated
in vitro in several pH solutions as a function of time.
13. Thirty minutes after the stability run has commenced, which of the
following is TRUE about the erythromycin salt?
1. In the pH 1.2 solution, it is almost totally degraded.
2. In the pH 2.2 solution, it is approximately 60% degraded.
3. In the pH 3.2 solution, it is approximately 40% degraded.
14. From the graph above, it can be concluded that
1. the rate constant for erythromycin degradation is the same for all three solutions
2. the stability of erythromycin increases toward a neutral pH
3. erythromycin degrades by a first-order rate of reaction

Figure 1
12 B

Figure 2
13 A

14 C

The effects of magnesium stearate (a tablet lubricant) on the dissolution rate of


Drug X and on the bioavailability of Drug X were evaluated and are shown in Figures
1 and 2, respectively.
Tablets each contained 300 milligrams of Drug X and 60 milligrams of starch. In
addition, Tablet A contained 1 percent magnesium stearate, Tablet B contained 2
percent magnesium stearate, and
Tablet C contained 3 percent magnesium stearate. Each data point is the average
for 10 tablets
(Figure 1) or 10 human adult males (Figure 2).

15. From Figures 1 and 2, it can be concluded that, in general,


1. all three formulations would be equally efficacious for therapy
2. in vitro data will predict in vivo data for all drugs
3. the in vitro data for Drug X was a good predictor of the in vivo data for Drug X
16. Prom Figure 2, it can be concluded that
1. after 24 hours, the products are not physiologically equivalent with respect to
Drug X
2. the percent of magnesium stearate in the formulation has affected the
bioavailability of Drug X
3. all three formulations reach the same peak urine concentration
17. From Figure 1, it can be concluded that
1. the inclusion of magnesium stearate has affected the rate if dissolution
2. the greater the amount of magnesium stearate, the faster the rate of dissolution
3. all three tablet formulations disintegrated at the same rate

15.E

16.B

17.D

Questions 18-20.
DIRECTIONS:
Questions 18-20 refer to the prescription below.
Preanisone
0.06 gram
Aspirin
9.00 grams
Mg trisilicate
4.50 grams
Mft
Sig:

caps #30
Caps 1

q.i.d.

18. The prescription can be filled correctly by using which of the


following?
A. Deltasone
B. Sterazolidin
C. Medrol
D. Ascriptin
E. Decadron
19. TRUE statements about prednisone include which of the
I.
The amount prescribed is an overdose.
II.
The amount in each capsule is 20 mg.
III.
Tablets can be ground up to obtain the amount needed.
The CORRECT answer is:
A. I only
B. II only
D. I and II
E. I, II, and III

following?

C. III only

20. Which of the following statements about the preparation of the


prescription above is TRUE?
A.
It is best prepared
in a mortar.
B.
It is best prepared
on an ointment slab.
C.
It is best prepared
with a hard rubber spatula.
D.
It is best prepared
with a sifter.
E.
It
should not be compounded
since the prescription is irrational.
21.

Rx: ZnO
Amylum
Glycerin
Rose water

Sig:

Apply

as
q.s.
q.s. a d.

3.0
60.0

b.i.d. to rash

In order to prepare the prescription above, the pharmacist should


A. solubilize the ZnO and amylum with the glycerin, and then add the rose water
B. emulsify the rose water with acacia, add the glycerin, and then add the ZnO and
amylum
C. use methylcellulose in a 1% concentration, levigate this solution, the ZnO, and
the amylum with glycerin, and then gradually add the rose water

D. dissolve the ZnO and amylum with 95% alcohol, add the glycerin, and then add
the rose water
E. levigate the ZnO and amylum with minimal mineral oil, add the glycerin, and
then add the rose water
18. A
19. C
20. A
21. C
22. Which of the following medications must be dispensed for patient-use
in its original container?
A. Achromycin-V pediatric suspension, 125 mg/5 ml
B. Kaon tablets
C. Phenergan expectorant, plain
D. Nitroglycerin tablets E. Tuss-Ornade liquid
Questions 23-24.
DIRECTIONS: Questions 23-24 refer to the prescription below.
Lasix 40 mg.
No. 100
Sig: 1 daily
23 The BEST time for the
A. in the morning
B. after lunch
C. mid-afternoon
D. an hour before supper
E. at bedtime

patient to take the

Lasix is

24 When filling the prescription above, the pharmacist should dispense a


A. green tablet
C. pink capsule
B. white tablet
D. maroon capsule
E. yellow tablet
25 Which of the following characteristics of a drug would prevent its
inclusion in a sustained release dosage form?
A. Biological half-life of 4-6 hours
B. Good water solubility
C. Absorption by active transport
D. Pharmacologic effect reflects the blood level
E. Moderate dose required for therapeutic effect
22. D

23. A

24. C 25. C

KEY (CORRECT ANSWERS)


1. C
5. A
7. A
11 D
19. C

2. B
6. B
8. D
12 B
17.D
20. A

3. D

4. A

9. A
13 A
18. A
21. C

10. C
14 C

15.E

22. D

23. A

16.B
24. C

25. C

Test 2
DIRECTIONS:
Each question or incomplete statement is followed by several sug- gested answers
or completions. Select the one that BEST answers the question or completes the
statement. PRINT THE LETTER OF THE CORRECT ANSWER IN THE SPACE AT THE
RIGHT.

1.
Parker, Robert
Birth:4-21-31
839 Oak St.
Allergies, Sensitivities: Chocolate
Family Physician: C. E. Sutton
Date Rx No. Patient Physician
Drug(s)
7/12
----Parker
Sutton
Darvon
65 mg
Mr. Parker's medication profile above indicates that
following is CONTRAINDICATED for him?

which of the

A. Gantrisin syrup
B. Kaon elixir
C. Novahistine DH
D. Pentids for syrup
E. Achromycin syrup

2.

Tom Carter 321 Carolina Ave.


Charleston, W.Va.
1/6
Digoxin
.25 mg.
1 q.d.
30
2/1
Digoxin
.25 mg.
1 q.d.
30
2/1
Diuril
500 mg.
1 b.i.d.
60

Mr. Carter comes into a pharmacy for a refill of his digoxin and Diuril prescriptions
and requests Riopan for heartburn. He mentions that he has been bothered by leg
cramps recently.
According to the medication profile above, which of the following statements is
CORRECT and PERTINENT?
A. Riopan is not indicated for heartburn.
B. Mylanta is a more suitable antacid.
C. Potassium replacement may be necessary.
D. Digoxin and Diuril are contraindicated.
E. Riopan is contraindicated due to its high sodium content.
3. The active ingredient of the parathyroid gland is

extracted with

A. water

B. alcohol and water

C. ether

D. alcohol

E. glycerin

4. Which of the following drugs is formulated into an enteric-coated


tablet in an effort to minimize possible gastric disturbances?
A. Acetaminophen
Sulfisoxazole

B. Diazepam

C. Bisacodyl

D. Pyribenzamine

E.

5. What route of administration should be used to administer 10


milligrams of morphine sulfate to a patient in the acute phase of a
myocardial infarction?
A. Subcutaneous injection
B. Intravenous injection
C. Intramuscular
injection
D. Oral tablet
E. Retention enema
6. Which is found in vitamin B12?
A. Magnesium
B. Nickel
C. Iron
Manganese

7.

D. Cobalt

E.

For:
Address:

Mr. Sidney Heimer


Today
424 Long Street
Premarin 1.25 mg.
No. 100
Sig: Tabs i t.i.d.
Lawrence Lang, M.D.

The pharmacist knows that Mr. Heimer has prostate cancer. When the prescription
above is filled, Mr. Heimer states, "Hey, those look like the female hormones my
wife used to
take! Isn't there a mistake?"
How should the pharmacist reply?
A. There is an error; let me call your doctor.
B. There is no error; female hormones are used for your condition.
C. I don't know; call your doctor.
D. Female hormones are safer than male hormones.
E. He meant to order a male hormone; let me call him.
8. Mrs. Selma Storey comes into the pharmacy in her tennis outfit for a
refill on her Ortho-Novum 1/50 and her NPH U-100. While waiting for her
prescription, she becomes shaky and diaphoretic. The patient should be
advised to
A. discontinue her birth control pills because they are aggravating her diabetes
B. inject herself with some insulin right away

C. relax and eat a candy bar while waiting for her prescription
D. stop exercising so strenuously
E. increase her insulin dosage because of the drug interaction with the oral
contraceptive
9. Boric acid is more soluble in
A. glycerin than in water B. water than in glycerin
C. water than in boiling water
D. alcohol than in glycerin
E. alcohol than in boiling water
Questions 10-16.
DIRECTIONS: Questions 10-16 refer to the following set of orders
2.0-gram
Na diet
Stat. protime and daily
Daily FBS
SMA- 6 every other day
Up in chair as tolerated
Elastic Hose
I&0
Irrigate with Foley catheter t.i.d. with NSS Coumadin, 7.5 mg. q.d. at 1,700
hr. P.O.
Lasix,
Lanoxin,
Mi-Cebrin T,
Orinase,
Quaalude,
MOM
Cascara

40 mg.
0.125 mg.

Talwin:

30 mg.
30 mg.

500 mg.
150 mg.
30 mg.
5 ml.

q.a.m.
daily
h.s.

P.O.
P.O.
q.d.
P.O.
p.r.n.
p.r.n. constipation
p.r.n. constipation

q.4-6h.
q.4-6h.
M. Niles, M.D.

p.r.n. pain P.O.


p.r.n. severe pain 1.M.

10. Lanoxin is a trade name for


A. digitalis
B. digitoxin
C. digoxin D. lanatoside

E. lanadrine

11. The Talwin order should be questioned because


A. it is only available for oral administration
B. it is only available for parenteral administration
C. the oral dose of Talwin is 50 mg.
D. 30 mg. is an excessive dose
E. it is not effective against pain
12. The drug that is available from other manufacturers under the trade
names Sopor or Parest is also available under the name
A. Lasix
B. Lanoxin C. Orinase
D. Quaalude
E. Talwin
13. The order for a daily FBS refers to a test for the determination of
A. congestive heart failure

B. diabetes mellitus
C. high blood pressure
D. mental depression
E. myocardial infarction

14. The Foley catheter called for in the order above is inserted through the
A. anus
B. ear
C. mouth
D. nose
E. urethra
15. The order "stat. protime and daily" is for a laboratory test used to
monitor therapy with
A. aspirin
B. Coumadin
D. Lasix
D. Orinase
E. Quaalude
16. According to the orders above, the patient apparently
from
A. a severe infection
B. agranulocytosis
C. a cardiovascular disease
D. malnutrition
E. hypoglycemia
17. Which is the vehicle for camphor liniment?
A. Cottonseed oil
B. Water and alcohol C. Olive oil
None of these
18.
Rx:

is suffering

D. Alcohol

E.

Roger Lamb
Room 208
Terramycin
500 mg.
Solu-Medrol
125 mg.
D5W
1,000 ml.
125 ml/hour

When the pharmacist receives the order above, he should


A. fill it as written
B. suggest to the physician that the Terramycin be given I.M. and the Solu-Medrol be
given I.V.
C. suggest to the physician that the Solu-Medrol be given I.M. and the Terramycin be
given I.V.
D. inform the physician that neighter drug should be given I.V.
E. inform the physician that the Solu-Medrol is an overdose.
19. A pharmacist dispenses Zyloprim to Mr. Swift on May 15. The next
day Mr. Swift presents another prescription for0.5 milligram of colchicine,
to be taken every 30 minutes until diarrhea occurs, then one three times
daily. The pharmacist should
A. fill the prescription as written

B. tell thepatient to stop taking the Zyloprim


C. adivse the physician to add Benemid to the therapy
D. advise the patient to take Kaopectate with each dose
E. refuse to refill the prescription
20. Camphor water is prepared by
A. steam distillation
B. sublimation
elutriation
E. use of special solvent

21.

C. simple solution

Pot. Penicillin G
KCI
D5W
Adm. I.V.

D.

5,000,000 U
120
mEq.
1,000
ml.

over

4 hr.

When presented with the hospital order above, the pharmacist should
A. fill the order as it is written
B. substitute NSS for the D5W since it is more compatible with the antibiotic and KCI
C. add the required amount of NaCl to make the solution isotonic
D. check with the physician as to the amount and infusion rate of KCI
E. suggest that the antibiotic be administered I.M. since it is incompatible with the
KCI
22. Another anme for methadone is
A. aminomone
B. dionin
C. metaphenone
E. dolamine

D. dolophine

23 A preparation that consists of two immiscible liquids, usually water and


oil, one of which is dispersed as small globules in the other, is defined as
a(n)
A. elixir
B. suspension
C. emulsion
D. fluidextract
E. syrup
24 The preservative for U.S.P. juices is
A. benzoic acid
B. sodium benzoate
E. chloroform
25.

C. alcohol

D. glycerin

St . James' Hospital
313 Cricket Street

281-

6222
For: Mrs. Richard Lent-Room 117-East
D5NS
1 amp.
KCI

1,000
B&C
40

ml.
mEq.

Dr. Consiglio

Date: Today

Which drugs in the prescription above are chemically INCOMPATIBLE in the


same I.V.?
A. Dextrose + B-complex with C
B. B-complex with C + KCI
C. NaCl + KCI
D. KCI + dextrose
E. None of the above
1
KEY (CORRECT ANSWERS)
1. A
2. C
3. A
7. B
8. C
9. A
10. C
14. E
15. B
16. C
17. A
21. D
22. D
23. C
24. A
PHARMACY
TEST 1

4. C

5. B

11. C

12. D

18. C

19. A

6. D
13. B
20. C

25. E

EXAMINATION SECTION

DIRECTIONS:
Each question or incomplete statement is followed by several suggested answers or
completions. Select the one that BEST answers the question or completes the
statement. PRINT THE LETTER OF THE CORRECT ANSWER IN THE SPACE AT THE
RIGHT.
1. Which of the following medications should be questioned if prescribed
in a Spansule dosage form?
A. Compazine
B. Dexedrine
C. Orinase
D. Temaril
E. Feosol
2. A physician calls to ask the name of a commercially available
steroid/antibiotic topical product. How should the pharmacist reply?
A. Cordran
B. Cort-Dome
C. Cortef
D. Cortisporin
E. Cortril
3. Of the following, which antibiotic product has the LONGEST stability in
its oral liquid dosage form?
A. Ampicillin (Polycillin)
B. Cephalexin (Keflex)
C. Tetracycline (Achromycin)
D. Penicillin (Pfizerpen)
E. Colistin (Coly-Mycin S)
4. Declomycin is available in which of the following strengths?
A. 2-5-10 mg.
B. 50-100 mg.
C. 75-150-300 mg.
D. 100-300 mg.
E. None of the above

5. If a physician desires a diuretic in an oral liquid dosage form, the


pharmacist should suggest
A. cyclothiazide (Anhydron)
B. acetazolamide (Diamox)
C. chlorothiazide (Diuril)
D. chlorthalidone (Hygroton)
E. ethacrynic acid (Edecrin)
6. Which of the following variables associated with the formulation of an
oral tablet will have the LEAST effect on the ultimate bioavailability of the
active ingredient?
A. Tablet compression force
B. Hydrate form of the drug
C. Adjuvants of the tablet formulation
D. Melting point of the drug
E. Granule size and size distribution of the bulk drug powder
7. Absorption of a drug into and through the skin is known
A. active transport
B. intercutaneous absorption
C. percutaneous absorption
D. passive absorption
E. subcutaneous absorption

as

8. Which of the following gases is a suitable propellant in a


pharmaceutical aerosol intended for topical administration?
A. Methane
B. Freon
C. Hydrogen
D. Helium
E. Nitrous oxide
9. Synalar (fluocinolone acetonide) solution utilizes pro pylene glycol as
a(n)
A. buffering agent B. antioxidant
C. emulsifier D. preservative
E. vehicle
10. Benzyl alcohol is included in injectables as a(n)
A. suspending agent
B. emulsifier
C. vehicle
buffering agent

D. preservative

11. Which of the following auxiliary labels should be affixed to a filled


prescription bottle of Tinactin drops?
A. External use only
B. For the eye
C. For the nose
D. To be taken by mouth
Warm before using
12.
Dr. Martin

Mr. Fisher ,

7934 Minor Street

Today

Cordran Ointment
Full strength
60 grams
Apply at bedtime Cover with an occlusive dressing

E.

E.

Mr. Fisher asks the pharmacist what he should use for the "occlusive dressing" indicated
in the prescription
above. The pharmacist should advise him to use
A. a gauze pad
B. a sling
C. Saran wrap D. an Ace bandage E. adhesive tape
13. What SPECIAL instructions should be given to a patient taking
nitroglycerin tablets?
A. Care should be exercised since nitroglycerin is an explosive.
B. Do not take with milk or dairy products.
C. Tablets should be chewed very thoroughly.
D. Tablets should be stored in the original container
E. Do not take on empty stomach.

DIRECTIONS: This group of questions consists of five lettered head ings


followed by a list of numbered phrases. For each numbered phrase, select the
one heading which is most closely related to it.
(One heading may be used once, more than once, or not at all.)
A. Oral
B. Parenteral
C. Rectal
D. Inhalation
E. Sublingual
Given the routes of administration for medications above, select the MOST
appropriate route for each of the following examples.
14. Three grams of iopanoic acid (Telepaque) for scheduled roentgenography for gall
bladder visualization.
15. Home administration of 25 milligrams of promethazine HC1 (Phenergan) to stop
vomiting in a 6-year-old as directed by the physician.
16. Twenty milligrams cromolyn Na (Intal) as an adjunct in the management of a patient
with recurring, severe bronchial asthma.
17. Sodium nitroprusside (Nipride) for a patient in hypertensive crisis.
18. A customer wishes to purchase an athletic supporter for her 15-year-old
son. The pharmacist should explain
to the customer that proper fitting of
an athletic supporter is determined by
A. hip measurement
B. waist measurement
C. pouch size D. penis length
E. testicle size
19. A basal thermometer is used PRIMARILY to determine
A. rectal temperature
B. orgasm
C. ovulation
D. vaginal temperature
E. pregnancy
20. A 100-unit insulin syringe is color-coded
A. green
B. red
C. yellow

D. orange

E. blue

21. A physician prescribes a sterile solution of isotonic sodium chloride for


use in a patient's IPPB machine. The IPPB machine is used in the treatment of

A. emphysema
sinusitis

B. glaucoma

C. phlebitis

D. diabetes

E.

22. Which of the following is the international unit of x-rays or gamma


radiation?
A. Curie
B. Radionuclide
C. Roentgen
D. Nanogram
E. Rad
23. A cholecystographic procedure entails the use of a suitable radiopaque
material for the visualization of
A. renal tubules
B. peripheral arteries
C. gastrointestinal tract
D. gall bladder
E. urinary bladder
24. Elevated levels of serum creatinine is indicative of depressed
A. hepatic function
B. renal function C. myocardial contractility
D. endocrine
function
E. muscle reflex
25. Pancytopenia, an adverse drug reaction associated with chloramphenicol
therapy, is defined as
A. a reduction in the normal number of erythrocytes in the blood
B. a reduction in the normal number of platelets in the blood
C. the formation of immature, nucleated erythrocytes
D. an increase in the number of erythrocytes in the blood
E. a deficiency of all of the formed cellular elements of the blood
KEY (CORRECT ANSWERS)
1. C
2. D
3. C
4. C
5. C

6. D
7. C
8. B
9. E
10. D

11.
12.
13.
14.
15.

A
C
D
A
C

16. D
17. B
18. B
19. C
20. D

21. A
22. C
23. D
24. B
25. E

TEST 2
DIRECTIONS:
Each question or incomplete statement is followed by several sug gested answers or
completions. Sel ect the one that BEST answers the question or completes the
statement. PRINT THE LETTER OF THE CORRECT II
ANSWER IN THE SPACE AT THE RIGHT.
1. A patient asks for a recommendation for an OTC analgesic. A check of his
medication profile indicates that he is taking colchicine and Benemid
Which of the following is the BEST analgesic to recommend?

A. Bufferin
Anacin

B. Ascriptin

C. Aspirin

D. Datril

E.

2. A month ago, a patient taking 300 milligrams of Dilantin daily and 60


milligrams of phenobarbital daily was placed on 500 milligrams of Aldomet
four times daily. When he re turns to his physician for a follow-up
examination, the physician does not detect any decline in blood pressure and
calls for consultation
The pharmacist should tell the physician that the
A. dose of Aldomet is subtherapeutic and the dose should be increased
B. phenobarbital has induced enzymes metabolizing Aldomet
C. Dilantin displaces Aldomet from its binding site
D. onset of action of Aldomet is 6 weeks, therefore no response is expected for 2 more
weeks
E. patient should be evaluated for compliance with Aldomet
3. Cold sterilization is effectively achieved through the use of which size of
membrane filter?
A. 0.22 micron
B. 0.39 micron
C. 0.45 micron D. 0.57 micron
E. 0.68 micron
4. As aspirin suspension is degrading at a zero-order rate of 100 milligrams
per hour. If the original suspension contained 1 gram of ASA in suspension,
how much ASA would remain in 3 hours?
A. 125 mg.
B. 300 mg.
C. 500 mg. D. 700 mg.
E. 900 mg.
5. Regular insulin injection was buffered at pH 3.0 where the first-order rate
for its decomposition at room tem perature was determined to be 0.10 per
year. Assuming the original solution contained 80 units of insulin per milliliter,
how much insulin would remain per milliliter after 1 year?
A. 72 units
B. 64 units C. 40 units D. 20 units E. 8 units
6. Sodium cloxacillin (legopen) for solution should be reconstituted at the
time of dispensing and stored in a refrigerator for maximum stability. If
stored at room temperature, however, the half-life of the cloxacillin
in solution is only 10 days. If the original concentration of the solution was
250 milligrams per 5 milliliters,
how much cloxacillin will remain per 5 milliliters after storage at room
temperature for 20 days?
A. 175.0 mg.
mg.

B. 150.0 mg.

C. 125.0 mg.

D. 62.5 mg.

E. 25.0

DIRECTIONS: Questions 7 through 9 refer to the following prescription and the


label that was placed on the filled prescription vial.

For:
Address:

Linda Merton
791 Underhill Road Today
Rx:
Valium
Sig:
1 a. c.
Refill:

and

Rx #29345

No. 38
h. s.
p.r.n.

E. Howard, M.D. 629 Colby Avenue


The label below was as placed on the filled prescription vial for Linda Merton.
Virgina City PHARMACY
37 Linden Boulevard
AN 6254213
No. 29345
Dr. E. Howard
Linda Merton
Take one (1) four times a day after meals and at bedtime ...
Valium
Today
7. Based on the data above, the prescription CANNOT be dispensed for which
of the following reasons?
I.
The strength is needed before filling.
II.
The dosage form is needed before filling.
III.
The label is typed incorrectly.
The CORRECT answer is:
A. I only
B. II only
C. I and III
D. II and III
E. I, II, and
III
8. Valium is commercially available in which of the following dosage
strengths?
I. 2 mg.
II. 5 mg.
III. 10 mg.
The CORRECT answer is:
A. I only
B. II only
C. I and III
D. II and III
E. I, II, and III
9. Valium is commercially available in which of the following non-injectable
dosage forms?
I. Tablets
II. Capsules
III. Suppositories
The CORRECT answer is:
A. I only
B. II only
C. I and III
D. II and III
E. I, II, and III

Questions 10-11.
DIRECTIONS: For Questions 10 and 11 below, ONE or MORE of the responses given are
correct. Decide which of the responses is (are) correct. Mark in your answer space
A
B
C
D
E

if
if only
if only
if only
if only

1, 2, and 3 are correct;


1 and 2 are correct;
2 and 3 are correct;
1 is correct;
3 is correct.

10. Under appropriate conditions, sterilization using ethylene oxide may be


employed for which of the following?
1. Otic drops
2. Rubber catheters
3. Disposable
needles
11. Under appropriate conditions (i.e., 170 C, 1 hour),
dry heat may be employed for which of the following?
1. Heat-labile ophthalmic solutions
2. Plastic syringes

sterilization using
3. White petrolatum

12. Oliguria is defined as


A. urea in the urine
B. protein in the urine
C. profuse secretion of urine
D. scanty secretion of urine
E. sugar in the urine
13. The generic equivalent of Zephiran chloride solution is solution.
A. benzalkonium chloride
B. benzethonium chloride
C. cetylpyridinium chloride
D. ethylammonium hydrochloride
E. hexachlorophene hydrochloride
14. The warning label, "DO NOT use for more than 2 days or in the presence of
high fever or in infants or children under3 years of age unless directed by a
physician," must appear on the lable of
A. antacids
B. antidiarrheals
C. antitussives
D. antipyretics
E. analgesics
15. Patients who are being treated for hypertension should be cautioned
about self-medication with OTC products that contain
A. antihistamines
B. sympathomimetics
C. aluminum salts D. cough
suppressants E. sugar
16. Keto-Diastix is used to test urine for the presence of
A. occult blood
B. albumin and ketones
C. protein and glucose
D. glucose and ketones

E. bilirubin and urobilinogen


17. Which of the following is an abrasive soap?
A. Fostril
B. Acnomel
C. Clearasil
D. Oxycel
Susan Tilden
Remarks:
CHP
Date
3-5
3-5
4-6
4-6

Quantity

E. Brasivol

433 Haynes Street


History: Myocardial infarction
Patient

Physician

Susan
Susan
Susan
Susan

Jackson
Jackson
Jackson
Jackson

Drug
Digoxin 0.25 mg. daily
Coumadin 5 mg. daily
Digoxin 0.25 mg. daily
Coumadin 5 mg. daily

30
30
30
30

18. Mrs. Tilden, whose patient medication profile is shown above, brings
prescriptions for Atromid-S and Colace. Which of the following statements is
MOST significant?
A. Atromid-S increases the action of Coumadin.
B. Atromid-S is contraindicated for a patient with a
history of myocardial infarction.
C. Colace increases the action of Coumadin.
D. Colace decreases the action of Coumadin, E. Atromid-S decreases the response to Coumadin.
19. Which of the following ophthalmic preparations must be stored in a
refrigerator until dispensed to a patient?
A. Isopto-Carpine 4% ophthalmic solution
B. Stoxil ophthalmic solution
C. NeoDecadron ophthalmic solution
D. Floropryl ophthalmic solution
E. Sodium Sulamyd 10% solution with meth cellulose
Atropine SO4
Demerol HC1
Adm.
1.M.

q.a.m.

0.4 mg.
75.0 mg.
x 2 days

20. Which of the following will be true if the medications above are mixed in
the same syringe?
A. Meperidine hydrochloride only will precipitate from solution.
B. Both meperidine and atropine will precipitate from solution.
C. The meperidine will complex with atropine, rendering it not bioavailable.
D. The mixture will result in color formation and gas evolution.
E. Nothing will happen because the medications are compatible.
21. Aminophylline (theophylline ethylenediamine) in an intravenous solution
is compatible with
A. potassium chloride
B. vitamin B complex
C. ascorbic acid
D. promethazine hydrochloride

E. magnesium sulfate

22. For optimal stability, tetracyclines are formulated into which of the
following ointment bases?
A. Anhydrous lanolin
B. Aquaphor
C. Polyethylene glycol
D. Hydrophilic petrolatum
E. White petrolatum
23. The trademark name of penicillin V that is manufactured by Squibb is
A. Compocillin
B. PenVee
C. Sumycin D. Uticillin
E. Veetids
24. Upjohn's injectable container that separates the lyophilized powder from
its diluent by means of a
rubber diaphragm has which of the following trademarks?
A. Isoject
B. Lyovac
C. Mis-O-Vial
D. Tubex
E. Turbinaire
25. Patients should restrict their dietary intake of foods that contain tyramine
if they are given a prescription for
A. Eutonyl
B. Proloid
C. Benemid
C. Ascriptin
E.
Declomycin
KEY (CORRECT ANSWERS)
1.D
6.D
11.E
22. E

17.E

2.E
7.C
12.D
23. E

18.A

3.A
8.E
13.A

19.B
24. C 25. C

9.A
14.B

4.D
20.E

10.C
15.B

5.A
21. A

16.D

PHARMACOLOGY
EXAMINATION SECTION
DIRECTIONS: Each question or incomplete statement is followed by several suggested
answers or completions. Select the one that BEST answers the question or completes
the statement. PRINT THE LETTER OF THE CORRECT ANSWER IN THE SPACE AT THE
RIGHT.
1. Drugs may be excreted in unchanged form through the
I. lungs
II. skin
III. saliva
IV. kidneys
The CORRECT answer is:
A. I, III, IV
B. I, IV, V
C. II, III, IV, V
D. III, V

V. colon
E. All of the above

2 Which of the following statements are TRUE regarding the adrenergic


receptors?
I.
Activation of the B-receptor results in cardiac stimulation.
II.
Activation of the a-receptor results in vasoconstriction
III.
A number of effects of the catecholamines are mediated by altering tissue levels
of cAMP.
IV.
The enzyme adenylcyclase may be a receptor for certain adrenergic drugs.
V.
Increased tissue levels of cAMP occur invariably with B-adrenergic stimulation.
The CORRECT answer is:
A. I, II, III
B. I, III
of the above

3. Cholinergic crisis may be produced by


I.
neostigmine
II.
scopolamine
III.
organophosphate insecticides
IV.
pilocarpine
V.
Isoproterenol
The CORRECT answer is:
A. I, II, III
B. I, III, IV
C. I, IV

C. II, III, IV

D. II, IV, V

D. III, IV

E. All

E. III, IV, V

4. Which of the following effects characterize drugs used for preanesthetic


medication?
I.
Morphine and other narcotics tend to prolong the anesthetic state.
II.
The barbiturates may contribute to postanesthetic excitement, particularly in the
presence of pain.
III.
Atropine may cause cardiac acceleration.

IV.
Scopolamine tends to obscure eye signs.
V.
The phenothiazines predispose to vomiting.
The CORRECT answer is:
A. I, II, III, IV
B. I, III, IV, V
C. II, III, IV
IV, V

D. II, V

E. III,

5 Administration of a(n)----------causes dry mouth, a decrease in secretory


and motor activity of the gastrointestinal tract, mydriasis and difficulty in
accommodation.
I.
antimuscarinic drug
II.
ganglionic blocking agent
III.
anticholinesterase
IV.
B-adrenergic agonist
V.
indirect-acting adrenergic drug
The CORRECT answer is:
A. I, II
B. I, II, IVC. I, III

D. IV, V

E. Any of the above

6 Which of the following are characteristics of morphine, meperidine and methadone?


They
I.
cause depression of respiration
II.
cause constipation
III.
are effective in the relief of severe pain
IV.
cause mydriasis
V. are antagonized by nalorphine
The CORRECT answer is:
A. I, II, III, IV
B. I, II, III, V
C. I, II, IV, V
D. I, III, IV, V
E. II, III, IV, V
7 Which of the following are TRUE regarding the cephalosporin antibiotics?
They
I. are chemically related to penicillin G
II. have a broad spectrum of activity
III. are effective against penicillinase-producing microorganisms
IV. are possibly cross-allergenic with penicillin
V. are bacteriostatic
The CORRECT answer is:
A. I, II, III
B. I, III, IV
C. II, III, IV
D. II, V
E. III, IV, V
8. Which of the following determine the rate of passive diffusion of a drug
across the intestinal wall membrane?
I. Size of the drug molecule
II. Degree of ionization of the drug
III. Concentration of the drug in the intestine
IV. Degree to which the drug is bound to plasma proteins
The CORRECT answer is:
A. I, II, III
B. I, II, IV
C. I, III, IV
D. II, III, IV
E. All of the above

9. Which of the following statements describe the pharmacologic and


toxicologic properties of ethyl alcohol?
I. Its effects on behavior are the result of depression of higher cortical centers.
II. It aids in the conservation of heat, which explains the feeling of warmth that attends
its use.
III. Blood levels of 100 mg. % or higher are generally taken as medicolegal evidence of
drunkenness.
IV. Delirium tremens are regarded by some investigators as a physiologically disruptive
withdrawal syndrome.
V. Chronic gastritis and cirrhosis of the liver are common in the alcoholic, possibly
because of
nutritional deficiencies.
The CORRECT answer is:
A. I, II
B. I, II, V
C. I, III, IV
D. I, III, IV, V
E. IV, V
10. Which of the following penicillins are readily INACTIVATED by B-lactamase
(penicillinase)?
I. Penicillin G
II. Phenoxymethyl penicillinIII. Methicillin
IV. Ampicillin
V.
Cloxacillin
The CORRECT answer is:
A. I, II, III
B. I, II, IV
C. I, II, V
D. II, III, IV
E. III, IV, V
11. Which of the following characterize nitrous oxide?
I. Slow recovery
II. Good analgesia
III. Respiratory rate stimulation
IV. Poor skeletal muscle relaxation
V. Rapid induction in concentrations of 80%
The CORRECT answer is:
A. I, II, V
B. I, III, IV
C. II, IV, V
D. III, IV, V

E. All of the above

12. Which of the following compounds is a ganglionic blocking agent?


A. Atropine
B. Hexamethonium
C. Succinylcholine
D. Gallamine triethiodide
E. Diisopropylfluorophosphate
13. Administration of which of the following drugs is most likely to PROLONG
clotting time?
A. Morphine
B. Vitamin K

C. A barbiturate
D. Acetaminophen
E. Acetylsalicylic acid
14. If a narcotic agent were chosen for intravenous sedation,
which of the
following drugs should be available as the antagonist of choice?
A. Naloxone
B. Doxapram
C. Pentazocine
D. Fentanyl
E. Diphenhydramine

15. Which of the following is a frequent side effect of antihistamine therapy?


A. Anemia
B. Anuria
C. Diuresis
D. Drowsiness
E. Autonomic nervous system overactivity
16. Which of the following is a local anesthetic subject to inactivation by
plasma esterases?
A. Procaine
B. Lidocaine
C. Prilocaine
D. Mepivacaine
E. Bupivacaine
17. Methoxylation of which of the following compounds results in tie
formation of codeine?
A. Morphine
B. Codeinone
C. Oxycodone
D. Methylmorphine
E. Diacetylmorphine
18. Which of the following drugs is used principally for its diuretic action?
A. Aldosterone
B. Angiotensin
C. Meprobamate
D. Vasopressin
E. Chlorothiazide
19. Which of the following is a predictable result of
A. Mydriasis
B. Xerostomia
C. Bronchoconstriction
D. Lowered blood pressure
E. Dryness of the nasal mucous membranes

a-adrenergic blockade?

20 B1-adrenergic blocking agents will antagonize which of the following


actions of epinephrine?
A. Glycogenolysis
B. Cardiac acceleration
C. Vasoconstriction in gastrointestinal vasculature
D. Vasodilatation of skeletal muscle vasculature
E. Bronchodilation
21. Which of the following may be useful as an infiltrative local anesthetic in
a patient allergic to both amide and ester anesthetic derivatives?
A. N20
B. Bupivacaine
C. Phenylephrine
D. Diphenhydramine
E. Ethylaminobenzoate
22. Which of the following is an IRREVERSIBLE side effect resulting from
long-term administration of phenothiazine antipsychotics?
A. Sedation
B. Xerostomia
C. Infertility
D. Parkinsonism
E.
Tardive dyskinesia
23. Withdrawal of a patient from which of the following drugs is often
attended by changes in the electroencephalogram and convulsions?
A. Heroin
B. Cocaine
C. Morphine
D. Secobarbital
Methamphetamine

E.

24. Which of the following drugs applied topically would produce mydriasis of
short duration with no cycloplegia?
A. Atropine
B. Ephedrine
C. Homatropine
D. Isoproterenol
E. Diisopropylfluorophosphate
25. It is a general rule in using antibiotic combination therapy to NEVER
combine agents.
A. two bactericidal
B. two bacteriostatic
C. a bactericidal with a bacteriostatic
D. two analgesic
E. an analgesic with a bactericidal

1E

2.E
6.B
10.B

13. E
19. D

14. A
20. B

KEY (CORRECT ANSWERS)


3.B
7.B
8.E
11. C
12. B
15. D
16. A
17. A
21. D
22. E
23. D

4.A

5.A
9.D
18. E
24. B

25. C

TEST 2
DIRECTIONS Each question or incomplete statement is followed by several suggested
answers or completions. Select the one that BEST answers the question or completes
the statement. PRINT THE LETTER OF THE CORRECT ANSWER IN THE SPACE AT THE
RIGHT.
1. Administration of which of the following drugs INCREASES the likelihood of
toxic responses to digitalis?
A. Reserpine
B. Diazepam
C. Lidocaine
D. Spironolactone
E. Chlorothiazide
2. A 5 grain aspirin tablet contains approximately milligrams of the drug.
A. 75
B. 150
C. 325
D. 500
E. 700
3. Excitement and delirium are commonly present in stage of ether anesthesia
A. I
B. II
C. III, plane 1
D. III, plane 3
E. IV
4. Which of the following drugs is safe to administer to a patient who suffers
from atrial tachycardia, is allergic to quinine and has no evidence of
congestive heart failure?
A. Digitoxin
B. Quinidine
C. Epinephrine
D. Procainamide
E. Acetylcholine

5. Which of the following types of drug responses is LEAST


occurrence?
A. Side-effect
B. Idiosyncrasy
C. Tachyphylaxis
D. Therapeutic action
E. Overdosage toxicity

predictable in

6. Which of the following drugs has the LEAST anti-inflammatory activity?


A. Prednisone
B. Aldosterone
C. Prednisolone
D. Hydrocortisone
E. Triamcinolone

7. Which of the following drugs, when administered intravenously, is LEAST


likely to produce respiratory
depression?
A. Fentanyl
B. Diazepam
C. Thiopental
D. Meperidine
E. Pentobarbital
8. Which of the following is NOT a property of tetracycline antibiotics?
A. Absorption is impaired when taken with milk.
B. They predispose to monilial superinfection.
C. They form a stable complex with the developing tooth matrix.
D. They have a low tendency for sensitization, but a high therapeutic index
E. They are effective substitutes for penicillin in subacute bacterial endocarditis
prophylaxis.
9. Which of the following drugs is an analgesic and an antipyretic but does
NOT have significant anti-inflammatory properties?
A. Indomethacin
B. Phenylbutazone
C. Acetaminophen
D. Sodium salicylate
E. Acetylsalicylic acid
10. Following administration of 1% lidocaine, the patient feels faint and light
headed and begins to wheeze. The injection of 0.5 cc. of 1:1,000 epinephrine
subcutaneously will produce all of the following effects EXCEPT
A. sedation
B. bronchodilation

C. cardiac acceleration
D. increased systolic blood pressure
E. none of the above
11. The blood level of a drug reflects all of the following EXCEPT
A. absorption
B. protein binding
C. mechanism of action
D. amount of drug given
E. metabolism and excretion
12. Each of the following drugs is subject to hydrolysis by esterases EXCEPT
A. procaine
B. epinephrine
C. tetracaine
D. acetylcholine
E. succinylcholine

13 Excessive does of acetylsalicylic acid can produce each of the following


EXCEPT
A. tinnitus
B. hypothermia
C. hyperventilation
D. metabolic acidosis
E. gastrointestinal irritation
14 Activity of the hepatic drug metabolizing enzymes may be enhanced by
each of the following EXCEPT
A. dicumarol
B. meprobamate
C. phenobarbital
D. acetylcholine
E. cigarette smoking
15 Administration of tricyclic antidepressants may cause each of the
following EXCEPT
A. xerostomia
B. orthostatic hypotension
C. immediate relief of acute depression
D. side-effects similar to those of the phenothiazines
E. inhibition of norepinephrine reuptake by specific central nervous system adrenergic
neurons
16. Each of the following is a naturally occurring local
EXCEPT
A. endorphin
B. histamine
C. bradykinin
D. prostaglandin

mediator in man

E. tetrahydrocannabinol
17. Propranolol is CONTRAINDICATED in each of the following clinical
conditions EXCEPT
A. hypoglycemia
B. bronchial asthma
C. atrial tachycardia
D. congestive heart failure
E. preexisting A-V block
18. Each of the following drugs may result in blood dyscrasias EXCEPT
A. codeine
B. phenytoin
C. indomethacin
D. phenylbutazone
E. chloramphenicol
19. The prostaglandins produce all of the following pharmacologic actions
EXCEPT
A. pyrexia
B. uterine contraction
C. increased gastric secretion
D. increased capillary permeability
E. pain when injected intradermally
20. Which of the following is NOT characteristic of the thiazide diuretics?
They
A. increase renal excretion of sodium and chloride
B. increase renal excretion of potassium
C. exacerbate existing diabetes
D. cause hypokalemia
E. cause hypoglycemia
21. Which of the following is NOT true about acetaminophen?
It
A. is a non-prescription drug
B. is cross-allergenic with aspirin
C. possesses both analgesic and antipyretic effects
D. may induce methemoglobinemia at high doses
E. may be the pharmacologically active form of acetophenetidin (phenacetin)
22. Which of the following antibiotics is cross-allergenic with penicillin and
should NOT be administered to the penicillin-sensitive patient?
A. Ampicillin
B. Erythromycin
C. Clindamycin
D. Lincomycin
E. Tetracycline
23. Which of the following is NOT true regarding succinyl choline?
A. It is structurally related to acetylcholine.
B. It is generally administered intravenously.

C. It is metabolized by pseudocholinesterase.
D. Its most serious toxic effect is apnea.
E. It is antagonized by anticholinesterases.
24. The sulfonylurea hypoglycemic drugs are usually NOT useful in childhood
diabetes because
A. they are antagonized by ACTH
B. they have low therapeutic indices
C. few pancreatic islet cells are active
D. they inhibit the physiologic production of insulin
E. none of the above
25. Adrenal steroids are used successfully to treat all of the following
conditions EXCEPT
A. gastric ulcers
B. Addison disease
C. lupus erythematosus
D. rheumatoid arthritis
E. aphthous stomatitis
KEY (CORRECT ANSWERS)
1. E
8. E
14. D
20. E

2. C
7. B
9. C

6. B
13.B
15. C
21. B

3. B

4. D

10. A
16. E
22. A

17. C
23. E

5. B

11.C
18. A
24. C

12.B
19. C
25. A

PHARMACOLOGY EXAMINATION
TEST 1
DIRECTIONS: Each question or incomplete statement is followed by several suggested
answers or completions. Select the one that BEST answers the question or completes
the statement. PRINT THE LETTER OF THE CORRECT ANSWER IN THE SPACE AT THE
RIGHT.
1. Nitroglycerin dilates the coronary arteries in angina
A. decreasing the heart rate reflexly
B. increasing the metabolic work of the myocardium
C. direct action on smooth muscle in the vessel walls
D. increasing the effective refractory period in the atrium
E. blocking beta-adrenergic receptors

pectoris by.

2. The probable mechanism of the bacteriostatic action of sulfonamides


involves
A. disruption of the cell membrane
B. coagulation of intracellular proteins
C. reduction in oxygen utilization by the cells
D. inhibition of metabolism by binding acetyl groups
E. competition with para-aminobenzoic acid in folic acid synthesis

3. In angina pectoris, the MOST rapid and dependable relief will be provided
by
A. oral glyceryl trinitrate
B. oral isosorbide dinitrate
C. oral erythrityl tetranitrate
D. sublingual glyceryl trinitrate
E. oral pentaerythritol tetranitrate
4. The MOST probable cause for a serious toxic reaction to
is
A. psychogenic
B. deterioration of the anesthetic agent
C. hypersensitivity to the vasoconstrictor
D. hypersensitivity to the local anesthetic
E. excessive blood level of the local anesthetic

a local anesthetic

5. The germicidal action of benzalkonium chloride is rapidly REDUCED in the


presence of
A. soap
B. water
C. fluorides
D. inorganic matter
E. cationic detergents
6. Amide-type local anesthetics are metabolized in the
A. serum
B. liver
C. spleen
D. kidneys
E. axoplasm
7. A patient with grand mal epilepsy would likely be under treatment with
A. meprobamate
B. pentobarbital
C. trimethadione
D. ethosuximide
E. phenytoin
8. Phenothiazines are used to
A. produce muscle relaxation
B. alter psychotic behavior
C. suppress coughing
D. produce analgesia
E. produce hypnosis
9. Cardiovascular collapse elicited by a high circulating
anesthetic may be caused by
A. syncope
B. vagal stimulation
C. histamine release
D. myocardial depression

dose of a local

E. medullary stimulation
10. The PRINCIPAL central action of caffeine is on the
A. cerebral cortex
B. corpus callosum
C. hypothalamus
D. spinal cord
E. medulla
11. Digitoxin is effective in the treatment of cardiac failure because it
A. is primarily a diuretic
B. reduces the ventricular rate
C. decreases abnormal cardiac rhythms
D. produces peripheral vasoconstriction
E. has a positive cardiac inotropic action
12. Quinidine is used to treat
A. hypertension
B. angina pectoris
C. atrial fibrillation
D. ventricular fibrillation
E. congestive heart failure
13. Diazepam is preferred to barbiturates as an antianxiety agent because
diazepam
A. produces no sedation
B. is a very short-acting drug
C. is substantially less expensive
D. does not potentiate CNS depressants
E. has less addiction potential
14. In general anesthesia, the LAST part of the CNS to be
A. medulla
B. cerebrum
C. midbrain
D. cerebellum
E. spinal cord
15. The only local anesthetic which increases the pressor
norepinephrine is

depressed is the

activity of both epinephrine and

A. cocaine
B. procaine
C. dibucaine
D. lidocaine
E. mepivacaine
16 The penetration of a local anesthetic into nervous tissue is a function of
the
A. length of the central alkyl chain
B. lipid solubility of the ionized form
C. lipid solubility of the unionized form
D. ester linkage between the aromatic nucleus and the alkyl chain
E. amide linkage between the aromatic nucleus and the alkyl chain

17. The antagonist of choice in the treatment of opioid


A. naloxone
B. nalorphine
C. pentazocine
D. levallorphan
E. propoxyphene

overdosage is

18. Inhalation general anesthetics possess relatively LOW therapeutic indices,


but their clinical safety is
greatly INCREASED by
A. rapid renal excretion
B. rapid liver metabolism
C. extreme ease in reversing tissue concentration
D. ready reversal of effects by antagonists
E. none of the above
19. Therapy with nonendocrine antineoplastic agents is
A. mutagenicity
B. bone marrow depression
C. decreased immune response
D. gastrointestinal and oral ulcerations
E. all of the above

associated with

20. Cardiac arrhythmias are MOST commonly seen during administration of


A. thiopental
B. halothane
C. ethyl ether
D. nitrous oxide
E. none of the above
21. The drug of choice for initial therapy for mild hypertension is
A. reserpine
B. guanethidine
C. phenobarbital
D. chlorothiazide
E. alpha-methyldopa
22. The inorganic ion which is primarily implicated in hypertension is
A. sodium
B. fluoride
C. potassium
D. magnesium
E. ammonium
23 The MOST common clinical cause of bacterial resistance is
A. improper antibiotic selection
B. faulty gastrointestinal absorption of antibiotics
C. use of antibiotics when they are not indicated
D. failure to use culture and sensitivity tests
E. none of the above

24. Neostigmine can stimulate denervated skeletal muscle because it


A. is a congener of acetylcholine
B. is a competitive blocking agent
C. has no effect on acetylcholinesterase
D. is more potent than diisopropylfluorophosphate
E. is capable of acting directly on the end-plate
25. Propranolol is of value in treating angina pectoris because it
A. has a direct action on vascular smooth muscle
B. blocks autoregulatory mechanisms in the heart
C. inhibits oxygen metabolism in cardiac cells
D. provides relief within seconds of an acute anginal attack
E. prevents chronotropic responses to endogenous epinephrine emotions and exercise
KEY (CORRECT ANSWERS)
1. C
6.B
7.E
13. E
18. C
24. E

2.E
14. A
19. E
25. E

8.B

3.D
15. A
20. B

9.D
16. C
21. D

4.E
10.A
17. A
22. A

5.A
11. E

12. C

23. C

TEST 2
DIRECTIONS: Each question or incomplete statement is followed by several
suggested answers or completions. Select the one that BEST answers the
question or completes the statement. PRINT THE LETTER OF THE CORRECT
ANSWER IN THE SPACE AT THE RIGHT.
1. The signature portion of the prescription contains
A. the full personal signature of the doctor
B. the drug name plus the unit dosage
C. instructions to the pharmacist concerning dispensing the drug
D. instructions to the patient regarding taking the medication
E. age, sex and address of the patient
2. The oral contraceptives exert their primary effect by
A. inhibiting ovulation
B. spermatocidal action
C. blocking implantation
D. decreasing uterine motility
E. stimulating FSH release from the pituitary

3. Amphetamine acts by
A. promoting storage of the mediator
B. causing a rapid release of the mediator
C. causing a slow depletion of the mediator
D. combining with a receptor substance on the effector cell
E. interfering with the response of the receptor to the mediator
4. The drug MOST commonly used to treat severe mental depression is
A. sodium
B. imipramine
C., chlorpromazine
D. tranylcypromine
E. dextroamphetamine
5 The mechanism of the antipyretic action of salicylates probably results
from
A. inhibition of prostaglandin synthesis in the CNS affecting hypothalamic temperature
regulation
B. inhibition of bradykinin in the periphery leading to sweating
C. depression of oxidative enzymes leading to decreased heat production
D. suppression of cholinergic mediators in the hypothalamus
E. stimulation of norepinephrine in the hypothalamus
6. Idiosyncracies to drugs are related to
A. species
B. genetic factors C. age of the patient
of the above

D. allergies

E. all

7 Gastric acid secretion has been shown to be MOST effectively reduced with
the use of
A. adrenal stenosis
B. anticholinergic drugs
C. serotonin antagonists
D. H1 histamine receptor antagonists
E. H2 histamine receptor antagonist
8 The ratio of the median lethal dose (LD50) to the median effective dose
(ED50) is the
A. morbidity index
B. mortality index
C. anesthetic ratio
D. therapeutic index
E. none of the above
9. In a patient being treated with an oral anticoagulant, thrombin solution
(applied topically) would be an
effective accelerator of the blood clotting process because
A. thrombin promotes fibrinolysis

B. thrombin combines with the anticoagulant and prevents the local actions of the drug
C. thrombin antagonizes those effects of the anticoagulant resulting in a decreased
synthesis of plasma
factor VII by the liver
D. the effect of thrombin is independent of whether thrombin exists in the plasma
E. thrombin acts directly to accelerate the formation of plasma thromboplastin
10. The sole therapeutic advantage of penicillin V over penicillin G is
A. greater resistance to penicillinase
B. broader antibacterial spectrum
C. more reliable oral absorption
D. slower renal excretion
E. none of the above
11. The use of vasoconstrictors in combination with local anesthetics is
CONTRAINDICATED in a dental patient
A. with angina pectoris
B. who is hypertensive
C. who has had rheumatic fever
D. who has had a myocardial infarction
E. with Parkinson disease who is on levodopa therapy
12. The blockade produced by d-tubocurarine is characterized by a(n)
A. defect in the conduction mechanism of the nerve
B. decrease of the end-plate potential to 30 percent or less of its original value
C. failure of the axon terminal to synthesize acetylcholine
D. failure of the axon terminal to release acetylcholine
E. initial stimulation, followed by a stabilization, of the postsynaptic membrane

13. A distinct advantage that tetracyclines have over penicillins is that


tetracyclines
A. have no side effects
B. do not cause superinfections
C. are safer to use during pregnancy
D. have a wider range of antibacterial activity
E. produce higher blood levels faster after oral administration
14. The occupational theory of drug-receptor interaction states that
I. the magnitude of the drug response is proportional to the number of receptors
occupied
II. a partial agonist has intrinsic activity but no affinity for the receptor site
III. an antagonist drug has affinity but no intrinsic activity
IV. the rate at which the drug-receptor complex associates and dissociates determines
drug efficacy
V. the degree of drug action is dependent on the law of mass action
The CORRECT answer is:
A. I, II, III
B. I, III, V
C. II, III, IV
D. II, V
E. III, IV, V

15. Therapeutic effects of aspirin include


I. analgesia
II. tranquilization
III. pyretic action
IV. anti-inflammatory action
V. antirheumatic action
The CORRECT answer is:
A. I, II, III
B. I, III, IV
C. I, IV, V

D. II, III, IV

E. II, IV, V

16. The beneficial effects of digitalis in congestive heart failure result in part
from the fact that digitalis
causes a(n)
I. decrease in end-diastolic volume
II. decrease in end-diastolic pressure
III. increase in stroke volume and cardiac output
IV. decrease in central venous pressure
V. decrease in rate of the heart where tachycardia exists
The CORRECT answer is:
A. I, II, III
B. I, III
C. III, IV
D. V only E. All of the
above

17. MOST anaphylactic reactions to penicillin occur


I. when the drug is administered orally
II. in patients who have already experienced an allergic reaction to the drug
III. in patients with a negative skin test to penicillin allergy
IV. when the drug is administered parenterally
V. within minutes after drug administration
The CORRECT answer is:
A. I, II, IV
B. II, III, IV
C. II, IV, V
D. II, V
E. III, IV, V
18. Blood-alcohol level is determined primarily by
I. rate of metabolism
II. body weight
III. mind-set
IV. presence of food in the stomach
V. rate of urinary excretion

The CORRECT answer is:


A. I, II, III
B. I, II, IV

C. I, IV, V

D. II, III, IV

E. III, IV, V

19. Based on its known mechanism and sites of action, scopolamine should
theoretically be useful in
I. treatment of peptic ulcer
II. providing euphoria and amnesia prior to surgery
III. relieving bronchoconstriction
IV. relieving some of the symptoms of Parkinson's disease
V. visualization of the retina
The CORRECT answer is:
A. I, II, IV, V
B. I, II, V
C. I, III
D. II, V
E. All of the above
20. Drugs which are additive with or potentiate the effects
include
I. methacholine
II. scopolamine
III. pralidoxime
IV. neostigmine
V. pilocarpine
The CORRECT answer is:
A. I, II, III
B. I, III, IV
C. I, IV, V
D. II, IV, V

of acetylcholine

E. III, IV, V

21. After pretreatment with phentolamine, intravenous administration of epinephrine


should result in
I. relaxation of bronchial smooth muscle
II. positive chronotropic and inotropic effects
III. splanchnic vasoconstriction
IV. dilation of skeletal muscle vascular beds
V. secretion of a mucoid viscous saliva
The CORRECT answer is:
A. I, II
B. I, II, IV
C. I, IV, V
D. III, IV E. III, IV, V

22. Therapy with reserpine may result in


I. an increase in salivary secretion
II. a decrease in heart rate
III. nasal congestion
IV. sedation
V. mental depression
The CORRECT answer is:
A. I, II, III
B. I, III
C. II, IV, V

D. IV, V

E. All of the above

23. Important steps in the treatment of barbiturate poisoning include


I. maintaining an open airway
II. increasing the input of afferent stimuli
III. maintaining respiration

IV. administering a narcotic antagonist


V. administering a central nervous system stimulant
The CORRECT answer is:
A. I, II
B. I, II, III, V
C. I, III

D. II, V

E. IV only

24. Therapeutic doses of morphine administered intramuscularly may produce


I. constipation
II. euphoria
III. dysphoria
IV. mental clouding
V. decreased response to pain
The CORRECT answer is:
A. I, II
B. I, II, IV
C. I, IV, V
D. III, IV, V
E. All of the above
25. Enhancement of the specificity of drug action may be achieved by
I. structural manipulation of the drug molecule
II. altering the dose of the drug
III. altering the route of administration of the drug
IV. administering the drug with another agent which specifically antagonizes certain of
the drug's effects
The CORRECT answer is:
A. I, II, III
B. I, IV
C. II, IV
D. IV only
E. All of the above
1.D
8.D
11.E
16.E
20.C
25. B

2.A
9.D
12.B
17.C
21. B

KEY (CORRECT ANSWERS)


3.B
4.B
5.A
10.C
13.D
14.B
15.C
18.B
19.E
22. E
23. C
24. E

6.E

7.E

PHARMACOLOGY
EXAMINATION SECTION
TEST 1
DIRECTIONS: Each question or incomplete statement is followed by several
suggested answers or completions. Select the one that BEST answers the
question or completes the statement. PRINT THE LETTER OF THE CORRECT
ANSWER IN THE SPACE AT THE RIGHT.
1. The MOST useful agent in resuscitation is
A. oxygen
B. an endotracheal tube
C. a respiratory stimulant
D. aromatic spirits of ammonia

E. 1:1000 epinephrine for injection


2. The PRINCIPAL danger associated with the use of nitrous oxide anesthesia in
concentrations exceeding 80% is

A. hypoxia
B. renal damage
C. liver damage
D. vomiting and nausea
E. irritation of the respiratory tract
3. Antiarrhythmic drugs, such as quinidine, suppress certain cardiac
arrhythmias by
A. stimulating the B-adrenergic receptor
B. suppressing cardiac ATP-ase activity
C. increasing ectopic pacemaker activity
D. increasing the refractory period of cardiac muscle
E. none of the above
4. Procaine differs from lidocaine in that
A. procaine is a p-aminobenzoic acid ester and lidocaine is not
B. lidocaine is a meta-aminobenzoic acid ester and procaine is not
C. the duration of action of procaine is longer than that of an equal total dose of
lidocaine
D. procaine hydrochloride is metabolized into diethylaminoethanol and benzoic acid
E. none of the above
5. A paralyzing dose of succinylcholine will initially elicit
A. CNS stimulation
B. CNS depression
C. muscle fasciculation
D. decreased salivation
E. extrapyramidal reactions
6. The agent of choice in the immediate management of anaphylactic crisis is
A. isoproterenol
B. norepinephrine
C. acetylcholine
D. epinephrine
E. dopamine
7. When the combined action of two drugs is GREATER than
individual actions, this is
A. induction
B. synergism
C. idiosyncrasy
D. hypersensitivity
E. cumulative action
8. Alpha or B-adrenergic blocking drugs act by
A. inhibiting synthesis of norepinephrine
B. increasing the metabolism of norepinephrine
C. competitive inhibition of postjunctional adrenergic receptors
D. a local anesthetic effect on the adrenergic nerve terminal

the sum of their

E. depleting norepinephrine from adrenergic nerve terminals


9. When an attempt is made to distinguish between habituation and
addiction, the GREATEST importance is usually given to the presence or
absence of
A. harmfulness of the drug
B. psychic dependence on the drug
C. physical dependence on the drug
D. the development of tolerance to the drug
E. acceptability of the drug in our society
10. For treating MOST oral infections, penicillin V is preferred to penicillin G
because penicillin V
A. is less allergenic
B. is less sensitive to acid degradation
C. has a greater gram-negative spectrum
D. has a longer duration of action
E. is bactericidal, whereas penicillin G is not
11. The use of epinephrine for local hemostasis during surgery might result
in
A. a drastic drop in blood pressure
B. an acute asthmatic attack
C. cardiac arrhythmia
D. hypoglycemia
E. any of the above
12. Dimercaprol (BAL) is an effective antidote for mercury poisoning because
it
A. produces emesis
B. increases renal tubular excretion of mercury
C. forms a stable complex with mercury and allows it to be excreted as an inactive
compound
D. causes mercury to be deposited in tissues in a nontoxic form
E. none of the above
13. Oral contraceptives prevent conception primarily by
A. inducing abortion
B. preventing ovulation
C. a spermatocidal effect
D. causing a cessation of menses
E. preventing implantation of the fertilized ovum in the endometrial wall
14. The mechanism of action of reserpine is to
A. inhibit monoamine oxidase
B. inhibit catechol-O-methyltransferase
C. prevent re-uptake of norepinephrine by storage granules in the nerve terminal
D. block the passage of the nerve action potential in the postganglionic nerve fibers
E. stabilize the axon terminal membrane thus preventing release of norepinephrine
15. The activity of procaine is TERMINATED by
A. elimination by the kidney

B. storage in adipose tissue


C. metabolism in the liver only
D. metabolism in the liver and by pseudocholinesterase in the plasma
E. none of the above
16. Administration of ganglionic blocking agents will result in
A. miosis
B. diarrhea
C. copious salivation
D. orthostatic hypotension
E. enhanced activity of the parasympathetic nervous system
17. A patient receiving propranolol has an acute asthmatic attack while
undergoing dental treatment. The MOST useful agent for management of the
condition is
A. morphine
B. epinephrine
C. phentolamine
D. aminophylline
E. norepinephrine
18. Gastrointestinal upset and pseudomembranous colitis has been
prominently associated with
A. nystatin
B. cephalexin
C. clindamycin
D. polymyxin B
E. erythromycin
19 According to the theory that agonists and antagonists
occupy the
same receptor site, an effective antagonist should exhibit
intrinsic activity
and affinity.
A. high; high
B. low; low
C. high; low
D. no; high
E. low;
high
20. The action of the ultrashort-acting barbiturates is terminated primarily by
the process of
A. oxidation
B. redistribution
C. renal excretion
D. plasma protein binding
E. conjugation with sulfate

21. The phenomenon in which two drugs produce OPPOSITE effects on a


physiologic system but do NOT act at the same receptor site is
A. potentiation
B. chemical antagonism
C. competitive antagonism
D. physiologic antagonism
E. noncompetitive antagonism

22. The MOST serious and life-threatening blood dyscrasia


drug toxicity is
A. aplastic anemia
B. agranulocytosis
C. thrombocytopenia
D. hemolytic anemia
E. megaloblastic anemia

associated with

23. When digitalis is used in atrial fibrillation, the therapeutic objective is to


A. abolish cardiac decompensation
B. inhibit vagal impulses to the heart
C. decrease the rate of A-V conduction
D. increase the rate of cardiac repolarization
E. produce a decrease in the rate of atrial contraction
24. The U.S. Federal regulatory agency that has the responsibility for
determining which drugs are to be
sold by prescription only is the
A. Federal Trade Commission
B. Internal Revenue Department
C. Food and Drug Administration
D. Bureau of Narcotics and Dangerous Drugs
E. Drug Enforcement Administration of the Department of Justice
25. Inhaled ammonia acts against syncope by
A. direct stimulation of the vasomotor center
B. direct stimulation of the respiratory center
C. releasing prostaglandins that modulate pain fibers
D. irritating the sensory endings of the trigeminal nerve
E. an analeptic response resulting from binding with a plasma co-factor
KEY (CORRECT ANSWERS)
1.
2.
3.
4.
5.

A
A
D
A
C

11. C
12. C
13. B
14. C
15. D

6.
7.
8.
9.

D
B
C
C

16. D
17. D
18. C
19. D

20. B

21. D

22. A

23. C

24. C
10. B

25. D

TEST 2
DIRECTIONS: Each question or incomplete statement is followed by several suggested
answers or completions. Select the
one that BEST answers the question or completes the statement. PRINT THE LETTER OF THE
CORRECT ANSWER IN

THE SPACE AT THE RIGHT.


1. Cimetidine is used therapeutically to
A. stimulate respiration
B. protect against anaphylaxis
C. decrease gastric acid secretion
D. hasten excretion of barbiturates
E. dilate smooth muscles of the broncioles
2. Anti-inflammatory agents, such as aspirin, interfere with hemostasis by
A. activating antithrombin
B. preventing vasoconstriction
C. inhibiting thrombin generation
D. inhibiting platelet aggregation
E. inhibiting polymerization of fibrin
3. Penicillin's effectiveness against rapidly growing cells is primarily due to its
effect on
A. protein synthesis
B. cell wall synthesis
C. nucleic acid synthesis
D. chelation of metal ions
E. cell membrane permeability
4. Sulfonamides and trimethoprim are synergistic bacteriostatic agents because
in bacteria they
A. both inhibit folic acid synthesis
B. interfere sequentially with folinic acid production
C. are both antimetabolites of p-aminobenzoic acid
D. are both inhibitors of dihydrofolic acid reductase
E. are both transformed in vivo into a single active compound
5 The mechanism of action of prazosin, an antihypertensive agent, is to
A. block B-adrenergic receptors
B. inhibit formation of angiotensin II
C. inhibit nerve-induced release of norepinephrine
D. stimulate central inhibitory a-adrenergic receptors
E. inhibit the postsynaptic action of norepinephrine on vascular smooth muscle
6 The more rapid onset of action of local anesthetics in
the
A. slightly lower pH of small nerves
B. greater surface-volume ratio of small nerves
C. increased rate of penetration resulting from depolarization
D. smaller nerves usually having a higher threshold
E. none of the above

small nerves is due to

7 The antipsychotic effects of the phenothiazines are


probably the result of
A. release of serotonin in the brain
B. release of norepinephrine in the brain
C. blockade of dopaminergic sites in the brain
D. prevention of the release of norepinephrine from brain neuron terminals

E. increase in the dopamine content of the cerebral cortex


8. Lithium carbonate is particularly effective in treating
A. parkinsonism
B. hypertension
C. schizophrenia
D. acute anxiety
E. manic-depressive psychosis
9. Small doses of barbiturates and morphine depress respiration primarily by
A. a specific effect at myoneural junctions of phrenic and intercostal nerves
B. a parasympathomimetic action
C. rendering the respiratory center less sensitive to changes in CO2
D. rendering the aortic chemoreceptor system insensitive to 02
E. inhibiting the Hering-Breuer reflex
10. Of the following, one of the MOST effective treatments currently available in
the U.S. for most patients
suffering from parkinsonism involves oral administration of
A. dopamine
B. amantadine
C. benztropine
D. levodopa alone
E. levodopa plus carbidopa
11. Currently, the drug of choice for status epilepticus is
A. diazepam
B. phenytoin
C. paraldehyde
D. paramethadione
E. chlordiazepoxide
12. A nonsteroidal, anti-inflammatory agent that appears to produce fewer
gastrointestinal disturbances than high doses of aspirin is
A. ibuprofen
B. probenecid
C. pentazocine
D. acetaminophen
E. phenylbutazone
13. Compared with the parent compound, biotransformation usually results in
drugs that are
A. more active
B. more ionized
C. less water soluble
D. more lipid soluble
E. none of the above

14. The CONTRAINDICATION to prescribing phenobarbital to a patient on


anticoagulant therapy is based on the ability of phenobarbital to cause
A. enzyme induction
B. gastric irritation
C. chemical antagonism

D. platelet destruction
E. physiologic antagonism
15. On oral administration, the blood level of a drug may be influenced by
A. body weight
B. presence of food in the stomach
C. quantity of the drug ingested
D. formulation of the drug
E. all of the above
16. When comparing drugs with respect to intensity of response, the drug that
produces the GREATEST MAXIMUM effect is the one with the HIGHEST
A. affinity
B. potency
C. efficacy
D. therapeutic index
E. dose
17. Hepatic "first-pass" metabolism of drugs is MOST significant after
administration
A. orally
B. rectally
C. sublingually
D. intravenously
E. subcutaneously
18. The therapeutic effect of the salicylates is explained on the basis of the ability
of the drug to
A. activate autonomic reflexes
B. uncouple oxidative phosphorylation
C. inhibit the synthesis of prostaglandins
D. increase the metabolic breakdown of prostaglandins
E. competitively antagonize prostaglandins at the receptor site
19. The use of a combination of an antipsychotic agent and a Schedule II narcotic
in pain control is referred to as
A. hyperalgesia
B. psychoanalgesia
C. relative analgesia
D. Stage III anesthesia
E. neuroleptic analgesia
20. Which of the following endocrine disorders may predispose a patient to
exaggerated responses to epinephrine in local anesthetic solutions?
A. Hyperparathyroidism
B. Hyperthyroidism
C. Hypogonadism
D. Cushing syndrome
E. Addison disease
21. The cardiac glycosides will REDUCE the concentration of which ion in an
active heart muscle?
A. Sodium
B. Bromide

C. Calcium
D. Chloride
E. Potassium
22. Which of the following acts by antagonizing cholinesterase?
A. Atropine
B. Muscarine
C. Neostigmine
D. Pilocarpine
E. Acetylcholine
23 Abuse of which of the following drugs is likely to
to nutritional deficiencies?
A. LSD
B. Cocaine
C. Alcohol
D. Barbiturate
E. Amphetamine

produce the GREATEST damage due

24. Which of the following drugs acts by inhibiting renal reabsorption of sodium?
A. Urea
B. Chlorothiazide
C. Theophylline
D. Digitalis glycosides
E. Procainamide
25. Organophosphate insecticides and nerve gases inhibit the action of
A. adenylate cyclase
B. monoamine oxidase
C. phosphodiesterase
D. acetylcholinesterase
E. carbonic anhydrase
KEY (CORRECT ANSWERS)
1.C
2.D
3.B
4.B
5.E

11.A
12.A
13.B
14.A
15.E

6.B
7.C
8.E
9.C
10.E
21. E
22. C
23. C
24. B
25. D

16.C
17.A
18.C
19.E
20.B

PHARMACOLOGY
EXAMINATION SECTION
TEST 1

DIRECTIONS Each question or incomplete statement is followed by several suggested


answers or completions. Select the one that BEST answers the question or completes the
statement. PRINT THE LETTER OF THE CORRECT ANSWER IN THE SPACE AT THE RIGHT.
1. The following properties common to all drugs of abuse are that they
A. have the ability to produce physical dependence
B. cause rapid development of tolerance
C. have the ability to elicit withdrawal symptoms upon discontinuance
D. have the ability to change one's mood and sensory perceptions
E. none of the above
2. Which of the following antibiotics should be considered the drug of choice in
the treatment of infection caused by a penicillinase-producing staphylococcus?
A. Neomycin
B. Ampicillin
C. Tetracycline
D. Penicillin V
E. Dicloxacillin
3. Which of the following antibiotics may be cross-allergenic with penicillin?
A. Neomycin
B. Cephalexin
C. Clindamycin
D. Erythromycin
E. All of the above
4. Which of the following is TRUE regarding the mechanism of action of local
anesthetics? They
A. usually maintain the nerve membrane in a state of hyperpolarization
B. prevent the generation of a nerve action potential
C. maintain the nerve membrane in a state of depolarization
D. prevent increased permeability of the nerve membrane to potassium ions
E. interfere with intracellular nerve metabolism
5. Which of the following statements MOST accurately describes the effectiveness
of action of methyldopa? It
A. causes marked cardiac slowing
B. directly relaxes vascular smooth muscle
C. causes rapid depletion of norepinephrine from adrenergic nerve terminals
D. causes formation of a false transmitter which is released at vascular smooth muscle
E. produces a false transmitter, the effect of which is primarily at central nuclei
6. Which of the following effects of nitroglycerin are antagonized by propranolol?
A. Increase in heart rate
B. Relaxation of arterial smooth muscle
C. Relaxation of nonvascular smooth muscle
D. Decrease in left ventricular end diastolic pressure
E. None of the above
7. Which of the following drugs is BEST to administer after
organophosphate cholinesterase inhibitor?
A. Atropine
B. Phenytoin
C. Pralidoxime
D. Propantheline

poisoning by an

E. Phenobarbital
8. Which of the following statements is TRUE regarding streptomycin? It
A. is bactericidal
B. has a gram-positive spectrum
C. is usually administered orally
D. disrupts bacterial cell membranes
E. is associated with a low incidence of bacterial resistance
9. Which of the following drugs chelates with calcium?
A. Erythromycin
B. Polymyxin B
C. Tetracycline
D. Penicillin G
E. Chloramphenicol
10. Which of the following has the GREATEST sodium-retaining effect in comparison with
cortisone?
A. Prednisone
B. Aldosterone
C. Prednisolone
D. Triamcinolone
E. Dexamethasone
11. Morphine and its surrogates produce their MAJOR effects
for
A. prostaglandin
B. histamine
C. bradykinin
D. enkephalin
E. 5-hydroxytryptamine
12. Which of the following plasma proteins has the GREATEST
A. Albumin
B. Fibrinogen
C. Hemoglobin
D. Gamma globulin
E. Beta lipoprotein

by activating the receptors

ability to bind drugs?

13. Which of the following antibiotics is the substitute of choice for penicillin in the
penicillin-sensitive patient?
A. Bacitracin
B. Erythromycin
C. Tetracycline
D. Chloramphenicol
E. Enderin
14. Which of the following prescriptions CANNOT be repeated
A. ASA, 300 mg.
B. Chloral hydrate, 500 mg.
C. Codeine phosphate, 30 mg.

without written authority?

D. Diazepam, 10 mg.
E. ASA, 300 mg., with caffeine, 30 mg., and codeine, 30 mg
15. Which of the following drugs is NOT used as an analgesic?
A. Morphine
B. Nalorphine
C. Meperidine
D. Levorphanol

E. Alphaprodine
16. Which of the following is NOT a property of diphenhydramine? It
A. relieves bronchospasm induced by histamine
B. blocks the depressor effect of isoproterenol
C. inhibits wheal formation after intracutaneous histamine
D. inhibits contractions of isolated intestine caused by histamine
E. reduces anaphylaxis induced by horse serum in sensitized guinea pigs
17. Which of the following is NOT true regarding coumarin anticoagulants?
A. Administered orally
B. Slow onset of action
C. Antimetabolites of vitamin K
D. Highly bound to plasma proteins
E. Differ in their mechanisms of action
18. Which of the following is NOT true regarding acetaminophen? It
A. has antipyretic properties
B. may induce methemoglobinemia
C. can be combined with codeine
D. has anti-inflammatory properties
E. is not cross-allergenic with aspirin
19. Which of the following statements does NOT characterize pentazocine?
A. It is equianalgesic with codeine.
B. It is a partial opioid antagonist.
C. Its abuse potential is less than that of heroin.
D. It may induce dysphoria and mental aberrations.
E. It is effective only on parenteral administration.
20. Which of the following statements is NOT true regarding barbiturates?
They
A. possess significant analgesic properties
B. possess serious drug dependence potential
C. vary in degree of lipid solubility
D. possess anticonvulsant properties
E. suppress REM sleep
21. Epinephrine, when administered intravenously in high dose, would NOT be
expected to
A. increase liver glycogenolysis
B. cause bronchiolar constriction
C. produce a rise in blood pressure
D. evoke extrasystoles in the heart
E. produce restlessness and anxiety
22. A local anesthetic injected into an inflamed area will NOT give maximum
effects because
A. the pH of inflamed tissue inhibits the release of the free base
B. the drug will not be absorbed as rapidly because of the decreased blood supply

C. the chemical mediators of inflammation will present a chemical antagonism to the


anesthetic
D. prostaglandins stabilize the nerve membrane and diminish the effectiveness of the
local anesthetic
E. of local shock
23. Each of the following drugs can be used in the prevention or treatment of
angina pectoris EXCEPT
A. digitalis
B. propranolol
C. nitroglycerin
D. isosorbide dinitrate
E. pentaerythritol tetranitrate
24. All of the following drugs are useful in the treatment of hypertension
EXCEPT
A. ephedrine
B. reserpine
C. methyldopa
D. thiazide diuretics
E. none of the above
25. All of the following agents are metabolized mainly by hydrolysis of the
ester linkage EXCEPT
A. procaine
B. lidocaine
C. piperocaine
D. chloroprocaine
E. metabutethamine
KEY (CORRECT ANSWERS)
1.
2.
3.
4.
5.

D
E
B
B
E

6. A
7. C
8. A
9. C
10.B
21. B
22. A
23. A
24. A
25. B

11.D
12.A
13.B
14.C
15.B
16.B
17.E
18.D
19.E
20.A

DIRECTIONS: Each question or incomplete statement is followed by several suggested


answers or completions. Select the one that BEST answers the question or completes
the statement. PRINT THE LETTER OF THE CORRECT ANSWER IN THE SPACE AT THE
RIGHT.
1. The central actions of ethyl alcohol are synergistic with all of the following
EXCEPT
A. caffeine
B. diazepam
C. meperidine
D. barbiturates
E. phenothiazines
2. All of the following are CONTRAINDICATIONS to the use of
contraceptives EXCEPT
A. hyperthyroidism
B. hypertension
C. history of breast cancer
D. undiagnosed genital bleeding
E. history of thromboembolic disease

oral

3. All of the following statements are TRUE regarding ethyl alcohol EXCEPT
that
A. it has diuretic properties
B. it initially stimulates the CNS
C. it dilates blood vessels of the skin
D. its emetic effects are due to local gastric as well as central effects
E. blood levels in excess of 500 mg. % usually result in coma and death
4 All of the following influence the rate of induction during anesthesia
EXCEPT
A. pulmonary ventilation
B. blood supply to the lungs
C. hemoglobin content of the blood
D. concentration of the anesthetic in the inspired mixture
E. solubility of the anesthetic in blood (blood-gas partition coefficient, Ostwald
coefficient)
5 Each of the following drugs is considered to be a direct acting
catecholamine EXCEPT
A. epinephrine
B. amphetamine
C. isoproterenol
D. norepinephrine
E. none of the above
6. All of the following pertain to general anesthesia induced by thiopental
EXCEPT
A. fast induction
B. decreased secretions
C. low therapeutic index
D. short duration of anesthesia
E. predisposition to laryngospasm
7. All of the following factors are significant determinants of the duration of
conduction block with amide-type local anesthetics EXCEPT the
A. pH of tissues in the area of injection

B. degree of vasodilatation caused by the local anesthetic


C. blood plasma cholinesterase levels
D. blood flow through the area of conduction block
E. concentration of the injected anesthetic solution
8. All of the following are TRUE regarding the intravenous agent ketamine
EXCEPT that it
A. produces hypnosis and true anesthesia
B. produces "dissociative anesthesia"
C. may produce tonic and clonic convulsions
D. is now used primarily in children
E. may produce delirium, hallucinations and schizoid reactions
9. Symptoms of digitalis toxicity include all of the following EXCEPT
A. extrasystoles
B. nausea and vomiting
C. yellow-green vision
D. A-V conduction block
E. decreased P-R interval
10. Each of the following drugs is used in the management of epilepsy
EXCEPT
A. phenytoin
B. ethosuximide
C. valproic acid
D. phenobarbital
E. diphenhydramine
11. All of the following statements are TRUE regarding the general aspects
of toxicology EXCEPT that
A. most drugs exert a single action
B. toxicity is both time- and dose-dependent
C. toxicity can be due to overdosage of a drug
D. toxicity can be anything ranging from nausea to death
E. for some drugs, even very minimal concentrations can be harmful
12. Central skeletal muscle relaxation produced by depression of the
polysynaptic reflex arcs is brought about by all of the following drugs EXCEPT
A. diazepam
B. meprobamate
C. lorazepam
D. tubocurarine
E. none of the above
13. Advantages of inhalation sedation with nitrous oxide include all of the
following EXCEPT
A. short recovery time
B. rapid onset of action
C. rapid biotransformation in the liver
D. ability to alter the depth of sedation from moment to moment
E. none of the above

14. All of the following statements are TRUE about anti neoplastic agents
EXCEPT that
A. generally they are not curative, although the success rate is improving
B. most agents possess a very high therapeutic index
C. generally they do not possess a selective effect on neoplastic cells
D. they are most effective in widely disseminated, non-solid tumors
E. they tend to suppress function of the bone marrow
15. All of the following are pharmacologic and toxicologic properties of aspirin
EXCEPT
A. tinnitus
B. analgesia
C. salicylism
D. antipyresis
E. suppression of the immune response
16. All of the following statements with regard to the autocoid prostaglandin
are correct EXCEPT that
A. it is a fatty acid
B. it is derived from essential amino acids
C. it is thought to be involved in the inflammatory process
D. it is thought to be involved in the reproductive process
E. its synthesis may be inhibited by nonsteroidal, anti-inflammatory agents
17. All of the following are examples of iatrogenic diseases EXCEPT
A. development of peptic ulcer with chronic use of aspirin
B. development of gout with use of thiazide diuretics
C. blood dyscrasias with use of chloramphenicol
D. pseudomembranous colitis from use of clindamycin
E. increased potassium excretion from hyperaldosteronism
18. Vasoconstricting drugs are included in local anesthetic solutions for all of the
following reasons EXCEPT to

A. reduce the toxicity of local anesthetics


B. prolong the duration of action of local anesthetics
C. reduce the rate of vascular absorption of local anesthetics
D. increase the concentration of the local anesthetic at the nerve membrane
E. prevent penetration of the local anesthetic at the nerve membrane
19. All of the following statements are TRUE regarding intra venous diazepam
EXCEPT that it
A. has a low therapeutic index
B. produces some degree of amnesia
C. can be employed to terminate most convulsive states
D. is locally irritating to tissue and may produce local thrombophlebitis
E. does not cause significant cardiovascular and respiratory depressions with normal
use
20. Hydrocortisone may be expected to be effective in each of the following
conditions EXCEPT

A. allergic urticaria
B. temporomandibular arthritis
C. ulcerative oral lesions
D. penicillin anaphylaxis
E. acute adrenocortical insufficiency
21. All of the following symptoms are associated with neostigmine poisoning
EXCEPT
A. diarrhea
B. salivation
C. convulsions
D. bronchiolar constriction
E. skeletal muscle paralysis
22. Heparin has all of the following effects EXCEPT
A. activation of plasma antithrombin
B. blocking of thromboplastin generation
C. neutralization of tissue thromboplastin
D. reduction of circulating fibrinogen
E. clearing of alimentary hyperlipemia
23. Xerostomia is a common side effect of all of the following EXCEPT
A. clonidine
B. reserpine
C. imipramine
D. scopolamine
E. promethazine
24. Which of the following diuretics is appropriate for a patient who is
suffering from congestive heart failure and tends to show digitalis toxicity?
A. Furosemide
B. Triamterene
C. Ethacrynic acid
D. Trichlormethiazide
E. Hydrochlorothiazide
25. Which of the following antibiotics is LEAST likely to cause superinfection?
A. Gentamicin
B. Tetracycline
C. Penicillin G
D. Streptomycin
E. Chloramphenicol

1.A
2.A
3.B
4.C
5.B

11.A
12.D
13.C
14.B
15.E

6.B

16.B

KEY (CORRECT ANSWERS)

7.C
8.A
9.E
10.E
21. C
22. D
23. B
24. B
25. C

17.E
18.E
19.A
20.D

PHARMACOLOGY
DIRECTIONS: Each question or incomplete statement is followed by several suggested
answers or completions. Select the one that BEST answers the question or completes
the statement. PRINT THE LETTER OF THE CORRECT ANSWER IN THE SPACE AT THE
RIGHT.
1 Which of the following drugs has its primary use in the treatment of the
manic phase of depressive psychosis?
A. Lithium
B. Reserpine
C. Imipramine
D. Amphetamine
E. Chlorpromazine
2 Which of the following penicillins has the BEST gramnegative spectrum?
A. Nafcillin
B. Ampicillin
C. Methicillin
D. Penicillin V
E. Phenethicillin
3. Which of the following is an endogenous catecholamine?
A. Dopamine
B. Ephedrine
C. Amphetamine
D. Isoproterenol
E. Phenylephrine
4 Which of the following plasma proteins has the GREATEST ability to bind
drugs?
A. Albumin
B. Fibrinogen
C. Hemoglobin
D. Gamma globulin
E. Beta lipoprotein
5 Which of the following is TRUE regarding the oral hypoglycemic agent
tolbutamide?
A. It blocks the destruction of insulin.
B. It depresses the beta cells of the pancreas.
C. Its use is primarily in maturity-onset diabetics.
D. It decreases activity of insulin antagonists.
E. It increases use of glucose by activating hexokinase.
6. Which of the following antibiotics may be cross-allergenic with penicillin?

A. Neomycin
B. Cephalexin
C. Clindamycin
D. Erythromycin
E. All of the above
7 Which of the following drug classes has definitely been established to be
teratogenic in humans?
A. Salicylates
B. Barbiturates
C. Antianginals
D. Anticancer drugs
E. Local anesthetics
8. Which of the following drugs is useful in treating the dermatologic
manifestations of an allergic response?
A. Diazepam
B. Atropine
C. Chlorpheniramine
D. Hexylresorcinol
E. Phenoxybenzamine
9. In treating xerostomia, which of the following might be prescribed?
A. Atropine
B. Ephedrine
C. Neostigmine
D. Scopolamine
E. Mecamylamine
10. Which of the following is most likely to be the MAJOR mechanism of
action of the tricyclic antidepressants?
A. Enhanced release of acetylcholine
B. Inhibition of neuronal synthesis of norepinephrine
C. Potentiation of serotonin synthesis in nervous tissue
D. Stimulation of re-uptake of norepinephrine from the synaptic cleft
E. Blockade of the re-uptake of amine neurotransmitters released into the synaptic cleft
11. Within which of the following drug groups is found a good correlation
between chemical structure and biologic activity?
A. Diuretics
B. Cathartics
C. Local anesthetics
D. General anesthetics
E. Non-barbiturate sedatives
12. In a young adult patient, deep sedation resulting from ingestion of 2 mg.
of diazepam characterizes a(n)
drug response.
A. reflex
B. allergic
C. iatrogenic
D. supersensitive
E. hypertensive
13. Which of the following compounds is a ganglionic blocking agent?
A. Curarine
B. Edrophonium
C. Mecamylamine

D. Succinylcholine
E. Gallamine triethiodide
14. Which of the following is a nonsteroidal anti-inflammatory agent with a
tendency to produce bone marrow depression?
A. Aspirin
B. Ibuprofen
C. Colchicine
D. Acetaminophen
E. Phenylbutazone

15. Which of the following factors is common to ALL forms of drug


dependence and abuse?
A. Tolerance
B. Tachyphylaxis
C. Physical dependence
D. Psychological dependence
E. None of the above
16. Which of the following drugs is thought to REDUCE arterial blood pressure
by activating alpha receptors in the vasomotor center of the medulla?
A. Prazosin
B. Clonidine
C. Propranolol
D. Guanethidine
E. Chlorothiazide
17. Selective B1 adrenergic agonists will produce
A. glycogenolysis
B. increased cardiac output
C. decreased diastolic pressure
D. decreased peripheral resistance
E. relaxation of bronchial smooth muscle
18. Which of the following ions augments the inotropic effect of digitalis?
A. Sodium
B. Lithium
C. Calcium
D. Chloride
E. Magnesium
19. Which of the following drugs is MOST effective as an antidepressant?
A. Diazepam
B. Reserpine
C. Amitriptyline
D. Chlorpromazine
E. Hydroxyzine
20. Which of the following drugs is MOST useful in treating or preventing
angina pectoris?
A. Digitalis
B. Quinidine
C. Propranolol
D. Procainamide
E. Pentobarbital

21. Aplastic anemia is a serious toxic effect that occurs particularly after a
course of treatment with
A. penicillin
B. lincomycin
C. tetracycline
D. streptomycin
E. chloramphenicol
22. Which of the following drugs are congeners possessing many similar
properties?
A. Aspirin and phenacetin
B. Phenylbutazone and ibuprofen
C. Phenacetin and acetaminophen
D. Propoxyphene and ethoheptazine
E. Phenytoin and indomethacin
23. Local anesthetics interfere with the transport of which of the following
ions during drug-receptor interaction?
A. Sodium
B. Calcium
C. Chloride
D. Potassium
E. Magnesium
24. Which of the following drugs competitively blocks the action of
norepinephrine at a-adrenergic receptors?
A. Atropine
B. Naloxone
C. Propranolol
D. Phentolamine
E. Hexamethonium
25. Which of the following is NOT true regarding combined oral
contraceptives? They
A. inhibit ovulation
B. contain an estrogen and a progestin
C. stimulate pituitary secretion of FSH and LH
D. produce endometrial changes resembling pregnancy
E. are contraindicated in the presence of estrogendependent cancers

KEY (CORRECT ANSWERS)


1.
2.
3.
4.
5.

A
B
A
A
C

6. B
7. D
8. C
9. C
10. E

11.C
12.D
13.C
14.E
15.D
16.B
17.B
18.C
19.C
20.C

21.
22.
23.
24.
25.

E
C
A
D
C

DIRECTIONS: Each question or incomplete statement is followed by several suggested


answers or completions. Select the one that BEST answers the question or completes
the statement. PRINT THE LETTER OF THE CORRECT ANSWER IN THE SPACE AT THE
RIGHT.
1. Which of the following drugs does NOT have a tendency to produce blood
dyscrasias?
A. Codeine
B. Phenytoin
C. Indomethacin
D. Phenylbutazone
E. Chloramphenicol
2. Which of the following is NOT a factor in determining the blood level of
alcohol?
A. Body weight
B. Rate of ingestion
C. State of mind and environment
D. Presence of food in the stomach
E. Alcoholic concentration of the beverage ingested
3. Which of the following compounds does NOT relieve pain?
A. Codeine
B. Methadone
C. Meperidine
D. Hydromorphone
E. Chloral hydrate
4. Which of the following statements is NOT true regarding the adrenal
corticosteroids? They
A. cause retention of sodium and fluid
B. decrease activity in lymphoid tissue
C. heighten the immune response to antigens
D. can produce a diabetes-like syndrome with high blood levels
E. are therapeutically beneficial when administered orally, parenterally or topically
5. The presence of which of the following conditions is NOT a
CONTRAINDICATION to the use of corticosteroids?
A. Peptic ulcers

B. Herpes simplex
C. Aphthous ulcers
D. Latent tuberculosis
E. AIDS
6. Which of the following regarding propoxyphene is NOT true?
A. It is a Schedule IV drug.
B. It is often formulated with aspirin or APC.
C. It is prepared as hydrochloride and napsylate salts.
D. It has an analgesic efficacy comparable to morphine.
E. It may produce side effects of drowsiness and disorientation.

7. For which of the following drugs is the duration of action NOT markedly
influenced by the activity of enzyme systems responsible for its
biotransformation?
A. Lidocaine
B. Diazepam
C. Morphine
D. Penicillin
E. Acetylcholine
8. Which of the following is NOT an action of a drug that inhibits
prostaglandin synthesis?
A. Antipyretic action
B. Relief of inflammation
C. Relief of sharp, piercing pain
D. Exacerbation of an existing gastric ulcer
E. Increased production of gastric hydrochloric acid
9. Which of the following is NOT important in the rate of absorption of aspirin
after oral administration?
A. pH of the stomach
B. Emptying time of the stomach
C. Solubility of the aspirin in the vehicle
D. Form of the crystals of aspirin
E. Level of prostaglandins in the gastric mucosa
10. Each of the following is a side effect of prolonged tetracycline
hydrochloride therapy EXCEPT
A. suprainfection
B. photosensitivity
C. vestibular disturbances
D. discoloration of newly forming teeth
E. gastrointestinal symptoms (when administered orally)
11. All of the following are possible effects of aspirin EXCEPT
A. reduction of fever
B. shortening of bleeding time
C. suppression of inflammatory response
D. bleeding from the gastrointestinal tract
E. increase in the renal excretion of uric acid at high doses
12. All of the following are TRUE about prescribing medication for elderly
patients EXCEPT that
A. lower doses may be in order

B. absorption of drugs may be decreased


C. efficiency of drug elimination is likely to be increased in old age
D. elderly patients are particularly susceptible to adverse drug reactions
E. none of the above
13. General anesthetics can do all of the following EXCEPT
A. produce delirium
B. stimulate medullary centers
C. produce a state of unconsciousness
D. reduce perception of painful stimuli
E. decrease excitability of the motor cortex
14. When given to treat a TRUE anaphylactic reaction, epinephrine has all of
the following desirable effects
EXCEPT
A. rapid onset of action
B. vasopressor activity
C. bronchodilator properties
D. ability to antagonize the effects of histamine
E. interference with the antigen-antibody reaction
15. All of the following statements regarding barbiturates are TRUE EXCEPT
that
A. they suppress REM sleep
B. their actions are potentiated by the monoamine oxidase inhibitors
C. various preparations differ primarily in duration of action
D. they are primarily detoxified by hepatic action
E. the greater the lipid solubility, the lesser the hypnotic potency and efficacy
16. All of the following are TRUE regarding the anticancer drugs EXCEPT that
they
A. have low therapeutic indices
B. tend to suppress bone marrow function
C. are more effective in treating leukemias than solid tumors
D. are generally curative if given in proper dosage
E. act on cellular processes common to both normal and cancerous cells
17. In the treatment of congestive heart failure, digitalis glycosides generally
DECREASE all of the following EXCEPT
A. edema
B. urine flow
C. heart size
D. heart rate
E. residual diastolic volume
18. All of the following influence the rate of induction during anesthesia
EXCEPT
A. pulmonary ventilation
B. blood supply to the lungs
C. hemoglobin content of the blood
D. concentration of the anesthetic in the inspired mixture
E. solubility of the anesthetic in blood (blood-gas partition coefficient, Ostwald
coefficient)
19. Each of the following is a symptom of cholinergic crisis EXCEPT
A. bradycardia
B. lacrimation
C. vasoconstriction
D. extreme salivation

E. weakness of voluntary muscles


20. All of the following statements are TRUE regarding the occupation theory
of drug-receptor interaction EXCEPT that
A. the affinity of a drug is dependent on its intrinsic activity
B. the maximum effect of a drug occurs when all receptors are occupied
C. an antagonist has affinity for the receptor but no intrinsic activity
D. the magnitude of the effect of a drug is proportional to the number of receptors
occupied
E. it follows the law of mass action
21. All of the following drugs are useful in the treatment of cardiac
arrhythmias EXCEPT
A. digitalis
B. lidocaine
C. phenytoin
D. procainamide
E. aminophylline
22. All of the following are classified as Schedule II drugs under the
Controlled Substances Act EXCEPT
A. codeine
B. oxycodone
C. meperidine
D. pentobarbital
E. acetaminophen with codeine
23. All of the following are pharmacologic or toxicologic properties of
acetylsalicylic acid EXCEPT
A. tinnitus
B. analgesia
C. antipyresis
D. methemoglobinemia
E. inhibition of prostaglandin synthesis
24. APC is composed of all of the following EXCEPT
A. aspirin
B. codeine
C. caffeine
D. phenacetin
E. none of the above
25. All of the following antibiotics are considered to have a high rate of
allergenicity EXCEPT
A. penicillins
B. erythromycin
C. sulfonamides
D. amphotericin B
E. none of the above
KEY (CORRECT ANSWERS)
1. A
2. C
3. E

6. D
7. D
8. C

11. B
12. C
13. B

16. D
17. B
18. C

21. E
22. E
23. D

4. C
5. C

9. E
10. C

14. E
15. E

19. C
20. A

PHARMACOLOGY

24. B
25. B

DIRECTIONS: Each question or incomplete statement is followed by several suggested


answers or completions. Select the one that BEST answers the queston or completes the
statement. PRINT THE LETTER OF THE CORRECT ANSWER IN THE SPACE AT THE RIGHT.
1. A prescription order for 2-PAM indicates an attempt to
A. boric acid ingestion
B. insulin overdose
C. petroleum distillate
D. digitalis toxicity
E. cholinesterase inhibitors

antagonize

2. The ingestion of 1 milligram per day of folic acid is known to mask the
clinical symptoms of which of the
following?
I.
Iron-deficiency anemia
II.
II. Pernicious anemia
III.
Anoxic anemia
The CORRECT answer is:
A. I only
B. II only
C. I and III
D. II and III
E. I, II, and III
3. Vomiting should NEVER be induced after the ingestion of which of the
following toxic substances?
I. Strychnine
II. Sulfuric acid
III. Aspirin
The CORRECT answer is:
A. I only
B. III only
C. I and II D. II and III
E. I, II, and III
4. Symptoms that suggest digitalis toxicity include which of the following?
I. Hyperkalemia
II. Yellow-greenish vision
III. Nausea and vomiting
The CORRECT answer is:
A. I only
B. III only
C. I and II D. II and III
E. I, II, and III
5. The toxic manifestation of drugs that are used in cancer chemotherapy are
MOST frequently seen in
A. old cells since the blood supply is reduced
B. liver cells because of their high metabolic activity
C. cells of the G.I. tract because they are rapidly dividing
D. cells on the surface of organs since they are more exposed
E. kidney cells because they receive a high percentage of the total blood flow
6. Spironolactone (Aldactone) is frequently prescribed concurrently with
thiazide diuretics MAINLY because spironolactone
A. enhances potassium excretion
B. promotes potassium retention
C. enhances uric acid excretion
D. promotes uric acid retention
E. reduces the hypotensive activity of the thiazide

7. A patient who is experiencing ventricular extrasystoles following a


myocardial infarction MOST likely would be given
A. digitoxin
B. quinidine
C. lidocaine
D. propranolol
E. spironolactone
8. Subsequent to surgery, a patient develops thrombophlebitis. To prevent
further formation of thrombi,
the recommended orally administered drug is
A. protamine sulfate
B. vitamin K1
C. menadione
D. heparin
E. warfarin
9. The oral antimicrobial agent of choice for the treatment of urinary tract
infections caused by pseudomonas organisms is
A. carbenicillin
B. polymyxin B
C. gentamicin
D. ampicillin
E. sulfisoxazole

10. Of the following, which is the drug of choice for the initial treatment of a
patient with mild essential
hypertension (e.g., diastolic pressure approximately 100 millimeters of
mercury)?
A. Guanethidine (Ismelin)
B. Pentolinium (Ansolysen)
C. Hydrochlorothiazide (HydroDIURIL)
D. Intravenous reserpine
E. Diazoxide (Hyperstat I.V.)
11. A teenage female patient has mild cyclic fluid retention. The MOST
appropriate diuretic dose of HydroDIURIL for her to take every morning is
A. 2.5 mg.
B. 25 mg.
C. 100 mg. D. 250 mg. E. 500 mg.
12. Which of the following is the MOST appropriate daily hypotensive dose of
reserpine?
A. 0.25 mg. B. 1 mg.
C. 5 mg.
D. 25 mg.
E. 50 mg.
13. The commonly accepted reason for tetracycline's beneficial action in the
treatment of acne is based upon the drug's ability to
A. increase hepatic metabolism of circulating fats
B. increase excretion of circulating fats
C. decrease intestinal absorption of dietary fats
D. decrease free fatty acid production within the follicle
E. dilate follicle pores to permit better drainage
14. In prescribing "alternate day therapy" for a patient who is taking
corticosteroids, a physician's rationale is MOST likely which of the following?
A. The patient's cost will be reduced
B. There will be fewer gastrointestinal effects
C. Inhibition of ACTH release will be minimized

D. It is more convenient for the patient


E. There are fewer chances for drug interactions to occur
15. A report in the literature indicates that propranolol depresses renin
release from the juxtaglomerular apparatus. This action, if clinically
significant, is MOST closely associated with the use of propranolol in treating
A. migraine
B. asthma
C. tachycardia
D. hypertension
E. angina
Question 16.
DIRECTIONS: Read the following paragraph and then answer the question that follows.
Caffeine stimulates gastric acid secretion and reduces the competence of the lower
esophageal sphincter in man. These effects of caffeine have been used as evidence that
regular coffee should not be used by patients with peptic ulcer disease or
gastrointestinal reflux. We compared the dose-response relations of caffeine, regular
coffee, and decaffeinated coffee for gastric acid secretion and sphincter pressure in
normal subjects. Decaffeinated coffee gave a maximal acid response of 16.5 2.6
milliequivalents per hour (mean
- S.E.), which was similar to that of regular coffee, 20.9 - 3.6 milli equivalents per hour,
both values being higher than that of caffeine, 8.4 1.3, on a cup-equivalent basis.
Sphincter pressure showed
minimal changes in response to caffeine, but was significantly increased by both regular
and decaffeinated coffee.
16. Which of the following is the MOST appropriate interpretation of the data
given?
A. Caffeine is safe to use in ulcer patients
B. Clinical recommendations based upon the known G.I. effects of caffeine may bear
little relation to the
actual observed action of coffee or decaffeinated coffee
C. Caffeine alone should not be compared with regular coffee or decaffeinated coffee in
clinical investiga tions
D. The results of this study suggest that both regular and decaffeinated coffee are less
potent stimulants
of gastric acid secretion than caffeine alone
E. The experimental design was not rigorous enough to draw any conclusions
17. A PRIMARY symptom of an incontinent patient is
A. intermittent claudication
B. an impacted bowel
C. frequent urination
D. partial impotency
E. severe mental depression
18. The clinical significance of achlorhydria is that it usually suggests that
the patient also has
A. aplastic anemia
B. pernicious anemia

C. Cooley's anemia
D. duodenal ulcers
E. gastric ulcers
19. The efficacious route of administration for procaine penicillin G is
A. topical
B. oral
C. subcutaneous
D. intravenous
E. intramuscular
20. Which of the following drugs must be given with extreme caution to
patients who have a known orythrocytic glucose-6-phosphate dehydrogenase
deficiency?
A. Indomethacin
B. Nitrofurantoin
C. Penicillin
D. Warfarin
E. Isoniazid
21. Following oral dosing of griseofulvin, enhanced absorption of the antiinfective agent can occur if the patient
A. takes antacids containing calcium
B. takes B-complex vitamins
C. drinks large amounts of water
D. takes the drug within 2 hours of a meal
E. eats foods rich in fat
22. Succinylcholine, a neuromuscular blocking agent, is used as a
preanesthetic medication because its duration of action is
A. short, due to rapid oxidation in the liver
B. short, due to a high affinity for plasma albumin binding sites
C. short, due to rapid hydrolysis by plasma esterases
D. long, due to relatively slow hydrolysis by plasma esterases
E. long, due to the body's low concentration of hepatic metabolizing enzymes
23. Among pediatric patients between 1 and 3 years of age, the MOST
prominent factor concerned with drug toxicity is
A. improperly developed absorption surfaces
B. inadequate circulatory systems
C. incompletely developed microsomal enzyme systems
D. incompletely developed plasma albumin
E. incompletely developed excretory mechanisms
24. In the event of carbon monoxide poisoning, a rational first-aid measure is
to
A. administer alcohol
B. administer amyl nitrite by inhalation
C. administer dimercaprol (BAL)
D. have the victim exercise to avoid losing consciousness
E. allow the victim to breathe fresh air
25. The MOST important antidotal measure in treating barbiturate toxicity is
to
A. keep the patient standing upright
B. support respiration
C. apply ice packs to the patient's forehead
D. give an oil-retention enema at once

E. inject morphine p.r.n. pain

KEY (CORRECT ANSWERS)


1.
2.
3.
4.
5.

E
B
C
D
C

6. B
7. C
8. E
9. A
10. C
21.
22.
23.
24.
25.

11. B
12. A
13. D
14. C
15. D
16. B
17. C
18. B
19. E
20. B

E
C
C
E
B

DIRECTIONS:
Each question or incomplete statement is followed by several suggested answers or
completions. Select the one that BEST answers the question or completes the
statement. PRINT THE LETTER OF THE CORRECT ANSWER IN THE SPACE AT THE RIGHT.
1. When deferoxamine is used in the treatment of iron toxicity, the
antagonism represents
A. chelation with iron to promote its excretion by the kidney
B. oxidation of iron to form an insoluble complex
C. displacement of iron from plasma protein binding sites
D. reaction with bone matrix to induce deposition of iron in bone
E. stimulation of microsomal enzymes to reduce iron levels
2. Which of the following oral hypoglycemic agents does NOT act primarily by
stimulating endogenous insulin release?
A. Acetohexamide
B. Phenformin
C. Chlorpropamide
D. Tolazamide
E. Tolbutamide

3. Which of the following chemotherapeutic agents is a folic acid antagonist?


A. Tetracycline
B. Methotrexate
C. Chloramphenicol
D. Isoniazid
E. Quinine
4. Which of the following agents is used to diagnose pheochromocytom a?
A. Phentolamine (Regitine)
B. Dehydrocholate (Decholin Sodium)
C. Tolbutamide (Orinase) D. Sodium aminohippurate
E. Metyrapone (Metopirone)
5. A patient complaining of gastric upset is admitted to the hospital to
undergo tests for achlorhydria. Of the following, the diagnostic agent of
choice is
A. diphenhydramine hydrochloride (Benadryl)
B. dehydrocholate (Decholin Sodium)
C. phentolamine (Regitine) D. bromsulphalein (BSP)
E. betazole hydrochloride (Histalog)
6. Forms of insulin that can be administered by intravenous infusion include
which of the following?
I. Isophane (NPH)
II. Regular (Iletin)
III. Insulin zinc (lente)
The CORRECT answer is:
A. I only
B. II only
C. I and III D. II and III
E. I, II, and III

7. Agents that would be suitable for a patient who has a known allergy to
aspirin include which of the following?
I. Acetaminophen
II. Propoxyphene
III. Phenacetin
The CORRECT answer is:
A. II only
B. III only
C. I and II D. I and III
E. I, II, and III
8. Of the following, which sulfonamide is used as a preoperative medication to
sterilize the bowel prior to
abdominal surgery?
A. Sulfisoxazole
B. Sulfamethoxazole
C. Phthalylsulfathiazole
D. Sulfadiazine
E. Triple sulfas
9. Which of the following oral hypoglycemic agents has the LONGEST
biological half-life?
A. Acetohexamide
B. Phenformin

C. Chlorpropamide
D. Tolazamide
E. Tolbutamide
10. Patients receiving metronidazole (Flagyl) for trichomonal infections
should be cautioned to avoid the ingestion of
A. certain cheeses
B. alcohol
C. milk
D. penicillin
E. anticholinergics
11. The antibacterial action of methenamine mandelate (Mandelamine) is
increased by the concurrent administration of
A. physiological saline
B. sodium benzoate
C. ammonium chloride
D. sodium bicarbonate
E. acetylsalicylic acid
12. A patron taking levodopa for parkinsonism should be advised against
concurrent administration of vitamin preparations that contain
A. folic acid
B. vitamin B2
C. vitamin B1
D. vitamin B12
E. vitamin B6
13. Ibuprofen is BEST classified as a(n)
A. anti-inflammatory agent
B. antiarrhythmic agent
C. hypotensive agent
D. anticholinergic
E. diuretic

14. Methapyrilene, a common component of OTC sleep preparations, can be


classified pharmacologically as a(n)
A. hypnotic
B. sympathomimetric
C. parasympathomimetic
D. analgesic
E. antihistaminic
15. Which of the following agents is considered the prototype of predominant
beta-adrenergic antivation?
A. Phenylephrine
B. Propranolol
C. Epinephrine
D. Isoproterenol
E. Norepinephrine
16. In adults, the MOST common side-effect associated with Benadryl is
A. gastrointestinal distress
B. sedation
C. xerostomia
D. blurred vision

E, excitation
17. Which of the following conditions CONTRAINDICATES the local instillation
of corticosteroids, such as Decadron, into the eye?
A. Glaucoma
B. Night blindness
C. Allergies to pollen
D. Astigmatism
E. Conjunctivitis
18. Diuril must be used with caution in persons with
A. hyperkalemia
B. hypernatremia
C. hypertension
D. hyperkinesis
E. hyperuricemia
19. Phenylephrine (Neo-Synephrine) acts as a nasal decongestant by
A. causing a local anesthetic action
B. blocking at the synaptic ganglia
C. constricting blood vessels by alpha-adrenergic stimulation
D. dilating blood vessels by beta-adrenergic stimulation
E. blocking nuscarinic receptors
20. Isoproterenol (Isuprel) stimulates the heart, in part, by
A. blocking the vagal reflex
B. stimulating the cardioaccelerator center in the medulla
C. blocking the Renshaw cells in the spinal cord
D. stimulating the beta-adrenergic receptors in the heart
E. inhibiting the effect of epinephrine on alpha- adrenergic receptors

21. The agent of choice for treating acute attacks of gout is


A. allopurinol
B. probenecid
C. phenylbutazone
D. sulfinpyrazone
E. colchicine
22. A prescription for 3.5 grams of ampicillin and 1 gram of probenecid to be
taken as a single oral dose is MOST likely for the treatment of
A. cystitis
B. rheumatic fever
C. gonorrhea
D. impetigo
E. ulcerative colitis
23. Drugs that are appropriate for a person who requests medication for
motion sickness include which of the following?
I. Dimenhydrinate
II. Meclizine
III. Dexamethasone
The CORRECT answer is:
A. I only
B. II only
C. I and II D. II and III
E. I, II, and III

24. A daily dose of 160 milligrams of propranolol (Inderal) is likely to be used


for the treatment of
A. angina
B. atherosclerosis
C. asthma
D. myalgia
E. gout
25. Benztropine mesylate (Cogentin) is indicated for treatment of
A. gouty arthritis
B. petit mal epilepsy
C. wide-angle glaucoma
D. Parkinson's disease
E. myasthenia gravis
KEY (CORRECT ANSWERS)
1. A
2. B
3. B
4. A
5. E
6. B
7. E
8. C
9. C
10. B
21. E
22. C
23. C
24. A
25. D

11
12
13
14
15
16
17
18
19
20

C
E
A
E
D
B
A
E
C
D

BIOCHEMISTRY / PHYSIOLOGY
SECTION
TEST 1

EXAMINATION

DIRECTIONS:
Each question or incomplete statement is followed by several suggested answers or
completions. Select the one that BEST answers the question or completes the
statement. PRINT THE LETTER OF THE CORRECT
ANSWER IN THE SPACE AT THE RIGHT.
1. Human bile salts are formed in the liver and their presence in the small
intestine aids lipid digestion.
Their composition includes
A. cholesterol
B. biliverdin and bilirubin
C. cholesterol esterified to palmitic acid
D. cholic acid conjugated with alanine or serine
E. cholic acid conjugated with glycine or taurine
2. Recruitment with respect to motor neurons refers to
A. an increase in the action potential of each fiber
B. an increase in the strength of discharge of each fiber

C. progressive increase in the number of motor units involved


D. continuing activity of neurons after stimulation has ceased
E. spread of reflex activity to different areas of the central nervous system
3. The absorption of calcium from the small intestine is increased by
A. the ingestion of fat
B. a high phosphate intake
C. the administration of vitamin A
D. lowering the pH in the intestine
E. the administration of parathyroid hormone
4. Standard limb III of the ECG represents cardiac potential differences as
they occur between
A. right arm and right leg
B. right arm and left leg
C. right arm and left arm
D. left arm and left leg
E. left arm and right leg
5. The extracellular polysaccharides synthesized by cariogenic streptococci in
the presence of excess sucrose are
BEST described as
A. amylose
B. amylopectin
C. mucopolysaccharide
D. dextran-like glucan
E. glycogen-like glucan
6. The resting potential of a nerve membrane is maintained by
A. osmoreceptors
B. Gibbs-Donn an equilibrium
D. electrical and chemical gradients
D. active transport of Na and K ions
E. membrane permeability to potassium ion
7. The MOST important function of hydrochloric acid in the stomach is
A. hydrolysis of protein
B. neutralization of chyme
C. activation of pepsinogen
D. destruction of bacteria
E. stimulation of pancreatic secretion
8. Beta oxidation of a mole of an 18 carbon fatty acid under physiologic
conditions produces
A. one mole of acetic acid
B. nine moles of acetyl CoA C, nine moles of Rooti
D. one mole of acetoacetic acid
E. one mole of hydroxybutyric acid
9. The carotid body receptors are stimulated MOST effectively by
A. elevated pH
B. low oxygen tension
C. high oxygen tension
D. low carbon dioxide tension
E. increased hydrogen ion concentration
10. Breathing ceases upon destruction of the
A. cerebrum
B. cerebellum
C. medulla oblongata

D. thalamus
E. hypothalamus
11. The PRIMARY effect of substituting a rigid arterial system for a compliant
arterial system would be that
A. continuous flow in the capillaries would change into pulsatile flow
B. arterial pulse pressure would diminish
C. diastolic pressure would not be affected
D. systolic pressure would not be affected
E. arterial pulse pressure would increase
12. Biotin is required in non-mitochondrial fatty acid biosynthesis to produce
which of the following intermediates?
A. Acetyl CoA
B. Malonyl CoA
C. Methylmalonyl CoA
D. Dimethylcrotonyl CoA
E. None of the above
13. The electrolytes in tissue fluid help to retain water. Which of the following
is of GREATEST influence in this phenomenon?

A. K+
Ca2+

B. Na+

C. Cl-

D. HPO42-

E.

14. Carbon dioxide is transported in four different chemical forms. Which of


the following is the PRINCIPAL mode of transport?
A. Dissolved carbon dioxide
B. Carbamino compounds
C. Bicarbonate ions
D. Carbonic acid
E. None of the above
15 Enzymes for electron transport reactions are MOST active and
concentrated in which of the following structures of the animal cell?
A. Nuclei
B. Lysosomes
C. Microsomes
D. Mitochondria
E. All of the above
16 In a nerve fiber, conduction velocity depends on the
A. diameter of the fiber
B. amplitude of the action
potential
C. intensity of the stimulus
D. presence of myelin sheath
E. both A and C above
F. both A and D above
G. both B and D above
17. Complete destructon of the anterior pituitary would result in Addison's
disease, myxedema, or
A. hypogonadism
B .diabetes mellitus
C. diabetes insipidus
D. Cushing's syndrome
E. Goldring's cushion
18. Thyroid hormones, glucocorticoids, an d gonadal steroids are similar in
that each
A. is derived from cholesterol
B. is derived from amino acids
C. acts on only one target organ or tissue

D. is released in response to signals from the hypo- thalamic-anterior pituitary complex


E. is released in response to signals from the hypo- thalamic-posterior pituitary complex
19. Local chemical anesthetics block nerve conduction by
A. depolarizing the nerve membrane to neutrality
B. increasing the membrane permeability to K+
C. increasing the membrane permeability to Na+
D. preventing the increase in membrane permeability to K+
E. preventing the increase in membrane permeability to Na+
20. One of the most prevalent consequences associated with ageing is that
A. collagen is not synthesized
B. there is more cross-linking in collagen
C. there is less cross-linking in collagen
D. turnover of connective tissue is more rapid
E. None of the above
21. A man working under conditions of high temperature should increase his
intake of
A. NaCl
B. iron
C. iodides
D. vitamin D
E. phosphates
22. In the completely isolated, blood-perfused mammalian heart, adjustments
to increased workload appear to be mediated PRIMARILY by mechanisms
associated with
A. increased heart rate
B. increased systolic volume
C. decreased systolic volume
D. decreased diastolic volume
E. increased diastolic volume
23. Which of the following hormones is NOT synthesized from cholesterol?
A. Progesterone
B. Testosterone
C. Estradiol
D. Cortisol
E. Relaxin

24. Which of the following fatty acids are ESSENTIAL to the human diet
because human tissue cannot synthesize sufficient quantities?
I.
Arachidonic
II.
II. Linoleic
III.
Linolenic
IV.
IV. Oleic
V.
Palmitic
VI.
VI. Palmitoleic
VII.
Stearic
The CORRECT answer is:
A. I, II, III
B. I, II, IV
C. I, II, VI
D. II, III, IV
E. III, IV, V

F. V, VI, VII
25. Which of the following dietary components are SPECIFICAL LY involved in
the synthesis of hemoglobin?
I. Sodium
II. Potassium
III. Iron
IV. Copper
V. Phosphate
VI. Selenium
The CORRECT answer is:
A. I, II
B. II, IV
C. III, IV
D. III, VI
E. IV, V

KEY (CORRECT ANSWERS)


1.E
11.A
2.C
12.B
3.D
13.B
4.D
14.C
5.D
15.D
6.D
7.C
8.B
9.B
10.C

16.F
17.A
18.D
19.E
20.B
21. A
22. E
23. E
24. A
25. C

DIRECTIONS:
Each question or incomplete statement is followed by several suggested answers or
completions. Select the one that BEST answers the question or completes the
statement. PRINT THE LETTER OF THE CORRECT
ANSWER IN THE SPACE AT THE RIGHT.
1. The termination of synthesis of a polypeptide is believed to involve
A. nonsense codons
B. anticodon-codon interaction
C. tRNA which cannot bind amino acids
D. hydrolysis of messenger RNA

E. none of the above


2. Progestins are being used as oral contraceptive substances due to their
ability to
A. suppress ovulation
B. combine with insulin
C. inhibit the production of FSH
D. inhibit the proliferation of the endometrium
E. increase ovulation
3. The amount of red blood cell destruction per day is BEST measured by the
A. hematocrit
B. amount of iron excreted
C. change in the red blood cell count
D. amount of hemoglobin/100 ml. blood
E. amount of bile pigments excreted by the liver
4. A substance is considered to undergo net secretion by the kidney tubules if
its
A. concentration in urine is greater than in blood
B. concentration in urine is greater than in plasma
C. extraction ratio is close to zero
D. clearance is greater than that of inulin
E. clearance is greater than unity
5. The physiologic importance of hemoglobin lies in its ability to combine
A. irreversibly with oxygen and CO2
B. reversibly with oxygen at the ferric heme prosthetic group
C. irreversibly with oxygen at the ferrous heme prosthetic group
D. reversibly with oxygen at the ferrous heme prosthetic group
E None of the above
6. The major component of the nonprotein nitrogen fraction of blood is
A. urea
B. uric acid
C. urobilinogen
D. hippuric acid
E. free amino acids
7. Proteins are able to buffer physiologic solutions over a wide range of pH
because they
A. are macromolecules of high molecular weight
B. contain many functional groups with differing pK's
C. have considerable secondary and tertiary structure which causes hydrogen ions to be
sequestered
D. have many peptide bonds which are very resistant to hydrolysis by hydrogen or
hydroxyl ions
E. are micromolecules of low molecular weight
8. Certain viruses have been isolated in crystalline form and have been found
to be
A. nucleotides
B. polypeptides
C. phospholipids
D. scleroproteins
E. nucleoproteins
9. Systemic and pulmonary circulations are alike in that they have the same

A. total capacity
B. pulse pressure
C. diastolic pressure
D. resistance to flow
E. volume flow of blood per minute
10. A sudden increased inflow into the right atrium will cause, in two or three
heart beats, a(n)
A. fall in capillary pressure
B. slight fall in blood pressure
C. increased blood flow to the lungs
D. fall in pressure in the large veins
E. rise in capillary pressure
11. Triglyceride absorbed into the lymphatic system is transported to the liver
as
A. chylomicrons
B. very low density lipoprotein
C. low density lipoprotein
D. high density lipoprotein
E. very high density lipoprotein
12. The exchange of gases between plasma and tissue fluid is a function of
A. partial pressures
B. hydrostatic pressures
C. osmotic pressure differentials
D. differences in volumes per cent of the gases
E. complete pressures
13. Sickle-cell anemia is caused by the presence of a valine substitution for
glutamate in the sixth amino acid from the N-terminal end of the hemoglobin
B-chain. This amino acid substitution is the result of
A. insufficient glutamyl-tRNA
B. a lack of glutamyl-tRNA synthetase
C. a base change in the DNA of the B-chain gene
D. a change in the anticodon loop of glutamyl-tRNA
E. superflous glutamyl-tRNA
14. The naturally occurring fruit sugar D-fructose has a specific rotation of
-88.5. It is a D sugar in spite
of the negative rotation because of its
A. similarity to D-glucose
B. occurrence in natural products
C. presence in sucrose, a dextro sugar
D. structural relationship to D-glyceraldehyde
E. utilization by organisms which require sugars with a D configuration

15. In the human female, ovulation is believed to be caused by


A. a shift in anterior pituitary gonadotropin secretion with LH predominating in the
mixture
B. a sudden diminution of anterior pituitary gonadotropin secretion
C. a sudden increase in FSH secretion by the anterior pituitary

D. all of the above


E. none of the above
16. Miniature end-plate potentials (MEPP) are due to
A. spontaneous pacemaker activity of the end-plate
B. release of subthreshold amounts of acetylcholine
C. artifacts of old recording systems, eliminated by modern equipment
D. primary oscillations in Na+ and K+ conductance of the end-plate membrane
E. secondary oscillations in Na+ and K+ conductance of the end-plate membrane
17. The cell bodies of sympathetic preganglionic neurons are located in the
A. dorsal root ganglia
B. sympathetic chain ganglia
C. brain stem and spinal cord
D. thoracic and lumbar segments of the spinal cord
E. None of the above
18. In relation to its effect upon the movement of water between the vascular
and extravascular space, plasma colloid osmotic pressure acts in the same
direction as --------------the pressure .
A. tissue
B. venous
C. capillary
D. arteriolar
E. tissue colloid osmotic
19. Glutamate decarboxylase, an enzyme which catalyzes the formation of
gamma-amino butyric acid, is unique to
A. bone
B. skin
C. heart muscle
D. nervous tissue
E. connective tissue
20. The bicarbonate buffer system of the blood is very efficient because
A. bicarbonate is rapidly excreted by the kidneys
B. bicarbonate is able to be stored in the tissue
C. carbon dioxide is able to combine with hemoglobin
D. carbon dioxide forms a stable combination with base
E. carbon dioxide is rapidly eliminated through the lungs
21. The sensory receptors serving the stretch reflex are classified as
A. nociceptors
B. chemoreceptors
C. proprioceptors
D. exteroceptors
E. extraceptors
22. The lipotropic effect of lecithin upon fatty livers may also be produced by
A. choline
B. lipoic acid
C. ethanolamine
D. beta-hydroxybutyric acid
E. acetylcholine
23. A magenta tongue and lesions in the corners of the mouth suggest a
deficiency of
A. niacin
B. thiamine
C. folic acid

D. riboflavin
E. cyanocobalamin
24. Which of the following are considered "trace elements" in nutrition?
I. Calcium
II. Manganese
III. Chloride
IV. Phosphorus
V. Copper
The CORRECT answer is:
A. I, II
B. I, III, IV
C. I, IV
D. II, V
E. III, V
25. The human cells which do NOT contain an active mitochondrial electron
transport system are found in the
A. pancreas
B. kidney
C. liver
D. brain
E. blood

KEY (CORRECT ANSWERS)


1. A
2. A
3. E
4. D
5. D
6. A

11. A
12. A
13. C
14. D
15. A
16. B

7. B
8. E
9. E
10. C

17. D
18. A
19. D
20.

BIOCHEMISTRY / PHYSIOLOGY
EXAMINATION SECTION
DIRECTIONS:

21. C
22. A
23. D
24. D
25. E

Each question or incomplete statement is followed by several suggested answers or


completions. Select the one that BEST answers the question or completes the statement.
PRINT THE LETTER OF THE CORRECT
ANSWER IN THE SPACE AT THE RIGHT.
1. The countercurrent theory is used to explain the functioning of
A. the kidney
B. nerve fibers
C. muscle fibers
D. the cerebral cortex
E. the basilar membrane in the cochlea
2. Regeneration of severed axons does NOT take place in the central nervous
system because of the
A. absence of a neurilemma
B. presence of spinal fluid
C. absence of a myelin sheath
D. impediment of scar tissue to growth
E. none of the above
3. The continuous flow of blood in arteries during diastole is made possible by
A. the thoracic pump
B. relaxation of arterioles
C. the pumping action of skeletal muscle
D. energy stored in arteries during systole
E. none of the above
4. Intrapleural pressure during normal respiration is
A. subatmospheric
B. positive during expiration
C. positive during inspiration
D. zero at the end of expiration
E. zero at the end of inspiration
5. In relation to its effect upon movement of water between vascular and
extravascular space, plasma colloid osmotic pressure acts in the same
direction as ------------- pressure.
A. tissue
B. venous
C. capillary
D. arteriolar
E. tissue colloid osmotic
6. The amino acid that contributes to the tertiary structure of a protein by
causing a bend when it occurs in the primary sequence is
A. leucine
B. alanine
C. proline
D. tyrosine
E. aspartic acid

7. The plasma protein subfraction that contains antibodies found bathing


mucous surfaces
(mouth, bronchial passages, small intestine) is

A. IgA
B. IgD
C. IgE
D. IgG
E. IgM
8. Under normal conditions, the resting potential across a muscle cell
membrane is lower than the equilibrium potential for potassium because
A. sodium permeability is equal to potassium permeability
B. sodium permeability exceeds potassium permeability
C. the membrane has a low permeability to the sodium ion
D. the membrane has a low permeability to the chloride ion
E. none of the above
9. Two enzymes that have been postulated to play very important roles in
calcification are
A. enolase and phosphorylase
B. alkaline phosphatase and catalase
C. pyrophosphatase and carbonic anhydrase
D. pyrophosphatase and alkaline phosphatase
E. carbonic anhydrase and alkaline phosphatase
10. Free fatty acids are transported in blood plasma as
A. sodium salts
B. undissociated acids
C. a complex with albumin
D. a micelle complexed with bile salts
E. a major constituent of plasma lipoproteins
11. Removal of the anterior lobe of the pituitary gland results in
A. diabetes
B. loss of capacity to oxidize fat
C. loss of capacity to store sugar
D. increased sensitivity to insulin
E. none of the above
12. Most bound calcium is transported in blood as
A. calcium phosphate
B. a globulin complex
C. an albumin complex
D. an associate with hemoglobin
E. none of the above
13. The MAIN effect of aldosterone is to cause
A. increased secretion of antidiuretic hormone
B. inhibition of the micturition reflex
C. increased blood flow into the vasa recta
D. increased permeability of sodium in the proximal tubule and Henle's loop
E. increased permeability of sodium in the distal tubule and the collecting duct
14. In a state of water intoxication, the extra volume of water is found
primarily
A. in potential spaces
B. in the interstitial compartment
C. in the intracellular compartment
D. in the intravascular compartment
E. equally distributed among all compartments
15. The MAJOR end-product of carbohydrate ingestion is
A. xylose

B. glucose
C. mannose
D. fructose
E. galactose
16. The discharge of impulses in small motor (fusimotor) neurons innervating
muscle spindles serves to
A. inhibit extrafusal muscle contraction
B. sustain extrafusal muscle contraction
C. protect the muscle from excessive strain
D. inhibit the discharge of impulses in large motor fibers
E. inhibit the discharge of impulses in small motor fibers
17. The greatest concentration of carbonic anhydrase is found
A. in plasma
B. in platelets
C. in leukocytes
D. in erythrocytes
E. equally distributed between plasma and erythrocytes
18. Electronegativity in living cells requires that
A. a potential for charge separation exists
B. biologic water assumes a crystalline pattern at the membrane
C. negative charges outnumber positive charges within the cell's cytoplasm
D. all of the above
E. none of the above
19. The rate of fluoride incorporation into bones of an animal depends upon
A. age of the animal
B. water-mineral ratio of the bone
C. rate of bone remodeling or turnover
D. amount of ionizable fluoride in the diet
E. all of the above
20. In mammals, the LAST stage of biologic oxidation is accomplished with
A. NAJD+
B. ubiquinone
C. flavoprotein
D. cytochrome C
E. cytochrome oxidase
21. Inhibition of lipolysis, stimulation of protein synthesis and increased entry
of glucose into muscles and adipose tissues are biologic actions of the
hormone
A. insulin
B. cortisol
C. glucagon
D. epinephrine
E. testosterone
22. Phosphorylation of some enzymes by ATP results in conversion of the
enzyme from an active to an inactive form. This is illustrated by the conversion
of
A. pepsinogen to pepsin
B. trypsinogen to trypsin
C. glycogen synthetase I to glycogen synthetase D
D. all of the above
E. none of the above
23. In untreated diabetes mellitus, polyuria is related to

A. the osmotic effect of glucosuria


B. decreased secretion of antidiuretic hormone
C. increased active transport of sodium by renal tubules
D. the direct effect of insulin on tubular reabsorption of water
E. none of the above
24. During passage of an impulse across either normal muscle or nerve fibers,
which of the following phenomena are exhibited? A(n)
I. electrical response
II. loss of impulse strength caused by resistance
III. increase in consumption of intracellular glucose
IV. increase in permeability of the cell membrane
V. inflow of extracellular sodium
The CORRECT answer is:
A. I, II
B. I, III, IV C. I, IV, V
D. II, III, IV
E. III, V
25. Which of the following are energy-rich phosphate carriers?
I. Glucose-6-phosphate
II. 1,3-diphosphoglyceric acid
III. Creatine phosphate
IV. 3-phosphoglyceric acid
V. Adenosine monophosphate
The CORRECT answer is:
A. I, II
B. I, IV
C. II, III
D. III, V
E. IV, V
KEY (CORRECT ANSWERS)
1.A
2.A
3.D
4.A
5.A

11.D
12.C
13.E
14.C
15.B

6.C
7.A
8.C
9.D
10.C

16.B
17.D
18.A
19.E
20.E
21. A

DIRECTIONS:

22.
23.
24.
25.

C
A
C
C

Each question or incomplete statement is followed by several suggested answers or


completions. Select the one that BEST answers the question or completes the statement.
PRINT THE LETTER OF THE CORRECT
ANSWER IN THE SPACE AT THE RIGHT.
1 Which of the following phospholipids would most likely be found in the outer
lamella of the plasma membrane?
I. Sphingomyelin
II. Phosphatidyicholine
III. Phosphatidylethanolamine
The CORRECT answer is:
A. I, II
B. I, III
C. II, III
D. I, II, III
E. none of the above
2 Which of the following describe an asymmetric model of membrane
assembly?
I. Some membrane proteins may have their N-terminal residues predominately on one
side of the membrane.
II. The polar head groups of the phospholipids may be primarily oriented toward one side
of the membrane.
III. The membrane has an inverted orientation.
The CORRECT answer is:
A. I, II
B. I, III
C. II, III
D. I, II, III
E. none of the above
3. A diet rich in tryptophan offsets a deficiency of
A. niacin
B. thiamine
C. riboflavin
D. all of the above
E. none of the above
4. The effect of a vitamin C deficiency on the developing tooth is primarily on
the calcification of dentin and
cementum. This can BEST be explained by which of the following statements.
A. This does happen, but the reason is obscure.
B. Dentin is the most sensitive tissue in the body to a vitamin C deficiency.
C. Vitamin C influences the formation of collagen, the organic matrix found in dentin and
cementum.
D. Vitamin C has metabolic interrelations with other vitamins that have a greater
influence on dentin and
cementum than on enamel.
E. The vascular system is more important to these tissues than to enamel and, in an
avitaminosis C, the
vascular system is subject to hemorrhage.
5. Which of the following is a vitamin?
A. Adenine
B. Inosine
C. Xanthine
D. Thyroxin
E. Pyridoxine

6. Reactions that have unfavorable energetics (i.e., + G) in metabolic


pathways may be driven to completion by
A. coupling
B. allosterism
C. modification
D. microscopic reversibility
E. none of the above
7. The primary structure of proteins is BEST described by the fact that
proteins
A. are amino acids
B. are polymers of dextrorotatory
C. include all nitrogen compounds of tissues
D. are polymers formed by hydrogen bonding between amino acids
E. are polymers formed by amide linkages between alpha carboxyl groups and alphaamino groups
8. Which of the following is a source of transferable methyl groups in
metabolism?
A. Valine
B. Choline
C. Leucine
D. Isoleucine
E. none of the above
9. Protein content of enamel from mature teeth is approximately what percent of enamel
weight?
A. 0.1-1%
B. 5-10%
C. 15-20%
D. 25-30%
E. 50-55%
10. Starting from the receptor, which of the following BEST represents the arc
of the Hering-Breuer reflex?
A. Lung-vagus-medulla-vagus-bronchial muscles
B. Carotid body-vagus-medulla-spinal cord-bronchial muscles
C. Lung-vagus-medulla-spinal cord-respiratory muscles
D. Lung-glossopharyngeal nerve-spinal cord-bronchial muscles
E. Carotid body-glossopharyngeal nerve-medulla-spinal cord-respiratory muscles
11. In which segment of the nephron does tubular fluid have the HIGHEST
osmolality?
A. Henle's loop
B. Distal tubule
C. Proximal tubule
D. Collecting duct
E. Bowman's capsule
12. Which tissue is metabolically inert (not in dynamic equilibrium with blood
and metabolites)?
A. Bone
B. Dentin
C. Adipose
D. Connective

E. None of the above


13. Fat in the small intestine
A. hastens gastric emptying through activity of the hormone, enterogastrone
B. inhibits gastric emptying through activity of the hormone, enterogastrone
C. initiates the enterogastric reflex that inhibits further gastric emptying
D. initiates the enterogastric reflex that stimulates further gastric emptying
E. none of the above
14. Which of the following portions of an ECG is related to
cardiac impulse between the SA node and the AV node?
A. QRS complex
B. PR interval
C. QS interval
D. QR interval
E. T wave

propagation of the

15. Which of the following vitamins is necessary as a coenzyme in the initial


steps of fatty acid synthesis?
A. Biotin
B. Thiamine
C. Vitamin A
D. Vitamin D
E. Riboflavin
16. A patient has cheilosis, angular stomatitis, glossitis and red, itching eyes.
The periodontal surgery that was performed one week ago has not healed
properly. Deficiency of which of the following vitamins is most likely?
A. Thiamine
B. Riboflavin
C. Tocopherol
D. Calciferol
E. Pantothenic acid
17. Which of the following is NOT a part of accommodation for near vision?
A. Constriction of pupils
B. Convergence of eyeballs
C. Release of visual purple
D. Contraction of ciliary muscle
E. None of the above
18 Which of the following is NOT a reflex mediated by the medulla
oblongata?
A. Blinking
B. Coughing
C. Vomiting
D. Patellar
E. Swallowing
19 Which of the following agents is NOT likely to be found in plasma?
A. Thrombin
B. Fibrinogen
C. Prothrombin
D. Calcium ion
E. Ascorbic acid

20. Which of the following is NOT a function of the liver?


A. Detoxification
B. Gluconeogenesis
C. Formation of plasma protein
D. Secretion of digestive enzymes
E. Regulation of blood sugar level
21. Which of the following does NOT occur when anesthetics are applied to a
neural membrane?
A. K flux remains unchanged.
B. Pores of the membranes become "frozelf."
C. There is a Ca++ flux through the membrane.
D. Resting potential drops to a more negative value.
E. None of the above
22. Which of the following is NOT involved as a cofactor in formation of acetyl
CoA from pyruvate?
A. NAD+
B. FAD
C. Pyridoxine
D. Lipoic acid
E. Thiamine pyrophosphate
23. Which of the following statements concerning collage is INCORRECT?
A. Collagen has a trihelical structure.
B. The molecular weight of collagen is above 100,000.
C. Hydroxyproline is incorporated into the molecule by tRNA.
D. Destruction of collagen can be caused by collagenases.
E. Collagen contains both hydroxyproline and hydroxylysine residues.
24. Which of the following hormones exerts the LEAST effect on calcium
metabolism of bone tissue?
A. Androgen
B. Estrogen
C. Norepinephrine
D. Thyroid hormone
E. Parathyroid hormone
25. The MAJOR protein produced by the odontoblast and contained in the
organic matrix of dentin is
A. chitin
B. keratin
C. elastin
D. collagen
E. cellulose
KEY (CORRECT ANSWERS)
1 A
2 A

11. A
12. E

3 A
4 C

13. B
14. B

5 E

15. A

6. A
7. E
8. B
9. A
10. C
21.
23.
24.
25.

D
C
C
D

16. B
17. C
18. D
19. A
20. D
22. C

EXAMINATION SECTION

TEST l
DIRECTIONS:
Each question or incomplete statement is followed by several suggested answers or
completions. Select the one that BEST answers the question or completes the statement.
PRINT THE LETTER OF THE CORRECT
ANSWER IN THE SPACE AT THE RIGHT.
1. To be President of the United States, a person MUST be a
A. citizen for at least ten years
B. citizen for at least fifteen years
C. citizen for at least twenty-five years
D. natural-born citizen
2. President Reagan was constitutionally forbidden from serving another term
as President because of the
Amendment.
A. Twenty-first
B. Twenty-second
C. Twenty-third
D. Twenty-fourth
3. Constitutionally, the Legislative Branch is provided for in Article
A. I
B. II
C. III
D. IV
4. The writ of habeas corpus is a court order providing that
A. the state can hold a prisoner indefinitely without trial
B. the state MUST show sufficient evidence for holding a prisoner
C. a prisoner's sentence MUST be reduced
D. a prisoner's sentence be increased
5 .If Texas enters into an agreement with Mexico, this agreement has to be
approved by
A. Congress
B. the President
C. the Supreme Court
D. no one
6 The Supreme Court handed down the separate but equal ruling in 1896. The
case was
A. Plessey vs. Ferguson
B. Brown vs. the Board of Education, Topeka
C. White vs. Texas
D. the Dred Scott case

7. The function of a grand jury is to


A. determine guilt
B. determine sentence for the guilty
C. determine if there is sufficient evidence against the accused for a trial to be held
D. advise the judge
8. If the Vice President of the United States should resign and the President
should die, who would become President of the United States?
A. Secretary of State
B. President of the Senate
C. Speaker of the House
D. Chief Justice of the Supreme Court
9 The governors of the Federal Reserve Board hold their
positions on the
basis of
A. Presidential appointment
B. Congressional appointment
C. direct election
D. election by member banks
10. Constitutionally, the Executive Branch is provided for in Article
A. I
B. II
C. III
D. IV
11. To--------------- is NOT one of the powers specifically granted to Congress by
the Constitution.
A. make laws governing land and naval forces
B. coin money and regulate its value
C. borrow money
D. pass laws concerning marriage, adoption, divorce, and inheritance
12. The presiding officer of the House of Representatives is called the
A. House Majority Leader
B. President of the House
C. President pro tempore
D. Speaker of the House
13. The -------------- has two titles and duties to perform in two branches of
government.
A. President
B. Vice President
C. Senate Majority Leader
D. Secretary of State
14. The ------------- has the constitutional power to advise and consent to
foreign treaties.
A. House of Representatives
B. Senate
C. President
D. Supreme Court
15. The-------------has the constitutional power to choose the President if no
candidate receives a majority of the electoral votes cast in a national election.
A. House of Representatives
B. Senate

C. President
D. Supreme Court
16. The------------------ has the constitutional power to act as jury in cases of
impeachment of federal officials.
A. House of Representatives
Supreme Court

B. Senate

C. President

D.

17. The------------------has the constitutional power to impeach


federal
officials.
A. House of Representatives
B. Senate
C. President
D. Supreme
Court

18. All bills concerning the raising of revenue must FIRST originate in
A. the House of Representatives
B. the Senate
C. a majority of state legislatures
D. the Department of Internal Revenue
19. Which of these offices is NOT filled by presidential appointment?
A. Ambassadors
B. Postmasters
C. Representatives
Federal judges
20. The office of---------------------citizens.
A. Senator
B. Representative
C. President
D. All of the above

D.

may be held only by natural-born U.S.

21. Which headline shows the operation of Section 2 of the Twenty-fifth


Amendment, which is printed below?
Section 1.
In case of the removal of the President from office or his death or resignation,
the Vice- President shall become the President.
Section 2.
Whenever there is a vacancy in the office of the Vice-President, the President
shall nominate
a Vice-President who shall take the office upon the confirmation by a majority
vote by
both houses of Congress.
A. JOHNSON TAKES OATH FOLLOWING KENNEDY'S ASSASSINATION IN DALLAS
B. NIXON RESIGNS! SUCCESSOR TAKES OATH IN QUIET CEREMONY
C. CONGRESS APPROVES FORD AS AGNEW'S REPLACEMENT

D. NIXON CARRIES ON WHITE EISENHOWER RECOVERS FROM HEART ATTACK


22. Which shows the Great Seal of the United States?

23. The passage at the right is


self-evident, that all
taken from the
are endowed
A. Resolutions of the Stamp
certain unalienable
are Life, Liberty and
Act Congress
That to secure these
B. Declaration of Independence
instituted among Men
C. Constitution of the United
the consent of
whenever any form of
States
of these ends,
D. Bill of Rights
to alter or to

We hold these truths to be


men are created equal, that they
by their Creator with
Rights, that among these
the pursuit of Happiness.
rights, Governments are
deriving their just powers from
the governed; that,
government becomes destructive
it is the right of the people
abolish it... .

24. According to the passage in the previous question, the PRIMARY purpose
of government is to
A. defend its people against foreign attack
B. guarantee the right of the people to overthrow any form of government they no longer
have faith and
trust in
C. protect private property
D. ensure that the people governed have the free exercise of certain rights

25. In which of these cases is the flag of the United States


displayed?
A.

B.

C.

improperly used or

D.

KEY (CORRECT ANSWERS)


1. D
8. C
14.B

2. B
7. C

3.A

9. A
15. A
21. C

10. B
16. A
22. A

17. A
23. B

4.B

5. A

6. A

11. D
18. A
24. D

12. D
19. C
25. C

13. B
20. C

DIRECTIONS:
Each question or incomplete statement is followed by several suggested answers or
completions. Select the one that BEST answers the question or completes the statement.
PRINT THE LETTER OF THE CORRECT
ANSWER IN THE SPACE AT THE RIGHT.
1. Which point BEST describes a republican form of government?
A. The administrations of Presidents Eisenhower, Nixon, and Ford
B. Rule by a federal chief executive whose office is elective
C. Rule by popularly elected representatives, subject to constitutional limitations
D. Rule by responsible leaders who rise from the ranks of the common people

2. Which of these is characteristic of a democratic form of government?


A. Authoritarian leadership
B. Individual citizens responsible to federal, state, and local levels of government
C. Communications media owned and operated by the government
D. Governmental policies subject to criticism and protest
3. Our first blueprint of government was discarded because it failed to
provide a strong central government for the newly formed United States of
America. This document was
A. a Plan for the Governance of the United Colonies of
North America
B. the Declaration of Independence
C. the Articles of Confederation
D. the Constitution of the United States of America
4. The MOST serious debate at the Constitutional Convention arose over the
issue of how many representatives each state would have in the law-making
branch of the new government.

--------------------was NOT part of the so-called Great Compromise, agreed upon


to settle the problem of state representation.
A. A two-house legislature (Congress) was established
B. Each state would have equal representation in both houses of Congress
C. Each state would have two senators
D. Representation in the House would be based upon a state's population
5 Which point is considered improper in the American
political tradition?
A. The majority rules, but the rights of minorities should be protected insofar as possible.
B. Powerful interest groups attempt to sway public opinion for or against proposed
legislation.
C. Political compromise can be achieved if all parties agree to make concessions and
abide by the terms of
the settlement reached.
.,
D. Military personnel are responsible for the formulation and implementation of foreign
policy
6 The principle that the power to govern belongs to the
delegated by them to their elected officials is called
A. popular sovereignty
B. separation of powers
C. federalism
D. national supremacy

people and is

7. The legitimate object of Government is to do for a community of people


whatever they need to have done but cannot do at all, or cannot so well do,
for themselves in their separate and individual capacities. But in all that
people can individually do for themselves, Government ought not to interfere.
In this quotation, Abraham Lincoln clearly supports the principle of
A. limited government
B. popular sovereignty
C. the supremacy of the federal government over state governments
D. separation of powers

8.-----------------------is basic to the idea of a federal system of


government.
A. Creation of a strong executive branch of government
B. Division of powers between a national government and state governments
C. Distribution of powers between the Senate and the House of Representatives
D. Establishment of an independent Supreme Court
9. The organization of the Judicial Branch is the responsibility of
A. the Supreme Court
B. the President
C. Congress
judicial committee

D. a

10. The Fifth Amendment to the Constitution is often cited by people


appearing before Congressional investigating committees. This amendment
provides that

A. an individual does NOT have to give incriminating evidence against himself


B. an individual has a right to appear before a Congressional committee
C. certain procedures MUST be followed by an investigating committee
D. none of the above
11. Before a person can be convicted of treason, there MUST be
to the same overt act.
A. one
B. two
C. four
D. six

witnesses

12. The Speaker of the House of Representatives is


A. politically neutral
B. leader of the majority party
C. leader of the minority party
D. Vice-President of the United States
13. Congress performs several different types of functions.
judicial function is
A. impeachment
B. providing of funds for the judiciary
C. approval of the appointment of judges
D. providing limits to the jurisdiction of the United States courts

An example of a

14. The admission of new states into the Union is a Constitutional


responsibility of the
A. House of Representatives
B. Senate
C. Congress
Secretary of the Interior

D.

15. The Speaker of the House of Representatives is


A. elected by the House of Representatives
B. appointed by the President
C. the Vice President of the United States
D. the leader of the minority party
16. The 23rd Amendment to the Constitution
A. provides that the President cannot serve more than two terms
B. gives Washington, D.C. residents the franchise in
Presidential elections
C. repeals the Sixteenth Amendment
D. repeals the Nineteenth Amendment

17. How many Representatives does Hawaii send to the United


of Representatives?
A. One
B. Two
C. Three
D. Four
18. A United States Supreme Court justice serves a term of
A. six years
B. ten years
C. twenty-five years

States House

D. life

19. The late President Kennedy statedthat Federal assistance to parochial


schoolswas unconstitutional.

His reasoning is based on the ----------------- Amendment.


A. First
B. Second
C. Third
D. Fourth
20. Before a person can become President of the United States, he MUST have reached
the age of
A. 21
B. 25
C. 30
D. 35
21. The right of the government to take private property for public use,
providing just compensation is made, is
A. habeas corpus
B. ex post facto law C. bill of attainder
D.
eminent domain
22. Which diagram below shows the relationship that exists between national
and state levels of government in a federal system?

A.

B.

C.

D.

23. Your case will not come to trial, said the judge, until you are represented
by legal counsel. You MUST stay in jail without bail until you can raise funds to
hire an attorney. It may take months or years for the case to come to trial, but
the court will wait patiently. If you are the accused in this situation,
A. you are being denied one basic right guaranteed in the Bill of Rights
B. you are being denied several basic rights guaranteed in the Bill of Rights
C. you have no legal cause for complaint
D. since you have been accused of committing a felony, you have forfeited your legal
rights until your
innocence has been established
24.----------------- branch of government is PRIMARILY responsible for enforcing
federal laws.
A. Executive
B. Military
C. Legislative
D. Judicial
25. To qualify for the office of , you must be at LEAST twenty-five years old,
have been an American citizen for seven years, and be a resident of the state
in which you are elected.
A. Senator
B. Representative
C. Supreme Court Justice D. President
1. C
11. B
12. B
13. A
14. C

2. B
3. C
4. B

KEY (CORRECT ANSWERS)


5. D
15. A
16. B
17. A
18. D

6. A
7. A
8. A

9. C
19. A
20. D

10. A
21. D
22. B
23. B
24. A

25. B
TEST 3
DIRECTIONS:
Each question or incomplete statement is followed by several suggested answers or
completions. Select the one that BEST answers the question or completes the statement.
PRINT THE LETTER OF THE CORRECT
ANSWER IN THE SPACE AT THE RIGHT.

1 The fate of a bill in Congress is usually determined


A. in the committee stage
B. by the Speaker of the House
C. by a special calendar committee
D. by the party leadership
2. Treaties MUST be approved by
A. the Senate
B. the House of Representatives
C. both houses in joint session
D. no one other than the President
3. How many Representatives does each state send to the House of
Representatives?
A. Two
B. Four
C. Number in proportion to the population of the state
D. An equal number of Representatives, as determined by the Congress
4. A charge of impeachment against a President would be tried by
A. the Supreme Court
B. the Senate
C. the House of Representatives
D. a special court
5. Each state of the United States is represented in the Senate by
A. one senator
B. two senators
C. a number of senators in proportion to the population
D. a number of senators equal to the state's votes in the electoral college
6. The United States often makes executive agreements with foreign
states. These agreements must be approved by
A. the Senate
B. the House of Representatives
C. both Houses in joint session
D. no one - they are made strictly on the authority of the President
7. Each party in the two Houses of the Congress is usually represented on
the standing committees
A. equally
B. in proportion to their number of representatives in the corresponding chamber
C. in accordance with permanent rules prescribing exactly the number for each
committee
D. according to the mandate of the Speaker of the House
8. The Office of Management and Budget is responsible to the

A. President

B. Senate

C. Congress

D. Cabinet

9. Candidates for the office of the Presidency are selected formally by


A. direct primary
B. Congressional caucus
C. national party convention
D. committee of governors
10. Congress CANNOT pass a bill of attainder. A bill of attainder is a law
A. increasing the sentence of a person serving a prison term
B. imposing punishment on a person WITHOUT benefit of trial
C. imposing unreasonable restrictions on aliens
D. that attempts to regulate a problem under state jurisdiction
11. Constitutionally, the President's power over legislation
A. does NOT exist
B. can be exercised by the veto
C. is exercised by his legal right to vote in Congress
D. none of the above
12. The Vice President's Constitutional duty in Congress is to
A. represent the President in Congress
B. serve as President of the Senate
C. serve as Speaker of the House
D. convene the Congress
13. The number of Supreme Court justices is
A. six
B. nine
C. twelve
D. fifteen
14. The selection of party candidates to run in a general election within a
state is usually done by
A. the party leader of the state
B. the state legislature
C. primary election
D. a nominating committee
15. Before a person can be elected to the House of Representatives, he
MUST have reached the age of
A. 21
B. 25
C. 30
D. 35
16. The President of the United States can make executive agreements
with foreign countries.
These agreements must be
A. approved by the Senate
B. approved by the House of Representatives
C. approved by the Cabinet
D. made solely on the authority of the President
17. The MOST common basic unit of local government in the United States
is
A. city
B. county
C. parish
D. district
18. If a Supreme Court Justice delivers an opinion which agrees with the
opinion of the other judges, but he does NOT arrive at his decision in the
same way, his opinion is referred to as
A. concurring
B. dissenting
C. majority
D. none of
the above
19. The regulation of elections in the United States rests primarily with the

A. President of the United States


B. United States Congress
C. state legislatures
D. Supreme Court

20. In the end, what authority determines who will hold a seat in the
House of Representatives?
A. Supreme Court
B. Court of Appeals
C. House of Representatives itself
D. President
21. The Secretaries of the various departments in the Executive Branch are
selected by
A. the Senate
B. the House of Representatives
C. the President
D.
popular vote
22. Marbury vs. Madison is known historically because of
A. interpretation of the contract laws
B. the guarantee of civil rights
C. equal protection clause
D. principle of judicial review
23. The National Guard in the United States during times of peace is under
the jurisdiction of the
A. Chief Executive of the state
B. President of the United States
C. Chairman of the Joint Chiefs of Staff
D. Secretary of Defense
24. The number of people on a grand jury ranges from
A. 3 to 6
B. 6 to 12
C. 12 to 23
D. 23 to 30
25. Before a person can be elected to the United States Senate, he MUST have
reached the age of
A. 21
B. 25
C. 30
D. 35
KEY (CORRECT ANSWERS)
1. A
2. A
3. C

11. B
12. B
13. B

4. B
5. B

14. C
15. B

6. D
7. B
8. A

16. D
17. B
18. A

9. C
10.B

19. C
20. C

21. C
22. D
23. A

24. C
25. C

TEST 4
DIRECTIONS:
Each question or incomplete statement is followed by several suggested answers or
completions. Select the one that BEST answers the question or completes the statement.
PRINT THE LETTER OF THE CORRECT
ANSWER IN THE SPACE AT THE RIGHT.
1. The United States Supreme Court handed down a decision relating to
slavery. The case was
A. Plessey vs. Ferguson
B. Brown vs. the Board of Education, Topeka
C. White vs. Texas
D. the Dred Scott case
2. The President's Cabinet is made up of
A. non-officials of government
B. party leaders from the Senate and House of Representatives
C. Secretaries of the major departments in the Executive Branch
D. anyone the President wishes to choose, regardless of his position
3. Each state sends Senators to the United States Senate.
A. two
B. four
C. six
D. a number in proportion to the population of the state
4. The District Court is a court of
A. original jurisdiction only
B. appellate jurisdiction only
C. both original and appellate jurisdiction
D. none of the above
5. The states in the United States system of government enjoy
-------------------power(s).
A. delegated
B. reserved
C. only concurrent
D. no particular grant of constitutional
6. Each state has a number of electors in the Electoral College equivalent to
the number of
A. Representatives in the United States House of Representatives
B. Senators and Representatives in the United States Congress
C. its state legislators
D. delegates sent to the national party conventions

7. The Supreme Court ruling rendering unconstitutional segregation in the


public schools was
A. Plessey vs. Ferguson
B. Brown vs. the Board of Education, Topeka
C. White vs. Texas
D. the Dred Scott case
8. Just prior to ratification of the 26th Amendment, MOST states required a
person to be 21 years of-age to vote, but two states required that a person be
only 18. These states were
A. Kansas and Nebraska
B. Georgia and Kentucky
C. Texas and Louisiana
D. Wyoming and Utah
9. The Congress does MOST of its legislative work in committees.
A. standing
B. joint
C. conference
D. select
10. Each Court of Appeals has judges.
A. one
B. three
C. six
D. nine
11. In the United States House of Representatives, Puerto Rico has
representative(s).
A. no
B. one
C. two
D. only a nonvoting
12. The Supreme Court has primarily jurisdiction.
A. original
B. appellate
C. equal responsibility for original and appellate
D. none of the above
13. Treaties are ratified by the
A. Secretary of State
B. Senate
C. House of Representatives
D. the
President
14. How many Congressmen does each state send to the House of
Representatives?
A. Two
B. Four
C. Six
D. A number in proportion to the population of the stat
15. A case appealed from a regulatory commission would go FIRST to the
A. Supreme Court
B. Court of Appeals
C. District Court
D. none of the
above
16. Constitutionally, the power to coin money rests with
A. Congress only
B. the state legislatures only
C. Congress and the state legislatures
D. the President of the United States
17. Which statement does NOT apply to the principle of judicial review?
A. Judicial review is the process by which courts interpret the meaning of the
Constitution.

B. Judicial review was established in a famous case in which, for the first time, the
Supreme Court
declared an act of Congress unconstitutional.
C. Judicial review is often used by Congress as a check on the power of the judicial
branch.
D. Judicial review is a powerful check upon both the legislative and executive branches
of government.
18. The qualifications for Supreme Court justices and other federal judges are
A. NOT specified in the Constitution
B. the same as those for the office of President
C. U.S. citizenship and thirty-five years of age
D. a law degree and a knowledge of the U.S. Constitution
19. Which of these American political customs is the o n l y one mentioned
anywhere in the Constitution?
A. How presidential candidates are selected
B. How many votes each state may cast in the electoral college
C. The formation of political parties
D. Holding national political conventions

20. To----------------is NOT a purpose for which an election would normally


be held in the United States.
A. nominate or elect public officials
B. approve or reject constitutional amendments
C. approve or reject a proposed law submitted to a direct popular vote
D. decide whether or not a treaty should be ratified
21. People who vote in public elections
A. perform a civic duty
B. exercise a basic freedom
C. participate in the democratic process
D. all of the above apply to persons who vote in public elections
22. All powers that the Constitution does NOT specifically give to the federal
government or deny to the states are reserved for
A. Congress
B. the President
C. state legislatures
D. the states, or for the people
23. A citizen may be required to pay federal income tax, a state income tax,
and a city income tax.
This shows how different levels of government exercise powers.
A. concurrent
B. reserved C. enumerated
D. implied
24 It is FALSE that state constitutions
A. deal with many specific aspects of governmental service
B. are apt to change more often than the federal Constitution
C. may or may NOT establish republican forms of state government
D. CANNOT violate any provision of the U.S. Constitution
25 A
is responsible for examining evidence and
persons charged with criminal
offenses should be brought to trial.

witnesses to decide if

A. military court martial

B. grand jury C. petit jury

D. judge

KEY (CORRECT ANSWERS)

1.D
2.C
3.A
4.A
5.B

11. D
12. B
13. B
14. D
15. B

6.B
7.B
25. B

16. A
17. C

8.B
9.A
10.B
21.
22.
23.
24.

18. C
19. B
20. D

D
D
A
C

TEST 5
DIRECTIONS:
Each question or incomplete statement is followed by several suggested answers or
completions. Select the one that BEST answers the question or completes the statement.
PRINT THE LETTER OF THE CORRECT
ANSWER IN THE SPACE AT THE RIGHT.
1. Official agreements between states MUST be approved by
A. Congress
B. the President
C. the Secretary of State
D. the Secretary of the Interior
2. An alien can acquire United States citizenship after he has resided in the
United States for -------year(s).
A. one
B. three
C. five
D. ten
3. Congress is forbidden by the Constitution to
A. declare war
B. tax imports
C. borrow money
D. none of the above
4. A system of-------------- BEST describes the Constitution of the
States.
A. unitarianism
B. federalism
C. confederation
D. none of the above
5. The United States court system is based on
A. civil law
B. common law
C. the French legal system

United

D. no one particular system


6. United States Senators serve a term of
A. two
B. four
C. six
D. eight

------------- years.

7. The chief executive officer in ALL the states is the


A. Governor
B. Secretary of State
C. Chief Minister
D. Governor General
8. An individual desiring to acquire United States citizenship would do so by
appearing before a
A. Congressional committee
B. United States court
C. United States immigration officer
D. state legislative committee
9. A declaration of war is made by the
A. President of the United States
B. Secretary of Defense
C. Senate
D. Congress

10. The
Plan presented to the Constitutional Convention was the Large
State Plan.
A. Virginia
B. New Jersey
C. Connecticut
D. Pennsylvania
11. MOST murder cases are tried by the -----------------------------------------court(s).
A. United States District
B. Supreme C. state
D. state supreme
12.-------------------------------------has a one-house legislature.
A. Nebraska
B. Oregon
C. Wyoming D. Indiana
13. ----------------------------------------is NOT a common requirement for voting.
A. Age
B. Citizenship
C. Residence
D. Literacy
14. Constitutionally, Congress is forbidden to
A. levy an income tax
B. levy an export tax
C. regulate interstate commerce
D. print paper money
15. The Bureau of the Budget is in the
A. Treasury Department
B. Congress
C. Executive Office
D. Federal Reserve Bank
16. The authors of the United States Constitution were, for the most part, men
A. without property
B. of property
C. with very little education

D. from the west


17. The President of the United States serves a term of -----------------------years.
A. two
B. four
C. six
D. eight
18. The Small State Plan presented to the Constitutional Convention was
the-------------------------- Plan.
A. Virginia
B. New Jersey
C. Connecticut
D. Pennsylvania
19. A trial jury rendering a verdict MUST do so by a
A. simple majority
B. two-thirds majority
C. three-fourths majority
vote

D. unanimous

20. Puerto Rico's relationship to the United States is


A. a state
B. a commonwealth C. an occupied territory
relationship

D. no legal

21. The Constitution establishes the important principle called separation of


powers by
A. guaranteeing equal justice under law for all citizens
B. guaranteeing that the people shall rule through elected representatives
C. establishing three co-equal branches of government
D. all of the above

22. The system of checks and balances was introduced into the Constitution so
that
A. the President would have only two terms
B. the executive branch would become the dominant branch of government
C. the people would have more direct control over their government
D. no single branch might gain control over the other branches of government
23. A check on the executive branch of government is the power to
A. approve persons appointed to high office
B. veto bills
C. appoint federal judges
D. negotiate treaties with foreign governments
24. The specific written guarantees of our individual liberties are to be found
in
A. the main body of the Constitution
B. the first ten amendments
C. certain later amendments
D. all of the above
25. The constitutional provision "...nor shall [any person] be compelled in any
criminal case to be a witness against himself..." is known as protection against
A. self-incrimination
B. cruel and unusual punishment
C. taking the Fifth Amendment
D. double jeopardy

KEY (CORRECT ANSWERS)

1. A
2. C
3. D
4. B
5. B

11.C
12. A
13. D
14. B
15. C

6. C
7. A
8. B
9. D
10. A

16. B
17. B
18. B
19. D
20. B
21. C
22. D
23. A
24. D
25. A

BIOCHEMISTRY PHYSIOLOGY
EXAMINATION SECTION
TEST 1

DIRECTIONS: Each question or incomplete statement is followed by several suggested


answers or completions. Select the one that BEST answers the question or completes the
statement. PRINT THE LETTER OF THE CORRECT ANSWER IN THE SPACE AT THE RIGHT.
1. The km value of an enzyme is numerically equal to
A. half the maximum velocity expressed in moles/liter
B. velocity of a reaction divided by substrate concentration
C. substrate concentration in moles/liter necessary to achieve half the maximum velocity
of a reaction
D. maximum velocity divided by half the substrate concentration in moles necessary to
achieve maximum velocity
E. substrate concentration divided by velocity of a reaction
2. The physiologically active form of vitamin D produced in the kidney is
A. 1,25-dihydroxycholecalciferol
B. 25-hydroxycholecalciferol
C. 7-dehydrocholesterol
D. cholecalciferol

E. ergosterol
3. Which of the following is an example of reduction?
A.
-CH = CH- + H2O ------ -CH2 - CHOH+
B.
R COOH
------
R-000- + H
C.

- CH2CH2

D.

Cu

E.

Fe

+3

+e

------

- CH = CH + H2

------

Cu

------

Fe

+2

+ e-

+2

4. The ketone body acetoacetate is synthesized inside mitochondria of


hepatocytes by
A. carboxylation of pyruvic acid
B. transamination of aspartic acid
C. cleavage of -hydroxy-(3-methylglutaryl CoA
D. oxidative decarboxylation of a-ketoglutarate
E. oxidation of L-fi-hydroxybutyrate followed by deacylation
5 Acids found in the citric acid (Krebs) cycle are --------------- and
--------------------------acids.
A. glutamic; succinic
B. glucuronic; aspartic
C. oxaloacetic; aspartic
D. oxaloacetic; pyruvic
E. oxaloacetic; a-ketoglutaric

6. The presence of glucose in the urine proves that the


patient has
A. diabetes mellitus
B. negative nitrogen balance
C. exceeded his renal threshold for glucose
D. excessive damage to a glomerulus of the kidney
E. none of the above
7. Carbonic anhydrase in kidney tubular cells is associated with reabsorption of
A. urea
B. chloride
C. carbohydrate D. bicarbonate ion
E. water
8. An impulse can travel from one nerve to another in ONLY one direction
because the
A. synapse limits the direction of travel
B. myelin sheath limits the direction of travel
C. myoneural junction limits the direction of travel
D. nerve fiber permits conduction in only one direction
E. cell body must be stimulated before the nerve fiber will conduct

9. The intracellular, second messenger for many peptide and polypeptide


hormones is
A. AMP
B. ATP
C. cyclic AMP
D. adenylate cyclase
E. a cytoplasmic receptor
10. Human parotid saliva is believed to be hyposmolar because
A. water is reabsorbed by excretory ducts of the gland
B. acinar cells transport water more readily than sodium ions
C. striated duct cells produce excess water and retain potassium
D. reabsorption of water by striated duct cells is less than reabsorption of sodium
E. none of the above
11. Which of the following is the MAJOR contributor to colloid osmotic
pressure?
A. Albumin
B. Alpha globulin
C. Beta globulin
D. Gamma globulin
E. None of the above
12. Some sense organs, when stimulated continuously by a constant stimulus,
may gradually lose their ability to continue to respond. This phenomenon is
known as
A. occlusion
B. summation
C. adaptation D. facilitation
E. sensory
deprivation

13. The Henderson-Hasselbalch equation shows that


A. dilution of a buffer increases its pH
B. pH is a function of temperature
C. pH = pka when an acid is tenth normal
D. pH = pka when an acid is half neutralized
E. pH is independent of the dissociation constant of the acid
14. In the absence of compensatory changes, a drop in blood pressure results from
A. vasoconstriction
B. increased hematocrit
C. increased stroke volume
D. increased cardiac output
E. decreased venous return
15. DNA damage by ultraviolet light is due to
A. alkylation of the guanine in DNA
B. excessive unwinding of the DNA molecule
C. frequent replacement in the DNA molecule of purines by pyrimidines
D. induction of dimerization by way of covalent bonds between adjacent thymine groups

E. all of the above


16. Sucrose is a glycoside of which of the following structures?
A.
4-Dglucosea-Dgalacto
pyranoside
B.
4-Dglucose-Dgalacto
pyranoside
C.
Alfa-Dgluco
pyranosido- f3-Dfructo
furanoside
D.
Beta-Dgluco
pyranosido-- Dfructo
furanoside
E.
Gamma-D- gluco
pyranosido- a-Dfructo
furanoside
17. Which of the following is a function of vitamin B6 (pyridoxal phosphate)?
A. Prevents pernicious anemia
B. Allows adaptation to dim light
C. Acts as coenzyme in transamination reactions
D. Prevents microcytic hypochromic anemia
E. All of the above
18. Carbon monoxide DECREASES the amount of
A. bicarbonate in the blood plasma
B. carbonic anhydrase available in the lungs
C. oxygen that can be transported by hemoglobin
D. alveolar surface available for gaseous exchange
E. carbon dioxide that can be transported by the blood
19. The MOST likely cause of an increase in filtration fraction is
A. elevation of ureteral pressure
B. efferent arteriolar constriction
C. afferent arteriolar constriction
D. elevation of renal vein pressure
E. elevation of plasma protein concentration
20. The two chains of double stranded DNA are so arranged that
A. they are parallel
B. heat does not affect interchain bonds
C. hydrophobic aromatic nitrogen bases are held close to each other
D. purines and pyrimidines alternate along each chain
E. an adenine residue on one chain is directly hydrogen bonded to a cytosine on the
other chain
21. A deficiency of rhodopsin is MOST likely caused by decreased dietary intake
of
A. glucose
B. adenine
C. vitamin A
D. tryptophan
E.
riboflavin
22. Removal of a molecule of water BETWEEN the carboxyl group of one amino
acid and the amino group of a second amino acid results in formation of a
A. zwitterion
B. peptide bond C. hydrogen bond
D. glycosidic bond
E.
hydrophobic bond
23. The PRIMARY effect of calcitonin is to

A. increase bone resorption


B. inhibit bone resorption
C. increase calcium absorption from the intestine
D. decrease calcium absorption from the intestine
E. both increase bone resorption and increase calcium absorption from the intestine
24. The belief that the secretion of saliva is an active process is supported by
the observation that
A. during secretion there is vasodilation within the gland
B. during secretion blood flow through the gland is unchanged
C. during secretion blood flow through the gland is decreased
D. secretion continues even when the pressure within the salivary duct is higher than the
blood pressure
E. secretion stops if the pressure within the salivary duct is the same as the blood
pressure
25. Which of the following enzymes catalyzes the formation of uric acid from
purines?
A. Urease
B. Uricase
C. Xanthine oxidase
D. Aspartate transcarbamylase
E. Carbamyl phosphate synthetase
26. The process by which extensor muscles acting at a particular joint are
inhibited by stretch of the flexor muscle acting at the same joint is
A. myotatic reflex
B. clasp-knife reflex
C. monosynaptic reflex
D. lateral inhibition
E. reciprocal inhibition
27. Carbon dioxide or bicarbonate is required in the biosynthesis of fatty acids
because
A. the bicarbonate/carbonic acid buffer system is very efficient at the pH optimum of
this sensitive enzyme
system
B. bicarbonate is a positive effector for this system and favorably alters the
conformation of its enzymes
C. carbon dioxide is incorporated into acetyl coenzyme A forming malonyl coenzyme A,
an intermediate in the synthetic process
D. carbon dioxide is incorporated into carbamyl phosphate, a reactive intermediate in
the synthetic process
E. carbon dioxide provides an anaerobic environment that prevents oxidation of the
sulfhydryl groups in the
reactive sites of the enzyme system

28. Any substance that is filtered by glomeruli and secreted, but not
reabsorbed, by renal tubules will have a renal clearance that is
A. equal to the clearance of urea
B. equal to the clearance of inulin
C. less than the clearance of inulin

D. greater than the clearance of inulin


E. equal to the glomerular filtration rate
29. Energy for ATP synthesis is derived from the electron transport system by
which of the following processes?
A. Transamination
B. Aldolization
C. Reductive synthesis
D. Oxidative deamination
E. Oxidative phosphorylation
30. Which of the following statements is TRUE concerning fatty acid synthesis
and fatty acid breakdown?
A. Both are mitochondrial functions
B. Both require phosphopantothenic acid
C. Both share common electron donors and acceptors
D. Both share common enzymes for oxidation and reduction reactions
E. All of the above
KEY (CORRECT ANSWERS)
1.
2.
3.
4.
5.

C
A
E
C
E

6. C
7. D
8. A
9. C
10. D

11. A
12. C
13. D
14. E
15. D

21. C
22. B
23. B
24. D
25. C

16. C
17. C
18. C
19. B
20. C

26. E
27. C
28. D
29. E
30. B

TEST 2
DIRECTIONS: Each question or incomplete statement is followed by several suggested
answers or completions. Select the one that BEST answers the question or completes the
statement. PRINT THE LETTER OF THE CORRECT ANSWER IN THE SPACE AT THE RIGHT.
1. The function of rhythmic segmentation is CHIEFLY that of
A. macerating food

B. mixing food with digestive secretions


C. increasing the surface available for absorption
D. transporting material down the gastrointestinal tract
E. all of the above
2. Lipids are MOST characteristic of which of the following cellular
components?
A. Cytoplasm
B. Ribosomes
C. Membranes
D. Chromosomes
the above

E. All of

3. Which of the following is TRUE of the Tm (melting temperature) of a given


DNA double helix?
A. Is a function of the base composition
B. Can be used to accurately predict its molecular weight
C. Can be measured by observing the change in chemical composition
D. Is higher if individual strands of the DNA double helix are parallel rather than
antiparallel
E. None of the above
4. A submaximal DIRECT stimulus to a skeletal muscle causes
A. contraction of no fibers
B. contraction of some fibers
C. partial contraction of all fibers in the muscle
D. less contraction than with a subminimal stimulus
E. more contraction than with a maximal stimulus
5. Over a period of time, a man uses 4 liters of oxygen and produces 3 liters of
CO2.
What is his RQ for that period?
A. 0.75
B. 1.00
C. 1.33
D. 5.00
E. 7.00
6. The gallbladder is caused to contract by the hormone
A. gastrin
B. secretin
C. pancreozymin
D. enterogastrone
E. cholecystokinin
7. The partial pressure of oxygen in arterial blood is lower
than that in
alveoli. The condition suggested is
A. hyperventilation
B. slow blood flow
C. high blood pressure
D. inadequate ventilation
E. thickening of the alveolar membrane
8. Which of the following tissues is PRIMARILY responsible
for formation of
urea?
A. Liver
B. Brain
C. Kidney
D. Muscle
E. Bladder
9. In which of the following situations is an osmotic diuresis observed?
A. During diabetes mellitus
B. During water deprivation
C. Following ingestion of starch
D. Following injection of Pitressin
E. None of the above
10. Which of the following causes SOME degree of temporary alkalosis?
A. Hyperventilation

B. High fluid intake


C. Excessive smoking
D. Severe muscular effort
E. Ingestion of ammonium chloride
11. Which of the following enzymes is involved in the termination of
neuromuscular transmission?
A. Choline acetylase
B. Monoamine oxidase
C. Methyltransferase
D. Adenylate cyclase
E. Acetylcholine esterase
12. A muscle devoid of tonus is
A. atonic
B. spastic
C. hypotonic
D. hypertonic
E. none of the
above
13. Which of the following characterizes an asymmetric carbon? A carbon
A. atom with four identical groups attached to it
B. atom with four different groups attached to it
C. with at least one carboxyl and one amino group attached to it
D. atom that has two heavy groups on one side and two light groups on the other
E. with an odd number of groups attached to it
14. Which of the following statements supports the observation that the
glomerulus acts like a simple physical microfilter?
A. Potassium ion is both secreted and reabsorbed by the nephron.
B. Antidiuretic hormone alters permeability of the tubular cell membrane.
C. Glomerular filtration rate is determined by plasma clearance of inulin.
D. Increased concentration of colloids in plasma diminishes the formation of filtrate.
E. The afferent arteriole is greater in diameter than the efferent arteriole.
15. Hydroxylation of proline and lysine during collagen biosynthesis occurs
A. after translation
B. in the mitochondrial matrix
C. before formation of their respective amino acyl-t-RNA's
D. while proline or lysine is bound to the peptidyl (P) site on the ribosome
E. before translation
16. Which of the following is MOST likely to promote depolymerization of the
ground substance?
A. Cortisone
B. Collagenase
C. Chymotrypsin
D. Hyaluronidase
E. Ammonium phosphate
17. Prostaglandins are made within cells
A. on an RNA template
B. on rough endoplasmic reticulum
C. from methionine
D. from progesterone
E. from polyunsaturated fatty acids

18. Thyroid secretion is stimulated by


A. hypoxia

B. propylthiouracil
C. exogenous thyroid hormone
D. high serum iodide concentration
E. prolonged exposure to a cold environment
19. Following exercise, ventilation is in excess of the metabolic requirement.
During this period, the level of lactic acid in blood
A. increases
B. decreases
C. remains unchanged
D. is unrelated to ventilation
E. sometimes increases, sometimes decreases
20. Which of the following vitamins can be supplied to humans by the normal
action of intestinal flora?
A. A
B. D
C. E
D. K
E. None of the above
21. Some amino acids need not be present in the diet of an animal because of
the animal's ability to synthesize the acids at an adequate rate. A PRINCIPAL
source of carbon for these amino acids is
A. nucleic acids
B. carbon dioxide
C. metabolism of carbohydrates
D. methylene folic acid derivatives
E. any of the above
22. Distribution of fluids between intracellular and extracellular compartments
is MAINLY achieved by
A. bulk flow across the capillary wall
B. unrestricted movement of sodium to achieve osmotic balance
C. unrestricted movement of water to achieve osmotic balance
D. unrestricted movement of both sodium and potassium to achieve osmotic balance
E. movement of potassium out of the intracellular compartment to achieve osmotic
balance
23. The sensory organs concerned with maintenance of skeletal muscle tonus
are
A. muscle spindles
B. gamma efferents
C. pacinian corpuscles
D. golgi tendon organs
E. none of the above
24. An electrocardiogram showing extra P waves before each QRS complex
indicates
A. fibrillation
B. atrial bradycardia
C. partial heart block
D. ventricular tachycardia
E. left bundle branch block
25. In which of the following parts of the circulatory system is blood pressure
the LOWEST?
A. Aorta
B. Veins
C. Arterioles
D. Capillaries
E. Large arteries
26. Carbohydrate is stored in the body PRINCIPALLY as
A. glucose
B. maltose
C. sucrose
D. glycogen E.
glycosaminoglycans

27. Within physiologic limits, activity of the respiratory center is DECREASED


directly by
A. decreased pH
B. increased pH
C. increased hemoglobin concentration
D. increased plasma carbon dioxide partial pressure
E. decreased plasma carbon dioxide partial pressure
28. The tone of the masseter muscle is maintained by the
A. flexor reflex
B. stretch reflex
C. crossed-extensor reflex
D. continuous production of metabolic products
E. none of the above
29. The air remaining in alveoli at the end of a forced maximal expiration is
the
A. tidal volume
B. vital capacity
C. residual volume
D. expiratory reserve volume
E. inspiratory reserve volume
30. The plasma membrane can be BEST described as a(n)
A. static trilamellar lipid-protein-lipid sheath
B. unorganized zone between cytoplasm and interstitial fluid
C. membrane whose composition is essentially constant for all mammalian cells
D. fluid mosaic composed of neutral fats and glycoproteins
E. fluid mosaic composed primarily of phospholipids, cholesterol, and glycoproteins
KEY (CORRECT ANSWERS)
1.B
2.C
3.A
4.B
5.A

11. E
12. A
13. B
14. D
15. A

21.C
22.C
23.A
24.C
25.B

6.E
7.E
8.A
9.A
10.A

16. D
17. E
18. E
19. B
20. D

26.D
27.B
28.B
29.C
30.E

TEST 3
DIRECTIONS: Each question or incomplete statement is followed by several suggested
answers or completions. Select the one that BEST answers the question or completes the
statement. PRINT THE LETTER OF THE CORRECT ANSWER IN THE SPACE AT THE RIGHT.
1. The DOMINANT factor controlling absorption of iron from the
gastrointestinal tract is
A. excretion of iron in the urine
B. excretion of iron in the stools
C. saturation of mucosal cells with iron
D. concentration of the ferrous iron in bone marrow, spleen, and liver
E. all of the above
2. Spatial summation results from the
A. convergence of several afferent impulses on the same postsynaptic nerve soma
B. repetitive firing of discharges (from the same pre- synaptic terminal), eventually
producing an action
potential
C. exhaustion of the stores of transmitter substance at the synapse
D. elaboration of some inhibitory substance, such as y-aminobutyric acid, at the synapse
E. none of the above
3. Two atoms are considered isotopic if
A. their nuclei contain the same number of neutrons
B. their atomic numbers are the same, but their mass numbers differ
C. their mass numbers are the same, but their atomic numbers differ
D. one is a beta emitter, and the other an alpha emitter
E. none of the above

4. The distribution of carbonate within dental enamel follows the same surface
to dentinoenamel junction patterns as which of the following?
A. Lead
B. Calcium
C. Fluoride
D. Strontium
E. None of the
above
5. A deficiency of vitamin A in a developing tooth MOST likely affects the
A. pulp
B. enamel
C. dentin
D. cementum
E. both the pulp and the enamel

6. Visual purple, rhodopsin, is USUALLY formed in the


A. lens
B. liver
C. retina
D. adrenals

E. vitreous humor

7 Aldosterone is normally associated with partial regulation of which of the


following processes?
A. Sodium balance
B. Gluconeogenesis
C. Lipid digestion
D. Protein degradation
E. Carbohydrate metabolism
8 The normal delay in sexual development until puberty is attributed to
A. lack of effect of sex hormones on somatic tissue
B. failure of the gonads to respond to gonadotrophins
C. inability of the hypophysis to synthesize gonadotroph ins
D. lack of hypothalamic stimulation of gonadotrophin release
E. inability of the hypophysis to respond to hypothalamic releasing hormones
9. Neutral fats contain mixtures of one or more fatty acids esterified with
A. sterol
B. glycerol
C. lecithin
D. sphingosine
E. alcohols of high molecular weight
10. At rest, the potential difference across the membrane of
following is GREATEST?
A. Odontoblast
B. Sinoatrial node
C. Skeletal muscle
D. Visceral smooth muscle
E. None of the above

which of the

11. Antidiuretic hormone acts to


A. decrease renal filtration fraction
B. increase storage capacity of the bladder
C. decrease permeability of distoconvoluted tubules and/or collecting ducts to water
D. increase permeability of distoconvoluted tubules and/or collecting ducts to water
E. none of the above
12. The rapid movement of a substance across a biologic membrane against a
concentration gradient requires
A. that the substance be negatively charged
B. participation of an energy-requiring active transport system
C. that the substance be readily soluble in the lipid barrier of the membrane
D. that the substance be carried across the membrane by rapid influx of the solvent
E. facilitated diffusion of the substance aided by some binding system in the membrane

13. With glutamic acid as an amino group donor, alanine can be synthesized
directly from which of the following acids?
A. Lactic acid
B. Pyruvic acid
C. Glutaric acid
D. a-ketoglutaric acid
E. 3-phosphoglyceric acid
14. Neutralization of acids by saliva results MAINLY from which of the following
salivary contents?
A. Mucin
B. Ammonia
C. Carbonate
D. Bicarbonate
E. Amino acids
15. Compared with hemoglobin A, the substitution of a valine for a glutamic
acid residue in hemoglobin S results from
A. a genetic mutation
B. irradiation of hemoglobin A
C. proteolytic action in the liver
D. exposure to low oxygen tension
E. irradiation of hemoglobin S
16. Where is norepinephrine stored?
nerve endings.
A. preganglionic sympathetic
B. postganglionic sympathetic
C. preganglionic parasympathetic
D. postganglionic parasympathetic
E. none of the above

At----------------------------------

17. The MOST abundant form of iron in human blood plasma is found
A. as free Fe++
B. as free Fe+++
C. as ferritin
D. associated with transferrin
E. associated with ceruloplasmin
18. The MAIN control over body temperature, located in a portion of the
central nervous system, is the
A. pons
B. cerebellum
C. hypothalamus
D. medulla oblongata
E. cerebrum

19. The second heart sound is related to


A. excitation of atria
B. opening of AV valves
C. closure of AV valves
D. opening of aortic valves
E. closure of aortic valves
20. In males, growth and development of secondary sex organs are under
DIRECT control of
A. FSH and LH
B. aldosterone
C. progesterone
D. testosterone
E. estrogen
21. The relative refractory period of a nerve corresponds to
A. sodium permeability changes
B. decreased potassium permeability
C. increased potassium permeability
D. threshold potential that is farther from zero
E. none of the above
22. The MAJOR center in the brain for autonomic nervous system regulation is
the
A. hypothalamus
B. pituitary
C. cerebral cortex
D. vestibular nuclei
E. floor of the fourth ventricle
23. Which of the following hormones does NOT induce activation of adenylate
cyclase?
A. Insulin
B. Glucagon
C. Epinephrine
D. Parathyroid hormone
E. All of the above
24. Which of the following is LEAST soluble in solutions with low pH?
A. Chlorapatite
B. Fluorapatite
C. Hydroxyapatite
D. Calcium fluoride
E. None of the above
25. All of the following function in buffer systems in the
A. NaCl
B. H2CO3
C. NaHCO3
D. Na2HPO4
E. NaH2PO4

blood EXCEPT

26. Which of the following compounds does NOT contain a high-energy bond?
A. ATP

B. Acetyl CoA
C. UDP-glucose
D. Glucose-6-phosphate
E. Phosphoenolypyruvate

27. Which of the following is NOT a cofactor or a substance


hydroxylation of proline during collagen biosynthesis?
A. Ascorbate
B. Ergosterol
C. Ferrous ion
D. a-ketoglutarate
E. All of the above

involved in

28. Which of the following statements is NOT true about ammonia and the aamino group of amino acids?
A. NH-I-4 is formed from glutamine in the kidney.
B. The amino group in carbamyl phosphate is directly donated by aspartate in a
transamination reaction.
C. Cellular levels of ammonia must be maintained at low concentrations because of its
toxicity.
D. Glutamate dehydrogenase can catalyze the formation of glutamate from ammonia
and a-ketoglutarate using NADPH as a cofactor.
E. None of the above
29. Which of the following is NOT a monosaccharide?
A. Amylose
B. Glucose
C. Fructose
D. Glyceraldehyde
E. Glucuronic acid
30. Which of the following statements is NOT true concerning fluoride?
A. is excreted rapidly by the kidney
B. is deposited in calcified tissues
C. occurs only in the ionic form in plasma
D. passes the placental barrier relatively slowly
E. is tasteless, odorless, and colorless at 1 ppm. in water
KEY (CORRECT ANSWERS)
1. C
2. A
3. B
4. E
5. B

11. D
12. B
13. B
14. D
15. A

21. C
22. A
23. A
24. B
25. A

6. C
7. A
8. D

16. B
17. D
18. C

26. D
27. B
28. B

It

9. B
10. C

19. E
20. D

29. A
30. C

MICROBIOLOGY / PATHOLOGY
EXAMINATION SECTION
TEST 1
DIRECTIONS: Each question or incomplete statement is followed by several suggested
answers or completions. Select the one that BEST answers the question or completes the
statement. PRINT THE LETTER OF THE CORRECT ANSWER IN ONE SPACE AT THE RIGHT.
1. Chronic inflammatory reactions are characterized by the presence of
lymphocytes in the exudate and by
A. production of mucin
B. production of fibrin
C. permeation of serum
D. proliferation of granulation tissue
2. A hypoparathyroid patient is likely to exhibit tetany because the
A. muscle protein is altered
B. blood has a low concentration of serum calcium
C. blood has a low concentration of serum phosphorus
D. patient develops degenerative central nervous system lesions
3. Staphylococci are characteristically grouped in
A. pairs
B. chains
C. tetrads
D. irregular clusters
E. palisade arrangement
4. Rickettsia are distinguishable from viruses because viruses contain
A. DNA; rickettsia contain RNA
B. RNA; rickettsia contain DNA
C. either RNA or DNA; rickettsia contain both RNA and DNA
D. both RNA and DNA; rickettsia contain either RNA or DNA
E. either RNA or DNA; rickettsia contain neither RNA nor DNA
5. Persons having anti-A and anti-B isoagglutinins in their serum belong in the
blood group
A. A
B. B
C. AB
D. 0
6. The Lancefield streptococcus group MOST likely to contain strains
pathogenic for man is
A. A
B. B
C. D
D. E
E. H

7. The microorganism which has been isolated MOST frequently from infected
tooth pulps is
A. yeast
B. antinomyces
C. streptococci
D. lactobacilli E. staphylococci
8. Intensification of the normal red color of tissues in an area of acute
inflammation is the result of
A. liberation of fibrin
B. increased amount of blood in the area
C. extravasation of blood into tissue spaces
D. accumulation of large numbers of neutrophils
E. damage to capillary endothelium resulting in edema
9. The leading cardiac cause of death in the United States is
A. mitral stenosis
B. coronary heart disease
C. myocarditis (all causes)
D. hypertensive cardiovascular disease
E. congenital anomalies
10. A positive tuberculin test means that the patient
A. has active tuberculosis
B. has a subclinical infection
C. is susceptible to tuberculosis
D. has an immunity to tuberculosis as a result of a past infection
E. has a hypersensitivity to the tubercle bacillus and its products
11. Scarlet fever is an acute disease caused by
A. viridans streptococci releasing toxin
B. Salmonella typhosa with endotoxin release
C. group A streptococci releasing erythrogenic toxin
D. a mixture of streptococci and staphylococci with coagulase production
12. Cretinism is the result of a metabolic disturbance related to
A. hypothyroidism
B. hyperthyroidism
C. hypoparathyroidism
D. hyperparathyroidism
E. hyperfunction of anterior pituitary
13. Desensitization against pollen is accomplished by injecting
A. histamine and histamine analogues
B. antihistaminic drugs
C. pollens or pollen antigens
D. antibodies specific against pollens
E. none of the above
14. Prolonged ischemia to an organ or tissue results in
A. atrophy
B. neoplasia
C. metaplasia
D. hyperplasia

15. A femoral osteosarcoma would MOST likely metastasize to


A. retroperitoneal lymph nodes
B. inguinal lymph nodes
C. femoral lymph nodes
D. the lungs
E. the jaws

16. Infectious hepatitis DIFFERS from serum hepatitis in that serum hepatitis
A. has a shorter incubation period
B. is more highly contagious
C. is transmitted solely by the oral-fecal route
D. is associated with the presence of Australia antigen in the patient's serum
17. The majority of antibodies have a half-life CLOSEST to
A. 20 minutes
B. 1 hour
C. 1 day
D. 1 week
E. 1 month
F. 1 year
18. Rheumatic fever is a disease which
A. can result from a transient bacteremia by viridans streptococci
B. is preceded by a Group A streptococcal infection of the heart
C. develops as a sequela to Group A streptococcal infections
D. affects the glomeruli of the kidney
E. all of the above
19. The Snyder test is used for the
A. diagnosis of mucous patches in the mouth
B. correlation of acidogenic bacteria and caries
C. isolation of Lactobacillus acidophilus
D. detection of dental caries
E. erosion of enamel in deciduous teeth
20. Adenomas are benign tumors arising from
A. cartilage
B. adipose tissue
C. glandular tissue
D. connective tissue
E. covering epithelium
21. Ribonucleic acid is considered to be the chemical basis for heredity in
poliomyelitis virus because
A. it is the only nucleic acid present
B. ultraviolet light induces mutations
C. DNA is infective under appropriate conditions
D. viral infectivity is destroyed by deoxyribonuclease
22. The presence of glossitis, pallor, macrocytic anemia, hyperplasia of the
bone marrow, achlorhydria, atrophy of the gastric mucosa, and focal
degenerative lesions of the dorsal and lateral columns of the spinal cord in a
person older than middle-age suggests the individual has had a deficiency of
vitamin
A. A
B. B1
C. B6
D. B12
E. C

23. The frequency of transient bacteremia has been shown to be HIGHEST in


patients with
A. pulpectomies
B. recurrent aphthae
C. epulis fissuratum
D. root canal therapy
E. multiple extractions

24. Antibiotics are obtained from growth of


A. fungi and virus
B. bacteria and fungi
C. tissue cell culture
D. mycoplasma and rickettsiae
E. bacteria, fungi, and spirochetes
25. A fungal disease which may be confused with tuberculosis because it produces
pulmonary calcifications is

A. amebiasis
B. candidiasis
C. blastomycosis
D. actinomycosis
E. histoplasmosis
KEY (CORRECT ANSWERS)

1. D
2. B
3. D
4. C
5. D

11. C
12. A
13. C
14. A
15. D

6. A
7. C
8. B
9. B
10. E

16. D
17. E
18. C
19. B
20. C
21. A
22. D
23. E
24. B
25. E

TEST 2
DIRECTIONS: Each question or incomplete statement is followed by several
suggested answers or completions. Select the one that BEST answers the
question or completes the statement. PRINT THE LETTER OF THE CORRECT
ANSWER IN THE SPACE AT THE RIGHT.
1. Avirulent Corynebacterium diphtheriae can be converted to the virulent
form by
A. plasmids
B. transfer of F factors
C. specific bacteriophage
D. purified diphtheria toxin
E. DNA from a toxigenic culture
2. Vegetations on the heart valves in acute bacterial endocarditis USUALLY
consist of
A. swelling caused by inflammatory edema
B. scar tissue containing masses of bacteria
C. fused platelets, fibrin, and masses of bacteria
D. areas where healing and calcification have occurred
3. Fragmentation of nuclei in necrotic cells is referred to as
A. pyknosis
B. karyolysis
C. algor mortis
D. karyorrhexis
E. cloudy swelling
4. The mechanism of action of quaternary ammonium compounds
the
A. metachromatic granules
B. cytoplasmic membrane
C. cell wall
D. flagella
E. DNA

is against

5. The reason for heating serum used in the complement fixation test to 56C
for 20-30 minutes is to inactivate
A. antigen
B. antibody

C. complement
D. any bacteria present
6. A viral disease which is stress-influenced, causes local
may also cause central nervous system disease is
A. rabies
B. smallpox
C. yellow fever
D. poliomyelitis
E. herpetic stomatitis

oral lesions, and

7. Fibroma, chondroma, myxoma, and lipoma are all examples of


A. choristomas
B. hamartomas
C. mixed tumors
D. malignant neoplasms
E. connective tissue tumors
8. For a bacterium to be seriously considered in the etiology of dental caries, it
must
A. exist regularly in the dental plaque
B. produce extracellular amylopectins
C. be lethal for germ-free animals
D. produce intracellular dextrans
E. secrete collagenase
9. The radiosensitivity of tissue is MOST closely related to
A. the mitotic rate of tissue
B. the RNA content of the cytoplasm
C. the nuclear cytoplasmic ratio
D. ionization density of the primary irradiation
E. the size of the cells that make up the tissue
10. Hyperplasia is a condition in which there is
A. an increase in the size of a part, organ, or cell
B. a decrease in the size of a part, organ, or cell
C. an increase in the number of individual tissue elements or cells in a given organ
D. a congenital defective formation in which there is incomplete development of an
organ or part
E. the production of one type of adult tissue by cells which normally produce tissue of
another type
11. Shingles is caused by ------------------------------------------A. rubella
B. herpes zoster
C. herpes simplex
D. Newcastle disease
12. The dermotropic viral diseases include
A. smallpox, chicken pox,and measles
B. hepatitis, mumps, and yellow fever

virus.

C. rabies, encephalitis, and encephalomyelitis


D. influenza, common cold, and primary atypical pneumonia
13. Absence of an organ because of failure of the anlage to
referred to as
I
A. aplasia
B. atrophy
C. anaplasia
D. hypoplasia
E. dysplasia

develop is

14. Several mediators of the inflammatory response are known to be activated


or released in the host by antigen- antibody reactions. One such mediator of
inflammation is
A. IgA
B. histamine
C. epinephrine
D. formic acid
E. cardiolipin
15. An in utero complication arising from rubella infection. OFTEN results in
A. jaundice
B. pneumonia
C. paralysis
D. chronic kidney disease
E. congenital malformations
16. The type of necrosis MOST commonly seen in tuberculosis
A. hyaline
B. caseous
C. fibrinous
D. gummatous
E. liquefaction

is

17. An acute diffuse inflammation is synonymous with


A. polyposis
B. cellulitis
C. serous exudate
D. agranulocytosis
E. granulation tissue
18. The dermatomycoses are IMPORTANT fungal diseases because the
A. infections tend to resist most forms of therapy
B. organisms are primarily found in the soil
C. organisms frequently cause epidemics
D. infections are often fatal
E. all of the above
19. Gout is an example of a disturbance in the metabolism of
A. calcium

B. purines
C. cholesterol
D. carbohydrates
20. Rough pneumococci grown in the presence of DNA from smooth
pneumococci develop capsules.
This is known as
A. mutation
B. conjugation
C. translation
D. transduction
E. transformation
21. Research on experimental caries in gnotobiotic animals has indicated
that the majority of oral microorganisms capable of producing smooth surface
dental caries with high carbohydrate diets are species of the genus
A. Veillonella
B. Streptococcus
C. Fusobacterium
D. Lactobacillus
E. Actinomyces
22. Organisms that cause gas gangrene have a metabolism which is
predominantly
A. lipolytic
B. proteolytic
C. saccharolytic
D. psychronophilic
23. Acute osteomyelitis may be caused by many microorganisms, but
statistically the one MOST frequently encountered is
A. gonococcus
B. enterococcus
C. streptococcus
D. staphylococcus
24. Squamous cell carcinoma is a predominant type of malignant neoplasm in
all of the following EXCEPT
A. bronchus
B. esophagus
C. lower lip
D. cervix uteri
E. corpus uteri
25. Oral or pharyngeal lesions are NOT found in
A. dysentery
B. diphtheria
C. candidiasis
D. scarlet fever
E. Ludwig's angina
KEY (CORRECT ANSWERS)

1.C
6.E
11.B
16.B
23. D

2.C
7.E
12.A
17.B
24. E

3.D
8.A
13.A
18.A
25. A

4.B
9.A
19.B

20.E

5.C
10.C
14.B
15.E
21. B

22. B

MICROBIOLOGY / PATHOLOGY
EXAMINATION SECTION
TEST 1
DIRECTIONS: Each question or incomplete statement is followed by several
suggested answers or completions. Select the one that BEST answers the question
or completes the statement. PRINT THE LETTER OF THE CORRECT ANSWER IN THE
SPACE AT THE RIGHT.
1 The early bronchial mucosal alteration most likely to be observed in
cigarette smokers is
A. dysplasia
B. neoplasia
C. metaplasia
D. hypoplasia
E. hyperplasia
2. A 65-year-old male who demonstrates urinary retention or difficulty in
voiding his bladder most likely has
A. carcinoma of the prostate
B. benign prostatic hyperplasia
C. malignant neoplasm of the ureter
D. bladder metastasis of bronchogenic carcinoma
E. polyps
3. Interstitial pulmonary inflammation is MOST characteristic of
A. lobar pneumonia
B. viral pneumonia
C. bronchial asthma
D. bronchopneumonia
E. pleurisy
4. Aerobic organotrophic (or heterotrophic) bacteria which oxidize a
substrate to CO2 and H2O use, in the final electron transport, enzymes
containing
A. cytochromes
B. coenzyme A
C. ribose nucleic acid
D. pyridine nucleotides
E. flavin adenine dinucleotide
5. The etiology of acute diffuse glomerulonephritis seems to be
A. circulatory deficiency associated with prolonged shock

B. bacteremia with localization of organisms in kidney tissue


C. injury of glomeruli by exogenous inorganic toxins
D. degenerative changes induced by sclerotic alterations of blood vessels
E. allergic reaction of glomerular and vascular tissue to beta-hemolytic streptococcal
products

6. Following the initial transient vasoconstriction, the NEXT vascular


reaction to injury in the sequence of events in inflammation is
A. margination of leukocytes
B. dilation of blood vessels
C. increased capillary permeability
D. movement of leukocytes toward the irritant
E. phagocytosis of bacteria and other particles
7 Recurrent aphthae resemble recurrent herpes in that
A. symptoms are similar
B. life-long immunity results
C. vesicles occur with both diseases
D. intranuclear inclusion bodies are present
E. circulating antibodies to the etiologic agents may be demonstrated
8 The BEST evidence for a causal relationship between a nasal carrier of
staphylococci and a staphylococcal infection in a hospital patient is the
demonstration that the organisms from both individuals
A. are coagulase-positive
B. are penicillin resistant
C. are of the same phage type
D. are aureus-type staphylococci
E. produce hemolysis, liquefy gelation and ferment mannitol
9. For the MAJORITY of individuals, the INITIAL infection with herpes
simplex virus results in
A. encephalitis as a young adult
B. a dermal rash in childhood
C. herpes labialis in puberty
D. a subclinical disease
E. genital herpes
10. The radiosensitivity of tissue is MOST closely related to the
A. mitotic rate
B. nuclear cytoplasmic ratio
C. RNA content of the cytoplasm
D. size of the cells that make up the tissue
E. ionization density of the primary irradiation
11. The MOST conspicuous clinical sign of right-sided heart failure is
A. hypertension
B. mitral stenosis
C. pulmonary edema

D. systemic venous congestion


E. brown induration of the lung
12. An example of hypoplasia is the
A. absence of an organ
B. underdevelopment of an organ
C. acquired reduction in size of an organ
D. increase in number of cells of an organ
E. none of the above

13. The renal disease most commonly related to hypertension


A. renal atresia
B. nephrosclerosis
C. acute pyelonephritis
D. chronic pyelonephritis
E. none of the above

is

14. Myxedema is an endocrine disturbance resulting from


A. secondary hyperparathyroidism in children
B. primary hyperparathyroidism in adults
C. hyperpituitarism in adults
D. hypothyroidism in children
E. hypothyroidism in adults
15. Some investigators believe streptococci are predominant in the
etiology of caries because streptococci
A. produce caries in germ-free rats when introduced as monocontaminants
B. are predominant in saliva
C. are beta-hemolytic
D. contain M protein
E. all of the above
16. Detergents kill bacteria by interfering with the function of the cell
A. wall
B. nucleus
C. capsule
D. membrane
E.
cytoplasm
17. Massive necrotizing lesions of the palate in a patient with poorlycontrolled diabetes mellitus are frequently related to
A. phycomycosis
B. blastomycosis
C. histoplasmosis
D. cryptococcosis
E. coccidoidomycosis
18. An example of an endogenous bacterial infection is
A. trachoma
B. Weil's disease
C. leptospirosis
D. salmonellosis
E. actinomycosis
19. MOST forms of lung cancer arise from the
A. peribronchial lymph nodes
B. lining epithelium of the alveoli

C. lining epithelium of the tracheobronchial tree


D. submucosal glands of the tracheobronchial tree
E. pleural sacs
20. Rheumatic fever, scleroderma and rheumatoid arthritis exhibit
------------------- degeneration.
A. fatty
B. amyloid
C. hyaline
D. fibrinoid
E. epithelial

21. A bacterium well known for its large polysaccharide capsule is


A. clostridium tetani
B. staphylococcus aureus
C. hemophilus influenzae
D. streptococcus pneumoniae
E. mycobacterium tuberculosis
22. Recurrent herpes labialis occurs in those people who
A. have had no previous contact with herpesvirus
B. have been infected with herpesvirus but who fail to produce antibodies against the
virus
C. have been infected with herpesvirus and who have antibodies against the virus
D. are hypersensitive to herpesvirus
E. have little resistance
23 Cellular swelling is one of the MOST common changes observed in tissues
obtained at autopsy. Its occurrence
A. proves only that the circulation was deficient
B. is useful only in identifying certain infections
C. usually indicates the action of specific etiologic agents
D. assists in evaluating the nutritional status of the organ involved
E. is of little practical diagnostic importance
24. IgG antibodies have a half-life of approximately
A. 20 minutes
B. 1 hour
C. 1 day
month

D. 1 week

E. 1

25 The MOST reliable finding in the serodiagnosis of an acute infectious


disease is
A. high antibody titer
B. rising antibody titer
C. falling antibody titer
D. positive complement-fixation test
E. positive tuberculin-type skin test
KEY (CORRECT ANSWERS)
1. C
11.D
2. B
12.B
3. B
13.B
4. A
14.E
5. E
15.A

6. B
7. A
8. C
9. D
10. A

16.D
17.A
18.E
19.C
20.D
21. D
22. C
23. E
24. E
25. B
TEST 2

DIRECTIONS: Each question or incomplete statement is followed by several suggested


answers or completions. Select the one that BEST answers the question or completes the
statement. PRINT THE LETTER OF THE CORRECT ANSWER IN THE SPACE AT THE RIGHT.
1. The end-product of glucose metabolism by streptococcus mutans is
A. lactate
B. pyruvate
C. citric acid
D. a combination of carbon dioxide and water
E. none of the above
2. Myocardial infarction results in
A. coagulation necrosis
B. liquefaction necrosis
C. amyloid degeneration
D. Zenker's degeneration
E. mutinous degeneration
3. The cellular infiltrate in a fully-developed delayed hypersensitivity reaction
consists mainly of
A. mast cells and erythrocytes
B. macrophages and lymphocytes
C. macrophages and polymorphonuclear leukocytes
D. plasma cells and polymorphonuclear leukocytes
E. none of the above
4. Active passage of leukocytes through capillary walls is accomplished by
means of
A. desmosome lysis
B. endothelial pores
C. pinocytotic vesicles
D. loosened interendothelial junctions
E. none of the above
5. The chemotactic accumulation at the site of immune complex deposition is a
result of
A. steroids
B. histamine
C. complement
D. antihistamines
E. analgesics
6. The primary value of soap lies in its

A. sporocidal action
B. bactericidal action
C. bacteriostatic action
D. removal of microbes from skin surfaces
E. detergent properties
7. The epithelial change MOST predictive of cancer is
A. acanthosis
B. dysplasia
C. metaplasia
D. parakeratosis
E. hyperkeratosis
8 The MOST reliable postmortem indicator of left ventricular cardiac failure is
A. ascites
B. venous congestion
C. enlargement of the spleen
D. peripheral edema of the ankles
E. chronic passive congestion of the lungs
9 Histoplasmosis is a highly infectious mycotic disease that is characterized
microscopically by
A. intranuclear inclusion bodies
B. flask-shaped ulcers of the ileum
C. intracytoplasmic microorganisms in the R-E system
D. focal abscesses of the liver and the intestinal tract
E. none of the above
10. "Late" proteins synthesized in viral replication include
A. DNA polymerases
B. virus structural proteins
C. proteins that inhibit host cell protein synthesis
D. proteins that cause cessation of host cell RNA synthesis
E. none of the above
11 A thymectomized animal or a person with an inborn deficiency of thymus
usually has
A. no lymph nodes
B. no phagocytic cells
C. a selective deficiency of IgE
D. total absence of circulating antibodies
E. decreased or absent delayed-type hypersensitivity
12. In transduction, DNA is transferred from donor cell to recipient cell by
A. a plasmid
B. an episome
C. a bacteriophage
D. purified nucleic acid
E. none of the above
13. The MOST common cause of hepatomegaly without other distinctive signs
and symptoms is
A. ascites
B. hematoma
C. hepatitis
D. neoplasia
E. fatty change

14. Arguments against indiscriminate use of antibiotics as chemotherapeutic


agents include
I. toxic effects of the antibiotics
II. allergic reactions induced in patients
III. development of drug resistance by an infectious agent
IV. secondary effects experienced due to creation of an imbalance in the normal body
flora
V. alteration of the immune response
The CORRECT answer is:
A. I, II, III
B. I, II, IV, V
C. II, IV, V
D. III, IV, V
E. I, II, III, IV,
V

15. Irreversible pathologic changes include


I. fatty degeneration
II. hydropic degeneration
III. autolysis
IV. coagulative necrosis
The CORRECT answer is:
A. I, III
B. I, III, IV
C. II, III
D. II, IV
E. III, IV
16. Seeding or transplantation metastasis would be likely in carcinomas of the
I. tongue
II. stomach
III. ovary
IV. skin
V. large bowel
The CORRECT answer is:
A. I, II, III
B. I, IV, V C. II, III, IV
D. II, III, V
E. II, IV, V
17. Which of the following therapeutic agents are classed as broad-spectrum
antibiotics?
I. Tetracycline
II. Chloromycetin
III. Dihydrostreptomycin
IV. Penicillin
V. Isoniazid
The CORRECT answer is:
A. I, II
B. I, II, IV, V
C. I, III, V
D. I, V
E. II, III, IV
18. Oversecretion of which of the following hormones causes phosphate
diuresis and results in elevated serum calcium?
A. Thyroxin
B. Cortisone
C. Pituitrin
D. Parathormone
E. Guanine
19. Of the following bone diseases, which is of endocrine etiology?
A. Myeloma
B. Acromegaly
C. Osteopetrosis

D. Paget's disease
E. Monostotic fibrous dysplasia
20. A Ghon focus (tubercle) is a
A. primary lung lesion in the periphery
B. lesion occurring only in the bronchi
C. fibrous lesion in 50 percent of the bronchi
D. secondary lung lesion in over 90 percent of adults
E. none of the above
21. Which of the following antibiotics is effective in treating candidiasis?
A. Nystatin
B. Bacitracin
C. Penicillin
D. Tetracycline
E. Griseofulvin
22. The presence of which of the following factors in viruses makes protective
vaccines a possibility?
A. Lipids
B. Enzymes
C. Protein coat
D. Polysaccharide
E. Methylcytosine
23. Which of the following statements is CORRECT regarding a patient
recovered from hepatitis type B infection?
A. The virus will be excreted in the feces.
B. The patient is a good candidate for blood donation.
C. The patient will have protective immunity to all viral hepatitides.
D. Detection of hepatitis B antigen in serum is indicative of the carrier state.
E. None of the above
24. Infectious mononucleosis is caused by the virus.
A. verruca
B. rubella
C. rubeola
D. Epstein-Barr
E. varicella-zoster
25. Which of the following viruses is suspect in cervical cancer?
A. HVH-1
B. HVH-2
C. EBV
D. C-type particles
E. ADP

KEY (CORRECT ANSWERS)


1. A
2. A
3. B
4. B
5. C

11. E
12. C
13. E
14. E
15. E

6. D
7. B
8. E
9. C
10. B

25. B

21. A
22. C
23. D
24. D

16. D
17. A
18. D
19. B
20. A

MICROBIOLOGY / PATHOLOGY
EXAMINATION SECTION
TEST 1
DIRECTIONS: Each question or incomplete statement is followed by several suggested
answers or completions. Select the one that BEST answers the question or completes the
statement. PRINT THE LETTER OF THE CORRECT ANSWER IN THE SPACE AT THE RIGHT.
1. An increase in the size of an organ caused by an increase in the number of
cells describes
A. atrophy
B. metaplasia
C. hypertrophy
D. hyperplasia
E. regeneration
2. The chemical constituents of bacteria and viruses that are MOST sensitive
to ultraviolet irradiation are
A. lipids
B. proteins
C. carbohydrates
D. nucleic acids
E. inorganic salts
3. In leukemia, prolongation of bleeding time is caused by
A. anemia
B. neutropenia
C. thrombocytopenia
D. the presence of leukemic cells in circulating blood
E. erythroblastosis
4. Increase in the size of the heart in an athlete is an example of
A. anaplasia
B. metaplasia
C. hyperplasia
D. hypertrophy
E. none of the above
5. Denaturation of protein and dissolution of lipid are antiseptic properties
associated with
A. soaps
B. alcohols
C. cresols
D. glutaraldehydes
E. mercuric chloride
6. An antibacterial substance found in saliva, tears and egg white is
A. albumin
B. isozyme
C. amylase
D. lysozyme
E. betalysin

7. Squamous cell carcinoma usually metastasizes by way of


A. the venous system
B. the arterial system
C. the lymphatic system
D. aspiration into the lung
E. none of the above
8. The appearance of HBs antigen in the plasma is associated with
A. Q fever
B. influenza
C. German measles
D. serum hepatitis
E. lupus erythematosus
9. The fluid that leaks out of vessels in noninflammatory conditions, such as
cardiac failure, is
A. exudate
B. effluvium
C. ecchymosis
D. metachysis
E. transudate
10. A benign neoplasm of the myometrium of the uterus is a
A. myeloma
B. fibroma
C. leiomyoma
D. myoblastoma
E. rhabdomyoma
11. The spores of bacillus anthracis are destroyed by
A. refrigerating
(-7 C.
for 48
hours )
B. autoclaving
(121 C.
for 20
minutes)
C. pasteurizing
(61.7 C.
for 30 minutes)
D. immersing in boiling water (100 C.
for 10
minutes)
E. placing in a hot air oven
(121 C.
for 20
minutes)
12. Anoxia MOST seriously affects the brain and the
A. lung
B. heart
C. liver
D. spleen
E. kidney
13. Deposition of calcium in dying or dead tissue is described as
A. calcium soap formation
B. embolic calcification
C. metastatic calcification
D. dystrophic calcification
E. none of the above
14. Viruses may cause disease by
A. lysing many cells of the host
B. transforming cells to malignant cells
C. making vital target cells nonfunctional
D. disrupting the normal defense mechanisms of the host
E. all of the above

15. Epithelial pearls and intercellular bridges observed in an infiltrating


malignancy are diagnostic of

A. adenocarcinoma
B. anaplastic carcinoma
C. squamous cell carcinoma
D. undifferentiated carcinoma
E. transitional cell carcinoma
16. The MOST common organism producing subacute bacterial endocarditis
is
A. staphylococcus
B. alpha-hemolytic streptococcus
C. beta-hemolytic streptococcus
D. gamma-hemolytic streptococcus
E. gonococcus
17. Metachromatic granules are characteristically observed in properly
stained smears of
A. salmonella typhi
B. escherichia coli
C. brucella melitensis
D. bordetella pertussis
E. corynebacterium diphtheriae
18. Resistance of staphylococcus aureus to penicillin is caused by
A. pleomorphism
B. an enzyme that attacks penicillin
C. production of a penicillin analogue
D. a lack of mycolic acid in the cell wall
E. the presence of a tough lipopolysaccharide capsule
19. Transformation is BEST described as
A. DNA coding for RNA synthesis
B. RNA coding for protein synthesis
C. acquisition of an inheritable trait by bacteria mediated by DNA
D. acquisition of an inheritable trait by bacteria mediated by RNA
E. ATP transference
20. Elevation of serum acid phosphatase levels is likely to be seen in patients
with carcinoma of the
A. colon
B. breast
C. cervix
D. uterus
E. prostate gland
21. The MOST common cause of esophageal varices is
A. thrombosis
B. coarctation
C. cor pulmonale
D. portal hypertension
E. ageing changes in the vessels
22. The leading cause of cardiac death in the United States is
A. mitral stenosis
B. congenital anomalies
C. coronary heart disease
D. myocarditis (all causes)
E. hypertensive cardiovascular disease

23. An infection following a serious skin burn that is characterized by the


production of greenish pus and is
generally resistant to antibiotic therapy is probably caused by.
A. proteus vulgaris
B. streptococcus mitis
C. streptococcus mutans
D. staphylococcus aureus
E. pseudomonas aeruginosa
24. When horse serum is injected intravenously into a rabbit, and again into
the skin two or three weeks later, a necrotizing reaction occurs at the site of
the second injection. This is
A. atopy
B. anaphylaxis
C. serum sickness
D. an Arthus reaction
E. a Prausnitz-Kustner reaction
25. The primary result of bacterial carbohydrate metabolism is production of
A. heat
B. energy
C. alcohol
D. acetone
E. fat
KEY (CORRECT ANSWERS)
1.D
2.D
3.C
4.D
5.B

11.B
12.B
13.D
14.E
15.C

6.D
7.C
8.D
9.E
10.C

16.B
17.E
18.B
19.C
20.E

21. D
22. C
23. E
24. D
25. B

TEST 2
DIRECTIONS: Each question or incomplete statement is followed by several
suggested answers or completions. Select the one that BEST answers the
question or completes the statement. PRINT THE LETTER OF THE CORRECT
ANSWER IN THE SPACE AT THE RIGHT.
1. Certain staphylococci, pseudomonas aeruginosa, escherichia coli and
streptococci share as a property the fact that they
A. are gram-positive
B. fail to ferment glucose
C. do not grow on nutrient agar
D. are common in hospital-acquired infections
E. are anaerobic
2. The class of immunoglobulin responsible for atopic allergy is
A. IgA B. IgD C. IgE D. IgG E. IgM
3. Passive congestion occurs when increased amounts of blood in the tissues
collect secondary to
A. hyperplasia
B. hypertension
C. inflammation
D. arterial dilatation
E. venous obstruction
4. A 56-year-old diabetic man had atrial fibrillation two years following a
myocardial infarct. He experienced for right flank pain and hematuria,
paralysis of the right side of the body and sharply demarcated ischemia in the
left foot. These signs and symptoms were most likely due to
A. septicemia
B. lymphangitis
C. venous thrombi
D. arterial emboli
E. venous emboli
5 In skin graft rejection, the MAJOR host response is
A. an Arthus reaction
B. delayed hypersensitivity
C. a Shwartzman reaction
D. passive cutaneous anaphylaxis
E. none of the above
6. Vegetations on the heart valves in acute bacterial endocarditis usually
consist of
A. swellings caused by inflammatory edema
B. scar tissue containing masses of bacteria
C. fused platelets, fibrin and masses of bacteria
D. areas where healing and calcification have occurred
E. none of the above
7. The etiologic agent of actinomycosis in man is
A. exogenous and is found in infected meat
B. exogenous and requires contact with a vector
C. exogenous and requires inhalation of spores
D. endogenous and is found in healthy mouths

E. endogenous and is carried in the lower gastro intestinal tract


8. Actinomycosis is characterized by a --------------------------------- exudate.
A. serous
B. purulent
C. hemorrhagic
D. pseudomembranous
E. fibrinous, nonpurulent
9. A lysogenic bacterium is one that
A. lyses red cells
B. produces properdin
C. harbors a temperate bacteriophage
D. produces lecithinase when incubated anaerobically
E. produces spheroplasts when incubated anaerobically
10. A complete blood cell count in a patient with severe emphysema is most
likely to show
A. anemia
B. leukopenia
C. polycythemia
D. leukocytosis
E. macrocythemia
11. The pulmonary neoplasm to which the endocrine effect of
hyperparathyroidism is attributed is
A. adenocarcinoma
B. oat cell carcinoma
C. pheochromocytoma
D. medullary carcinoma
E. squamous cell carcinoma
12. Of the following, the MOST common site of a basal cell carcinoma is the
A. tongue
B. gingiva
C. lower lip
D. upper face
E. oral mucosa
13. Immune serums showing high agglutination titers often fail to agglutinate
homologous bacteria in low dilution. This is designated as
A. the prozone
B. the opsonic index
C. an Arthus reaction
D. agglutination adsorption
E. the Bordet-Gengou phenomenon
14. As opposed to lobar pneumonia, bronchopneumonia is characterized
grossly and microscopically by
A. inflammation of a bronchus
B. organization of alveolar exudate
C. a patchy, inflammatory distribution
D. a diffuse, inflammatory distribution
E. pleural inflammation
15 The MOST common malignancy found in bones of the human skeleton is
A. osteosarcoma
B. chondrosarcoma
C. multiple myeloma

D. giant cell tumor


E. metastatic carcinoma
16 The MOST common cause of acute purulent meningitis between ages 3
months and 2 years is
A. myxovirus
B. hemophilus influenzae
C. mycoplasma pneumoniae
D. streptococcus pneumoniae
E. smallpox
17 The two genera MOST frequently associated with bacterial cystitis are
A. salmonella and shigella
B. brucella and hemophilus
C. pseudomonas and escherichia
D. escherichia and proteus
E. proteus and pseudomonas
18 The active mechanism directly responsible for damage to blood vessels in
an immune complex disorder is the
A. activation of the complement system
B. formation of small, soluble, immune complexes
C. formation of large, insoluble, immune complexes
D. phagocytosis of immune complexes by the RE system
E. release of histamine from mast cells causing a vasomotor response
19. Vitamin K is necessary for
A. ossification of osteoid
B. formation of fibrinogen (Factor I)
C. formation of prothrombin (Factor II)
D. maturation of megakaryocytes to platelets
E. maturation of collagen from tropocollagen
20. MOST viral antigens of diagnostic importance are
A. lipids
B. proteins
C. nucleic acids
D. polysaccharides
E. lipopolysaccharides
21. The MOST common cause for megaloblastic anemia is
A. gastric bleeding
B. lack of dietary iron
C. lack of dietary folic acid
D. lack of dietary pyridoxine
E. lack of absorption of vitamin B12
22. Vitamin A functions to
A. prevent pellagra
B. promote absorption of calcium
C. promote differentiation of epithelial cells
D. maintain the integrity of connective tissues

E. maintain skin hue


23. Glutaraldehyde, betapropiolactone, formaldehyde and ethylene oxide
share the common characteristic of being
A. unstable
B. explosive
C. cidal agents
D. static agents
E. carcinogens
24. The normal form of bacterial gene transfer that is LEAST susceptible to
DNAase and does NOT require cell-to-cell contact is
A. transition
B. conjugation
C. transduction
D. transformation
E. incubation
25. Which of the following is NOT characteristic of plasmids
A. Confer conjugal fertility
B. Carry genetic information
C. Exist as circular RNA molecules
D. Exist as extrachromosomal elements in bacteria
E. None of the above
KEY (CORRECT ANSWERS)
1. D
2. C
3. E
4. D
5. B

11.B
12.D
13.A
14.C
15.E

6. C
7. D
8. B
9. C
10. C

16.B
17.D
18.D
19.C
20.B

21.
22.
23.
24.
25.

C
C
C
C
C

MICROBIOLOGY / PATHOLOGY

EXAMINATION SECTION
TEST 1
DIRECTIONS: Each question or incomplete statement is followed by several
suggested answers or completions. Select the one that BEST answers the
question or completes the statement. PRINT THE LETTER OF THE CORRECT
ANSWER IN THE SPACE AT THE RIGHT.
1. Competitive inhibition BEST describes the desired effect in
A. opsonization
B. phagocytosis
C. tuberculin reaction
D. allergic desensitization
E. first-set graft rejection
2. Cytologic and morphologic characteristics of epithelial dysplasia include
I. normal architectural arrangement
II. hyperchromatic nuclei
III. mitoses near the surface layer
IV. pleomorphism of cells
V. invasion of underlying connective tissue
The CORRECT answer is:
A. I, II, III
B. I, IV, V
C. II, III, IV
D. II, IV
E. II, V
3. Causes of atrophy include
I. disuse
II. pressure
III. loss of innervation
IV. chemical stimulation
V. decreased nourishment
VI. overstimulation with hormones
The CORRECT answer is:
A. I, II, III, IV
B. I, II, III, V
C. I, III, V, VI
D. II, IV, V, VI
4. Endospores are formed by the genera
I. actinomyces
II. bacillus
III. clostridium
IV. corynebacterium
V. micrococcus
The CORRECT answer is:
A. I, II
B. I, V
C. II, III

5. Granulation tissue typically contains


I. fibroblasts
II. nerve fibers

E. III, IV, V, VI

D. III, IV

E. IV, V

III. endothelial cells


IV. epithelioid cells
V. giant cells
The CORRECT answer is:
A. I, II
B. I, III
C. II, IV
D. III, V
E. IV, V
6. Pulmonary atelectasis may be secondary to
I. fibrous pleural adhesions
II. blockage of a bronchus by an aspirated foreign body
III. a calcified nodule in the apex of the lung
IV. blockage of a bronchus by mucus
V. hydrothorax
VI. pneumothorax
The CORRECT answer is:
A. I, II, IV
B. I, II, IV, V
C. II, III, IV, V

D. II, III

E. III, V, VI

7. Malignant bone tumors likely to be encountered in children or young adults


include
I. myeloma
II. metastatic carcinoma
III. Ewing's sarcoma
IV. osteogenic sarcoma
V. liposarcoma
The CORRECT answer is:
A. I, II, III
B. I, III
C. II, III, V
D. II, IV
E. III, IV
8. Thrombotic occlusion of a coronary artery may result in
I. infarction of the myocardium
II. proliferation of myocardial fibers
III. fibrosis of the myocardium
IV. stenosis of the mitral valve
V. no changes in the myocardium
The CORRECT answer is:
A. I, II, III
B. I, III, IV C. I, III, V
D. II, IV, V
E. I, II, III, IV, V
9. Features of heptatis A viral infections include
I. high incidence of morbidity and less than 1 percent mortality
II. insidious onset and a long incubation period
III. acquisition generally by non-parenteral routes
IV. transmission only through blood or blood products
The CORRECT answer is:
A. I, II
B. I, III
C. I, IV
D. II, III

10. Thymic hypoplasia may result in


I. hypergammaglobulinemia
II. a decreased propensity to develop cancer
III. an increased predisposition to fungal and viral infections

E. II, IV

IV. an absence of plasma cells


The CORRECT answer is:
A. I, II
B. I, III
C. II, III

D. II, IV

11. Long bones may be affected by


I. rickets
II. osteopetrosis
III. chondrodystrophy
IV. fibrous dysplasia
V. osteogenesis imperfecta
The CORRECT answer is:
A. I, II, V
B. II, III
C. II, III, IV, V
V

E. III, IV

D. III, IV, V

E. I, II, III, IV,

12. Which of the following cells are associated with liberation of histamine?
I. Lymphocytes
II. Eosinophils
III. Plasma cells
IV. Mast cells
The CORRECT answer is:
A. I, II
B. I, III
C. I, IV
D. II, III
E. II, IV
13. Which of the following substances are considered to be chemical mediators
for vascular dilatation at the initial phase of acute inflammation?
I. Serotonin
II. Histamine
III. Bradykinin
IV. Hyaluronidase
The CORRECT answer is:
A. I, II, III
B. I, II, IV
C. I, III, IV
D. II, III, IV
E. I, II, III, IV
14. Which of the following are phagocytes?
I. Plasma cells
II. Lymphocytes
III. Kupffer's cells of liver sinusoids
IV. Polymorphonuclear leukocytes
V. Eosinophils
The CORRECT answer is:
A. I, II, III
B. I, III, V
C. II, III, V
D. II, IV, V

E. III, IV, V

15. Cirrhosis of the liver can be a sequela of


A. alcoholism
B. biliary obstruction
C. infectious hepatitis
D. congestive heart failure
E. all of the above
16. Which of the following characteristics is common to rickettsiae and
viruses?
A. Structure
B. Cell wall synthesis

C. Requirement of living cells for growth


D. Susceptibility to broad-spectrum antibiotics
E. Caused by vitamin deficiency
17. Which of the following characteristically produces a latent infection?
A. ECHO virus
B. Measles virus
C. Smallpox virus
D. Herpes simplex
E. Coxsackie B virus
18. Which of the following is an example of naturally-acquired passive
immunity in humans?
A. Smallpox immunization
B. Injection of gamma globulin
C. Transfusion of immune blood
D. Placental transfer of antibody
E. Antibody produced during recovery from measles
19. Which of the following cell types is associated with antibody formation?
A. Basophil
B. Monocyte
C. Eosinophil
D. Neutrophil
E. Plasma cell
20. Which of the following diseases, in which gastroenteritis is the principal
feature, has an incubation period of 2-4 hours?
A. Cholera
B. Botulism
C. Salmonellosis
D. Summer diarrhea
E. Staphylococcal food poisoning
21. In which of the following mycotic infections are the organisms
characteristically found microscopically within reticuloendothelial cells?
A. Candidiasis
B. Aspergillosis
C. Blastomycosis
D. Histoplasmosis
E. Sclerosis
1

22. Which of the following conditions is suggested in a patient with nausea


and vomiting, pigmentation of the oral mucous membrane and skin,
hypotension and asthenia?
A. Myxedema
B. Pineal tumor
C. Graves' disease

D. Addison's disease
E. Peutz-Jeghers syndrome
23. Which of the following has been shown to have a relationship to carcinoma
of the cervix?
A. Papovavirus
B. Varicella-zoster virus
C. Herpesvirus hominis type I
D. Herpesvirus hominis type II
E. None of the above
24. A patient with which of the following diseases is predisposed to develop
osteosarcoma?
A. Osteomalacia
B. Osteopetrosis
C. Paget's disease of bone
D. Osteogenesis imperfecta
E. Osteomyelitis
25. Organisms belonging to which of the following genera can tolerate the
lowest pH?
A. Veillonella
B. Bacteroides
C. Streptococcus
D. Lactobacillus
E. Staphylococcus
KEY (CORRECT ANSWERS)
1. D
2. C
3. B
4. C
5. B

11. E
12. E
13. A
14. E
15. E

6. D
7. E
8. C
9. B
10. E

16. C
17. D
18. D
19. E
20. E
21. D
22. D
23. D
24. C
25. D

TEST 2
DIRECTIONS: Each question or incomplete statement is followed by several
suggested answers or completions. Select the one that BEST answers the

question or completes the statement. PRINT THE LETTER OF THE CORRECT


ANSWER IN THE SPACE AT THE RIGHT.
1. In which of the following organs or tissues are ascending infections
common?
A. Bone
B. Kidney
C. Heart
D. Peritoneum
E. Subcutaneous connective tissue
2 Which of the following viruses has been found to have an etiologic
relationship with infectious mononucleosis?
A. Rhinovirus
B. Coxsackie virus
C. Epstein-Barr virus
D. Newcastle disease virus
E. Hodgkins disease virus
3 Which of the following conditions is most likely to be associated with a
heterophile agglutination titer greater than 1:128?
A. Syphilis
B. Tuberculosis
C. Rheumatic fever
D. Hodgkin's disease
E. Infectious mononucleosis
4. When streptococci are growing on glucose as a carbon and energy source in
an unbuffered medium, which of the following is likely to occur in the culture
medium?
A. An elevated pH
B. A lowered pH
C. Production of a gas
D. Production of maltose
E. None of the above
5. Which of the following lesions is MOST common in infants?
A. Chondrosarcoma
B. Neuroblastoma
C. Malignant melanoma
D. Basal cell carcinoma
E. Squamous cell carcinoma
6. To which of the following sites does carcinoma of the prostate metastasize
with the greatest frequency?
A. Lung
B. Liver
C. Brain
D. Spleen
E. Skeleton
7. During which phase of bacterial growth does ampicillin exert its greatest
cidal activity?
A. Early lag phase

B. Lag phase
C. Log phase
D. Stationary phase
E. Phase of decline
8. Which of the following has the LEAST capacity for regeneration?
A. Bone
B. Cardiac muscle
C. Liver parenchyma
D. Squamous epithelium
E. Fibrous connective tissue
9. Which of the following conditions is LEAST likely to have clinical signs and
symptoms?
A. Pyemia
B. Toxemia
C. Septicemia
D. Bacteremia
E. None of the above
10. Which of the following enteric microorganisms is NOT typically found in
the normal human intestinal flora?
A. Escherichia coli
B. Salmonella typhi
C. Proteus vulgaris
D. Bacteroides fragilis
E. Pseudomonas aeruginosa
11. Which of the following would NOT be common to lesions of both sarcoid
and tuberculosis?
A. Giant cells
B. Lymphocytes
C. Epithelioid cells
D. Caseation necrosis
E. None of the above
12. Which of the following conditions is NOT mediated by antibody?
A. Hay fever
B. Anaphylaxis
C. Serum sickness
D. Contact dermatitis
E. Autoimmune hemolytic disease
13. Which of the following is NOT characteristic of plasmids?
A. Confer conjugal fertility
B. Carry genetic information
C. Exist as circular RNA molecules
D. Exist as extrachromosomal elements in bacteria
E. None of the above
14. Of the following, amphotericin B therapy is NOT indicated for
A. nocardiosis
B. histoplasmosis
C. cryptococcosis
D. coccidioidomycosis
E. systemic candidiasis
15. Histiocytosis X encompasses all of the following clinical entities EXCEPT

A. Gaucher's disease
B. Letterer-Siwe disease
C. eosinophilic granuloma
D. Hand-Schuller-Christian disease
E. Guillon-Beret disease
16. Jaundice is seen in each of the following EXCEPT
A. aplastic anemia
B. liver cell damage
C. excessive hemolysis
D. bile duct obstruction
E. carcinoma of the head of the pancreas
17. Each of the following is useful in the identification of viruses EXCEPT
A. nature of the viral nucleic acid
B. morphology of the viral protein coat
C. ability of the virus to grow on complex media
D. neutralization of the virus with specific antiserums
E. ability of the virus to be inactivated by certain solvents, such as ether or chloroform
18. Which of the following conditions is NOT mediated by antibody?
A. Hay fever
B. Anaphylaxis
C. Serum sickness
D. Contact dermatitis
E. Autoimmune hemolytic disease
19. Which of the following statements if NOT characteristic of chronic
myelogenous leukemia?
A. It is most common in adults.
B. It is characterized by massive splenomegaly.
C. It is the only leukemia with a specific chromosomal marker.
D. Microscopically, the cells resemble nearly normal granulocytes.
E. Because of the great numbers of WBC's, infections are uncommon.
20 Which of the following lesions is LEAST likely to produce acute abdominal
symptoms?
A. Cholelithiasis
B. Acute pancreatitis
C. Carcinoma of the ampulla of Vater
D. Carcinoma of the tail of the pancreas
E. Appendicitis
21. Which of the following has the LEAST relevance to the prognosis of a
patient with malignant melanoma?
A. Depth of invasion
B. Multiple biopsies
C. Sex of the patient
D. Degree of pigmentation
E. Palpable lymphadenopathy

22. Hypothyroidism in children results in


A. myxedema
B. gigantism

C. cretinism
D. acromegaly
E. diabetes insipidus
23. Severe chronic bronchitis is characterized by any of the following EXCEPT
A. chronic cough
B. purulent sputum
C. dyspnea on exertion
D. decreased airway resistance
E. ventilation-perfusion imbalance
24. Autoimmune disorders are thought to arise by any of the following
mechanisms EXCEPT
A. release of sequestered antigen
B. persistent depression of the immune system
C. cross-reaction between exogenous and self-antigens
D. loss of T-suppressor cell activity against autoreactive (forbidden) clones
E. none of the above
25. Metastasis to bone is frequent with malignant tumors of each of the
following tissues EXCEPT the
A. lung
B. breast
C. kidney
D. brain
E. prostate
KEY (CORRECT ANSWERS)
1.
2.
3.
4.
5.

B
C
E
B
B

11. D
12. D
13. C
14. A
15. A

6.
7.
8.
9.
10.

E
C
B
D
B

16. A
17. C
18. D
19. E
20. D
21. D
22. C
23. D
24. B
25. D

ANATOMIC SCIENCES
EXAMINATION SECTION

TEST 1
DIRECTIONS: Each question or incomplete statement is followed by several suggested
answers or completions. Select the one that BEST answers the question or completes the
statement. PRINT THE LETTER OF THE CORRECT ANSWER IN THE SPACE AT THE RIGHT.
1. The jugular foramen transmits which cranial nerves?
A. VII, IX and XII
B. IX, X and XI
C. IX, XI and XII
D. X, XI and XII
2. The thyroid gland is characterized by the fact that it
A. becomes enlarged when it is overactive
B. becomes smaller after the parathyroid glands are removed
C. functions as the controller of general body metabolism
D. causes a decrease in body weight when the person has simple goiter
3. The posterior lobe of the hypophysis (pituitary gland develops from)
A. diencephalon
B. mesenchyme
C. choroid plexus
D. rhombencephalon
4. A feature which characterizes a vein, compared with its companion artery, is
that the vein has a(n)
A. smaller caliber
B. indistinct endothelial lining
C. more prominent tunica medica
D. more prominent tunica adventitia
5. The common hepatic artery is a branch of the artery.
A. celiac
B. gastric
C. splenic
D. superior mesenteric
6. The thoracic duct extends from the upper part of the abdomen to the base
of the neck where it ends in the
A. subclavian artery
B. superior vena cava
C. junction of external and internal jugular veins
D. junction of internal jugular and subclavian veins
7. The structure that passes through rather than posterior to the diaphragm
is the
A. aorta
B. esophagus
C. azygos vein
D. thoracic duct

8. The muscle whose tendon loops around the pterygoid hamulus is the
A. tensor tympani

B. tensor veli palatini


C. levator veli palatini
D. superior pharyngeal constrictor
9. A sinusoidal arrangement of blood vessels is found in the
I. hypophysis (pituitary)
II. spleen
III. kidney
IV. liver
The CORRECT answer is:
A. I, II
B. I, II, IV
C. I, III
D. II, III, IV
10. In the human fetus, developing blood cells are found in
A. liver, pancreas and lymph nodes
B. red bone marrow, liver, spleen and lymph nodes
C. red bone marrow, liver, spleen and lungs
D. submucosa of the small intestine and lymph nodes
11. Intervals between the bones in the midline of the cranial base of a
newborn child consist of
A. hyaline cartilage
B. elastic cartilage
C. fibrous cartilage
D. fibrous connective tissue
12. The papillary layer of the dermis differs from the reticular layer in that the
papillary layer
A. contains larger blood vessels than the reticular layer
B. has a few elastic fibers, whereas the reticular layer has no elastic fibers
C. is composed of coarse collagenous fibers, whereas the reticular layer is composed of
predominately fine
collagenous fibers
D. is more finely constructed, whereas the reticular layer contains coarser collagenous
fibers as a
component
13. The sternal angle is used in locating precisely the
A. clavicle
B. first rib
C. second rib
D. xiphoid process

14. The preganglionic cell bodies of the parasympathetic portion of the


autonomic nervous system are situated in
A. the anterolateral horn of the grey substance of the spinal cord in the twelve thoracic
and upper three
lumbar cord segments
B. the three cervical ganglia and second, third and fourth sacral ganglia of the
sympathetic trunk
C. association with the nuclei of certain cranial nerves and in the anterolateral cell
column of the grey
substance of second, third and fourth sacral cord segments
D. none of the above

15. The lesser omentum is a


A. peritoneal fold connecting the lesser curvature of the stomach and the first part of the
duodenum to
the liver
B. part of the peritoneal cavity separated from the greater sac
C. layer of retroperitoneal fat around the kidneys
D. mesentery connecting the liver to the anterior abdominal wall
16. The uterine cavity, excluding the cervical canal, is
A. cylindrical
B. wider at its junctions with the cervical canal
C. wider anteroposteriorly than from right to left
D. roughly triangular in shape and compressed anteroposteriorly
17. Cytoplasmic ribonucleic acid is localized in
A. granular endoplasmic reticulum
B. the Golgi apparatus
C. mitochondria
D. lysosomes
18. MOST of the antibodies in the body which act against bacterial antigens
are produced by
A. T-lymphocytes
B. B-lymphocytes
C. macrophages
D. plasma cells
19. Embryologically, the spinal autonomic ganglia are derived from
A. the neural crest
B. the neural tube
C. cranial placodes
D. all of the above
20. The auriculotemporal nerve carries some fibers which are
A. secretory to the parotid gland
B. motor to the masseter muscle
C. afferent from the carotid body
D. sensory to the lining of the tympanic cavity
21. The cells of the sulcular epithelium are joined by
A. desmosomes
B. tonofibrils
C. terminal webs
D. hemidesmosomes
22. The cellular organelles which may be characterized as being numerous
during mitosis and serving as the cytoskeleton when present in intermitotic
cells are
A. the Golgi apparatus
B. microtubules

C. mitochondria
D. nucleoli
23. The zona reticularis is the inner layer of the
A. cortex of the thymus
B. medulla of the thymus
C. cortex of the suprarenal (adrenal)
D. medulla of the suprarenal (adrenal)
24. The folding of the embryo during the fourth week is the result of the
A. prominent growth of the neural tissues
B. development of the umbilical arteries
C. development of the branchial arches
D. twisting of the developing heart
25. The alveolar bone proper (cribriform plate) usually consists of
A. woven bone
B. bundle bone only
C. lamellar bone only
D. bundle bone and lamellar bone
KEY (CORRECT ANSWERS)
1. B
11. A
2. C
12. D
3. A
13. C
4. D
14. C
5. A
15. A
6. D
7. B
8. B
9. B
10. B

21. A
22. B
23. C
24. A
25. D

16. D
17. A
18. D
19. A
20. A

TEST 2
DIRECTIONS: Each question or incomplete statement is followed by several
suggested answers or completions. Select the one that BEST answers the
question or completes the statement. PRINT THE LETTER OF THE CORRECT
ANSWER IN THE SPACE AT THE RIGHT.
1. The fate of Meckel's cartilage is believed to be
A. dissolution without contribution to any definitive structure
B. dissolution with minor contribution to ossification
C. an active role in ossification of the mandibular body
D. an active role in ossification of the alveolar process
2 Lips are derived EMBRYOLOGICALLY from processes.
A. maxillary and mandibular
B. maxillary, mandibular and medial nasal
C. maxillary, mandibular and lateral nasal
D. maxillary, mandibular, lateral nasal and medial nasal
3. True denticles are characterized by
A. having irregular shapes
B. containing dentinal tubules
C. being located in the coronal pulp
D. being larger than false pulp stones
4. Primary cementum possesses
A. lacunae
B. lamellae
C. canaliculi
D. cementocytes
5. The degree of mineralization of normal cementum is approximately that of
A. bone
B. dentin
C. enamel
D. cartilage
6. Maturation of enamel is characterized by a percentage increase in inorganic
content and a percentage
A. increase in
water
B. decrease in
water and
organic content
C. decrease in
water and
increase in organic content
D. increase in
water and decrease in organic content
7. The articulating surfaces of the temporomandibular joint are covered by
A. hyaline cartilage
B. elastic cartilage
C. vascular fibrous tissue
D. fibrous or fibrocartilaginous tissue

8. The nerve to the masseter passes


A. through the mandibular notch to enter the muscle on its medial surface
B. superior to the zygomatic arch to enter the muscle on its lateral surface
C. posterior to the temporomandibular joint
D. deep to the medial pterygoid muscle
9. Reversal lines which may be seen on the cribriform plate (alveolar bone
proper) of the alveolar process indicate the cessation of ----------------------activity.
A. osteoblastic
B. osteoclastic
C. myeloid
D. healing
10. The majority of the muscles of the larynx receive their innervation from
the
A. cervical plexus
B. laryngeal plexus
C. superior laryngeal nerve
D. inferior laryngeal nerve
11. In the post-eruptive phase, a tooth undergoes primarily
A. movements in an occlusal direction
B. movements in an apical direction
C. movements in a distal direction
D. rotational movement
12. The lingual artery passes
A. deep to the hypoglossus muscle
B. superficial to the mylohyoid muscle
C. superficial to the digastric muscle
D. between digastric and mylohyoid muscles
13. The cell component that is genetically continuous from one cell generation
to the next is the
A. nuclear membrane
B. Golgi complex
C. central bodies
D. chromatin
14. Prickle cells are found in
A. skin dermis
B. sweat glands
C. peripheral nerve tissue
D. stratified squamous epithelium
15. One important collateral circulation of the hepatic portal system is via the
veins.
A. renal
B. mesenteric
C. esophageal

D. external iliac
16. A section of the posterior lobe of the hypophysis (pituitary) contains

mainly
A. follicles
B. alpha and beta cells
C. chromophobes and chromophils
D. unmyelinated nerve fibers
17. The alternate loosening and tightening of a primary tooth that is about to
be shed may result from
A. lack of developing permanent successors
B. alternate resorption and apposition of cementum and bone
C. alternate softening and hardening of the alveolar bone proper
D. movement in a lingual direction of the developing permanent tooth
18. Those papillae of the tongue,which are the largest, the LEAST numerous,
have many taste buds, and are associated with the ducts of von Ebner's glands
are
A. foliate
B. filiform
C. fungiform
D. circumvallate
19. Incremental lines of Retzius appear
A. as lines in the enamel running at right angles to the enamel surface
B. as lines in the dentin running at right angles to the dentinoenamel junction
C. in enamel and follow the appositional pattern
D. in dentin and follow the appositional pattern
20. The pillars of the fauces are formed by mucosa and the underlying
palatoglossus and
A. styloglossus muscles
B. stylopharyngeus muscles
C. palatopharyngeus muscles
D. none of the above
KEY (CORRECT ANSWERS)
1. B
2. B
3. B
4. B
5. A

11. A
12. A
13. D
14. D
15. C

6. B
7. D
8. A
9. B
10. D

16. D
17. B
18. D
19. C
20. C

ANATOMIC SCIENCES
EXAMINATION SECTION
TEST 1
DIRECTIONS: Each question or incomplete statement is followed by several
suggested answers or completions. Select the one that BEST answers the
question or completes the statement. PRINT THE LETTER OF THE CORRECT
ANSWER IN THE SPACE AT THE RIGHT.
1. A section of the posterior lobe of the hypophysis contains MAINLY
A. follicles
B. alpha and beta cells
C. chromophobes and chromophils
D. unmyelinated nerve fibers
E. all of the above
2. The artery MOST commonly involved in stroke is the
A. lenticulostriate
B. anterior cerebral
C. posterior cerebral
D. middle meningeal
E. any of the above
3 Which of the following structures may be found within the
mediastinum?
A. Heart
B. Phrenic nerve
C. Thoracic duct
D. Arch of the aorta
E. Recurrent laryngeal nerve

posterior

4. In skeletal muscle, a triad consists of


A. a T tubule and associated mitochondria
B. a T tubule and invagination of the sarcolemma
C. a zone of attachment between adjacent cells
D. terminal cisternae and sarcoplasmic reticulum
E. terminal cisternae and a fingerlike invagination of the sarcolemma
5. The cytoplasm of osteoblasts stains intensely with basic
caused by the
A. high ribonucleic acid content
B. low content of ribonucleic acid
C. high content of potassium ions
D. increased number of mitochondria

dyes. This is

E. presence of acidophilic substances

6. In the Pain Gate Theory, a controller system modulates sensory input so


that there is a selective and integrative action occurring before impulses reach
the first synapse for onward transmission. The gate controller in this system is
the
A. T-cell
B. large A fiber
C. small C fiber
D. central control
E. substantia gelatinosa
7. Colloid in the usual thyroid follicle stains
A. basophilic
B. acidophilic
C. positive for DNA
D. positive for steroids
E. none of the above
8. After exertion, an athlete is puffing. Which of the following paired muscles
helps maintain a
wide airway through the larynx?
A. Cricothyroid
B. Aryepiglottic
C. Thyroarytenoid
D. Posterior cricoarytenoid
E. None of the above
9. The arrow in the sketch at the right points to the
PICTURE
A. abducens nucleus
B. oculomotor nucleus
C. motor nucleus of the trigeminal nerve
D. motor nucleus of the hypoglossal nerve
E. none of the above
10. Which of the following layers is TOTALLY
lacking in thin skin? Stratum
A. basale
B. lucidum
C. spinosum
D. corneum
E. granulosum
11. The normoblast is a developmental stage of the
A. monocyte
B. lymphocyte
C. eosinophil
D. erythrocyte
E. none of the above

12. When a tendon or a ligament is attached to bone, the attaching fibers


are--------------------- fibers.
A. Tomes'
B. principal
C. haversian
D. Sharpey's
E. interstitial
13. The middle cerebral artery is distributed on the
A. ventral surface of the brain stem
B. ventral surface of the cerebral hemisphere
C. lateral surface of the cerebral hemisphere
D. anterior region of the medial surface of the cerebral hemisphere
E. posterior region of the medial surface of the cerebral hemisphere
14. The medial (internal) pterygoid muscle is attached to the
A. pterygoid hamulus
B. lateral surface of the lateral pterygoid plate
C. medial surface of the lateral pterygoid plate
D. medial surface of the medial pterygoid plate
E. lateral surface of the greater wing of the sphenoid
15. Cyclic DNA has been found in or associated with which of the following
organelles?
A. Centriole
B. Mitochondrion
C. Golgi apparatus
D. Rough endoplasmic reticulum
E. Smooth endoplasmic reticulum
16. The upper five or six anterior intercostal arteries are
A. thoracic aorta
B. musculophrenic arteries
C. pericardiacophrenic arteries
D. internal thoracic arteries
E. superior epigastric arteries

branches of the

17. Which of the following cells contains a large amount of


endoplasmic reticulum?
A. Lymphocyte
B. Eosinophil
C. Plasma cell
D. Skeletal muscle cell
E. Reticuloendothelial cell

rough-surfaced

18. The greater peritoneal sac communicates with the lesser


by means of the
A. aortic hiatus
B. inguinal canal
C. epiploic foramen
D. lesser pelvic aperture
E. superior pelvic aperture

peritoneal sac

19. Right subclavian and right common carotid arteries arise from the
A. thoracic aorta
B. ascending aorta
C. arch of the aorta
D. pulmonary artery
E. brachiocephalic artery
20. Schwann's cells arise from
A. glial cells
B. spongioblasts
C. neural crest cells
D. neuroepithelial cells of the ependymal zone
E. all of the above
21. Which of the following describes the manner in which tissue fluid reaches
the epithelium of the skin?
A. Arterioles are directly involved in tissue fluid exchange with the epithelium.
B. Capillaries are directly involved in tissue fluid exchange with the epithelium.
C. Tissue fluid is obtained through the ground substance of connective tissue from
arterioles.
D. Tissue fluid is obtained through the ground substance of connective tissue from
capillaries.
E. None of the above
22. In the condition of hypersecretion of growth hormone, which of the
following determines whether gigantism or acromegaly results?
A. The genetic make-up of the individual
B. The nutritional state of the individual
C. Whether epiphyses of the long bones are fused
D. Whether there is a concomitant excess of thyroxine
E. All of the above
23. Which of the following muscles serves as the prime extensor of the
forearm?
A. Biceps brachii
B. Triceps brachii
C. Brachioradialis
D. Latissimus dorsi
E. Extensor digitorum communis
24. The circulatory system of a near-term fetus differs from that of an adult in
that, in the fetus, the
A. ductus venosus carries unoxygenated blood
B. ductus arteriosus carries fully oxygenated blood
C. foramen ovale allows blood to pass primarily from the right atrium to the left atrium
D. foramen ovale does not allow blood to pass from the right atrium to the left atrium
E. none of the above
25. During deep inspiration, the diaphragm behaves in which of the following
ways? It
A. relaxes

B. moves inferiorly
C. moves superiorly
D. does not change position
E. compresses thoracic viscera
KEY (CORRECT ANSWERS)
1. D
2. A
3. C
4. E
5. A

6. E
7. B
8. D
9. D
10. B

11. D
12. D
13. C
14. C
15. B-

16. D
17. C
18. C
19. E
20. C

21.D
22.C
23.B
24.C
25.B

TEST 2
DIRECTIONS: Each question or incomplete statement is followed by several suggested
answers or completions. Select the one that BEST answers the question or completes the
statement. PRINT THE LETTER OF THE CORRECT ANSWER IN THE SPACE AT THE RIGHT.
1. Peripherally located nuclei are found in which of the following types of adult
muscle cells?
A. Smooth only
B. Cardiac only
C. Skeletal only
D. Smooth and cardiac
E. Cardiac and skeletal
2. The spheno-occipital synchondrosis in the midline of the cranial base of a
newborn consists of
A. hyaline cartilage
B. elastic cartilage
C. fibrous cartilage
D. fibrous connective tissue
E. none of the above
3. The papillary layer of the dermis differs from the reticular layer in that the
papillary layer
A. contains larger blood vessels than the reticular layer
B. has a few elastic fibers, whereas the reticular layer has no elastic fibers
C. is composed of coarse collagenous fibers, whereas the reticular layer is composed of
predominately fine
collagenous fibers
D. is more finely constructed, whereas the reticular layer contains coarser collagenous
fibers as a component
E. none of the above
4. Long bones of the skeleton increase in length because of
A. mitotic division of osteocytes
B. mitotic division of osteoblasts
C. resorption of primary bone by osteoclasts
D. appositional growth on the cartilaginous epiphyseal plate

E. interstitial growth in the cartilaginous epiphyseal plate

5. The facial process indicated with the letter A in the sketch at the right of
the head and neck region
of a human embryo gives rise to the

A. nasal septum
B. nasal conchae
C. primary palate
D. secondary palate
E. none of the above
6. Polymerization of amino acids to form procollagen filaments occurs
A. on ribosomes
B. in the golgi apparatus
C. on the fibroblast surface
D. in rough, endoplasmic reticulum
E. after cleavage of the registration peptide
7. The cell body of a somatic afferent neuron is found in the
A. dorsal horn of the spinal cord
B. ventral horn of the spinal cord
C. dorsal root ganglion of a spinal nerve
D. ventral root ganglion of a spinal nerve
E. ganglion of the autonomic nervous system
8. The cores of intestinal villi consist of
A. loose connective tissue and folds of muscularis externa
B. folds of submucosa that contain blood vessels and Meissner's plexus

C. a lamina propria that contains blood vessels, nerves, and lacteals


D. connective tissue and two fairly substantial layers of smooth muscle
E. none of the above
9. Eversion of valves of atrioventricular orifices of the heart is prevented by
papillary muscles and
A. the terminal crest
B. the ligamentum teres
C. the ligamentum venosum
D. chordae tendineae cordis
E. trabeculae carneae cordis

10. A lymph node is characterized by


A. containing medullary cords
B. lacking afferent lymphatics
C. having crypts lined with stratified squamous epithelium
D. having efferent lymphatics leaving at multiple sites from the capsule
E. all of the above
11. Considering number and location, the parathyroid glands are USUALLY
the thyroid gland.
A. single and dorsal to
B. single and embedded in the dorsum of
C. multiple and embedded in the dorsum of
D. multiple and just superior to
E. not related to
12. Which of the following is found in the curve of the duodenum?
A. Spleen
B. Cystic duct
C. Left kidney
D. Head of the pancreas
E. Fundus of the stomach
13. A major tract connecting right and left cerebral hemispheres is the
A. cingulum
B. corpus callosum
C. internal capsule
D. superior occipitofrontal bundle
E. none of the above
14. In which of the following glands is the blood supply MOST isolated from
the parenchyma?
A. Thymus
B. Spleen
C. Lymph node
D. Peyer's patch
E. Pharyngeal tonsil

15. An organ with a retroperitoneal location that can be approached surgically


without violating the community of the peritoneum is the
A. ovary
B. kidney
C. spleen D. gallbladder
E. liver
16. The component of bone tissue that gives bone tensile strength is the
A. elastic fibers
B. calcified ground substance
C. interconnecting canaliculi
D. periosteal connective tissue
E. collagenous fibrils of matrix
17. Which of the following epithelial surface variations serve PRIMARILY to
increase the functional surface area?
A. Cilia
B. Flagella
C. Microvilli
D. All of the above E. None of the
above
18. Articular surfaces of most diarthrodial joints are covered by
A. fibrocartilage
B. hyaline cartilage
C. elastic cartilage
D. articular disks
E. none of the above
19. Ependymal cells constitute the tissue that
A. lines the ventricles of the brain
B. lines the ventricles of the heart
C. forms a part of the peripheral neuroglia
D. covers nerve cell bodies in a ganglion
E. all of the above
20 Which of the following articulate directly with the body of the sternum?
I. Clavicle
II. Eleventh rib
III. Manubrium
IV. First rib
V. Xiphoid process
The CORRECT answer is:
A. I, II, IV
B. II, IV, V
C. III, IV, V
D. III, V
E. All
of the above
21. Ligamentous remnants of the fetal circulatory system persisting in the adult include
ligamentum
I. nuchae
II. venosum
III. arteriosum
IV. teres of the liver
V. teres of the uterus
The CORRECT answer is:
A. I, II, III
B. I, II, V
C. I, IV, V
D. II, III, IV
E. III, IV, V

22. In an adult, cerebrospinal fluid can be aspirated MOST safely by inserting


the needle between third and fourth lumbar vertebrae because
A. the spinal cord does not extend below lumbar
B. the subarachnoid space does not extend below lumbar
C. there is more space between laminae of these vertebra
D. there are no important nerves in this part of the vertebral canal
E. there is little danger of entering the internal vertebral plexus at this level
23. Epithelium of which of the following structures of the respiratory system
does NOT normally have cilia?
A. Trachea
B. Bronchioles
C. Alveolar ducts
D. Primary bronchus
E. All of the above
24. Which of the following organs have the CLOSEST anatomic relationship to
the right kidney?
I. Colon
II. Liver
III. Spleen
IV. Stomach
V. Duodenum
VI. Pancreas
The CORRECT answer is:
A. I, II, III
B. I, II, V
C. I, III, IV
D. II, IV, VI
E. III, V, VI
25. Which of the following organelles have a double-unit membrane?
I. Nucleus
II. Lysosome
III. Golgi complex
IV. Mitochondrion
V. Rough endoplasmic reticulum
The CORRECT answer is:
A. I, II, III
B. I, III
C. I, IV
D. I, IV, V
E. II, III, IV
KEY (CORRECT ANSWERS)
1.C
11.C
2.A
12.D
3.D
13.B
4.E
14.A
5.D
15.B
6.B
7.C
8.C
9.D
10.A

16.E
17.C
18.B
19.A
20.D
21. D
22. A
23. C

24. B
25. C

ANATOMIC SCIENCES
EXAMINATION SECTION
TEST 1

DIRECTIONS: Each question or incomplete statement is followed by several suggested


answers or completions. Select the one that BEST answers the question or completes the
statement. PRINT THE LETTER OF THE CORRECT ANSWER IN THE SPACE AT THE RIGHT.
1. What is a node of Ranvier?
A. One of the supporting cells in brain tissue
B. A nervous receptor for the sensation of pressure
C. The point of junction between two neurolemma (Schwann) cells
D. The point of near-contact between the processes of two neurons (a snyapse)
2. When stratified squamous epithelium thickens, which set of conditions
usually prevails? The
A. rete pegs increase in size and the intercellular bridges become more evident
B. basement membrane becomes more irregular and the dermis acquires an increased
lymphatic supply
C. stratum spinosum becomes less evident and the basement membrane thickens
D. number of vascular capillaries in the dermis increases and more sweat glands develop
3. Clots are more likely to appear in the cavernous sinus than in the other
dural sinuses because the cavernous sinus is located in close proximity to the
internal carotid artery?
A. Both statement and reason are correct and related.
B. Both statement and reason are correct but NOT related.
C. The statement is correct but the reason is NOT.
D. The statement is NOT correct but the reason is an accurate statement.
4. Which of the following structures leaves an impression on the right lung?
A. Azygos vein
B. Right vagus nerve
C. Right phrenic nerve
D. Descending thoracic aorta
5. The backflow of urine from bladder into the ureter is prevented by
A. a sphincter at the end of the ureter
B. a valve at the opening of the ureter into the bladder
C. the oblique course of the ureter through the bladder wall
D. none of the above

6. A tumor of the maxillary air sinus may cause an overflow of tears by


exerting pressure on the
A. lacrimal pore
B. lacrimal lake
C. lacrimal gland
D. nasolacrimal duct
7. The thoracic duct ascends through the thoracic cavity
A. in the anterior mediastinum
B. to the left of the descending aorta
C. in the posterior mediastinum
D. along the right side of the superior vena cava
8. In normal gingival epithelium, small spaces exist between the cells in
stratum spinosum (prickle cell layer). These spaces normally are filled by
A. keratin
B. a small amount of tissue fluid
C. capillaries which course between cells to approach the free surface
D. none of the above
9. The glands of the hard palate are
A. mucous predominantly
B. serous predominantly
C. serous with fat interspersed
D. albuminous with fat interspersed
10. The ducts of salivary glands which are lined by a single layer of tall,
columnar epithelial cells with radially arranged mitochondria are called
A. lobar ducts
B. primary ducts
C. striated ducts
D. intercalated ducts
11. The left recurrent laryngeal nerve is in intimate relation at its beginning to
the
A. arch of the aorta and the left pulmonary veins
B. ligamentum arteriosum and the arch of the aorta
C. pulmonary trunk and the left brachiocephalic vein
D. ligamentum arteriosum and the left brachiocephalic vein
12. The esophageal branches of the right vagus nerve are found MAINLY on the
A. left wall of the esophagus
B. right wall of the esophagus
C. anterior wall of the esophagus
D. posterior wall of the esophagus
13. Under normal conditions, the cell population of the connective tissue of
the free gingiva differs from that of the attached gingiva in the proportion of
A. monocytes
B. mast cells
C. lymphocytes
D. macrophages
14. Which of the following would be particularly well developed in cells
synthesizing proteins for secretion?
A. Lysosomes
B. Mitochrondria
C. Lipofuscin granules
D. Granular (rough) endoplasmic reticulum

15. Which of the following does NOT connect with the cavernous sinus
directly?
A. Sphenoparietal sinus
B. ,iuperior petrosal sinus
C. Inferior petrosal sinus
D. Transverse sinus
16. Which of the following statements concerning the periodontal ligament is

CORRECT?
A. Elastic fibers provide for most of the tooth movement.
B. Fibers of the periodontal ligament lengthen after a tooth becomes functionless.
C. The periodontal ligament has some continuity with the connective tissue of the
gingiva.
D. There are no epithelial cells in the periodontal ligament.
17. Which of the following groups of cells in the adrenal gland secrete
epinephrine?
A. Medullary cells
B. Zona fasiculata cells
C. Zona glomerulosa cells
D. Zona reticularis cells
18. The spleen is in close relation with the
A. duodenum
B. inferior surface of the diaphragm
C. ascending colon
D. right lobe of the liver
19. Myocardial fibers are characterized by
I. presence of sarcoplasm
II. interlacing arrangement
III. central location of nuclei
IV. presence of striations and intercalated disks
The CORRECT combination is:
A. I, II, III
B. I, III
C. II, III, IV
D. All of the above
20. The branches of the common hepatic artery are the
A. splenic arteries
B. short gastric arteries and gastroduodenal artery
C. gastroduodenal, left gastric and proper hepatic
D. gastroduodenal, right gastric and proper hepatic
21. The function of the chondroblast is to
A. provide nutrition to the cartilage
B. secrete glycogen that is needed during cartilage calcification
C. elaborate chondroitin-sulfuric acid by which the cells become entrapped
D. form fibers that are developed in the cartilage in interstitial growth
22. A function of the normal adult spleen is to
A. support life
B. filter lymph
C. store red blood cells
D. produce red blood cells
23. The mesencephalic portion of the trigeminal nerve is concerned with
A. connecting the mesencephalic nucleus with the spinal nucleus of the trigeminal nerve
B. sensory innervation of the larynx
C. proprioceptive sensations from the intrinsic muscles of the eye

D. proprioceptive sensations from the mandibular muscles and periodontal ligament


24. Histologically, adult compact bone is seen to
A. be identical in bones formed intramembranously and endochondrally
B. be lamellated in bones formed intramembranously but not in those formed
endochrondrally
C. contain cartilage cores in bones formed endochondrally
D. contain haversian systems only in bones formed intramembranously
25. The branch of the maxillary artery which provides the chief blood supply to
the posterior portion of the nasal cavity is the
A. buccal
B. masseteric
C. sphenopalatine
D. posterior superior alveolar
KEY (CORRECT ANSWERS)
1.C
7.C
8.B
14. D
15. D
21. C
22. C

2.A

3.B

4.A

9.A

10.C

11. B

16. C
23. D

17. A
24. A

18. B

5.C

6.D

12. D
19. D

13. C
20. D

25. C

TEST 2
DIRECTIONS: Each question or incomplete statement is followed by several suggested
answers or completions. Select the one that BEST answers the question or completes the
statement. PRINT THE LETTER OF THE CORRECT ANSWER IN THE SPACE AT THE RIGHT.
1. Osteoblasts are cells the cytoplasm of which stains intensely with basic
dyes. This is caused by the
A. high ribonucleic acid content
B. high content of potassium ions
C. increased number of mitochondria
D. presence of minute crystals of calcium salts
E. presence of sodium
2. Parathyroid glands are characterized by
A. forming from endoderm, and by being concerned with calcium metabolism
B. forming with duct systems that retrogress, and by having the same developmental
origin as the thyroid
gland
C. causing less dental calculus to form during hyperactivity
D. causing a removal of calcium from the body calcium stores during hypoactivity and by
being uniform in
location and number
E. excreting waste materials
3. The pituitary gland is most intimately related anteriorly to the
A. optic chiasma
B. mammillary bodies
C. anterior commissure

D. anterior communicating artery


E. posterior commisure
4. The connective tissue cells MOST concerned with defense against bacterial
invasion are
A. eosinophils
B. erthrocytes
C. basophils
D. macrophages
E. mast cells
5. An important cell organelle related to biochemical breakdown and
phagocytosis in the oral region is the A. lysosome
B. Llicrotubule
C. mitochondrion
D. Golgi apparatus
E. endoplasmic reticulum
6. The mitochondrial cristae are associated with
A. fat storage
B. proteolytic enzymes
C. hydrolytic enzymes
D. respiratory enzymes
E. none of the above
7. The bifurcation of the trachea lies at the level of the
A. sternal angle
B. xiphisternal junction
C. second thoracic vertebra
D. attachment of the fourth costal cartilage to the sternum
E. none of the above
8. The thyroid gland receives its blood supply from branches of the
A. external carotid arteries
B. thyrocervical trunks
C. brachiocephalic artery
D. all of the above
E. none of the above
9. The MAJOR portion of the medial pterygoid muscle is attached to the
A. medial surface of the medial pterygoid plate
B. medial surface of the lateral pterygoid plate
C. lateral surface of the lateral pterygoid plate
D. lateral surface of the great wing of the sphenoid
E. lateral surface of the lesser wing of the sphenoid
10. The sella turcica lies directly above the
A. pons
B. foramen ovale
C. frontal sinus
D. sphenoid sinus
E. maxillary sinus
11. The line of junction between the stratus germinativum of the skin and the
tissue which immediately underlies it (deep to the stratum germinativum) is
called

A. lamina propria
B. stratum lucidum
C. stratum granulosum
D. basement membrane
E. none of the above
12. Stabilization of the atrioventricular valves during contraction of the
ventricles is established by
A. papillary muscles
B. pectinate muscles
C. chordae tendineae
D. papillary and pectinate muscles
E. papillary muscles and chordae tendineae
13. The respiratory epithelium of the conducting portion is
A. pseudostratified ciliated columnar
B. stratified squamous
C. simple columnar
D. simple squamous
E. none of the above

usually

14. The common bile duct, hepatic artery and portal vein are found grouped in
the
A. mesocolon
B. lesser omentum
C. greater omentum
D. gastrosplenic ligament
E. none of the above
15. The external branch of the superior laryngeal nerve supplies the
A. omohyoid muscle
B. thyrohyoid muscle
C. cricothyroid muscle
D. sternothyroid muscle
E. anterior belly of the digastric muscle
16. The nerve plexus found in the submucosa layer of the small intestine is
called
A. Paneth's plexus
B. Auerbach's plexus
C. Meissner's plexus
D. Lieberkuhn's plexus
E. Goldstein's plexus
17. The primary lymph nodes draining the mandible are the
I. parotid nodes
II. retropharyngeal nodes
III. submandibular nodes
IV. submental nodes
The CORRECT combination is:
A. I, II
B. I, IV
C. II, III
D. II, IV
E. III, IV

18. Which of the following articular directly with the body of the sternum?
I. Clavicle
II. Eleventh rib
III. Manubrium
IV. Fifth rib
V. Xiphoid process
The CORRECT combination is:
A. I, IV, V
B. II, IV, V
C. III, IV, V
D. III, IV E. All of the above
19. Elastic cartilage is characterized by
A. having capsules around its lacunae that stain basophilic, and by being found in the
ribs
B. being found in the external ear, and by containing chondroitin-sulfuric acid
C. being located in the vocal cords, and by having parallel elastic fibers in the matrix
D. having a perichondrium containing blood vessels and by acquiring nutrition for the
chondrocytes by
lymphatic channels
E. none of the above
20. The structures which give rise to the face include the
A. first and second branchial arches
B. second and third branchial arches
C. frontal process and first branchial arch
D. frontal process and second branchial arch
E. all of the above
21. Podocytes form the
A. Henle's loop
B. distal convoluted tubule
C. proximal convoluted tubule
D. tunica media of afferent arteriole
E. visceral layer of Bowman's capsule
22. The right coronary artery arises from the
A. aortic sinus, supplies the right ventricle, right atrium, and empties into the coronary
sinus
B. coronary sinus, traverses the coronary sulcus, and anastomoses with the left coronary
artery
C. right aortic sinus, traverses the coronary sulcus, and anastomoses with the left
coronary artery
D. left aortic sinus, supplies the right ventricle, and empties into the coronary sinus
E. none of the above
23. Von Kupffer's cells specialize in
A. osmosis
B. pinocytosis
C. phagocytosis
D. chromocytosis
E. none of the above
24. Infections or neoplasms spreading by lymphatics from the skin of the nose
will pass to the

A. cavernous sinus
B. parotid nodes
C. anterior auricular nodes
D. pterygoid plexus
E. submandibular nodes
25. The feature which distinguishes bone from osteoid is
A. bone is not resorbed
B. osteoid contains a different type of fiber in its matrix
C. osteoid contains fewer lacunae per unit area
D. bone has a mineralized matrix
E. none of the above
KEY (CORRECT ANSWERS)
1. A
11. D
2. A
12. E
3. A
13. A
4. D
14. B
5. A
15. C
6. D
16. C
7. A
17. E
8. D
18. C
9. B
19. B
10. D
20. C
21. E
22. C
23. C
24. E
25. D

ANATOMIC SCIENCES
EXAMINATION SECTION
TEST 1
DIRECTIONS: Each question or incomplete statement is followed by several
suggested answers or completions. Select the one that BEST answers the
question or completes the statement. PRINT THE LETTER OF THE CORRECT
ANSWER IN THE SPACE AT THE RIGHT.
1. The ophthalmic artery is a branch of the
A. lacrimal artery
B. vertebral artery
C. middle cerebral artery
D. internal carotid artery
E. anterior meningeal artery
2. The pituitary gland is located in the hypophyseal fossa of which of the
following bones?
A. Ethmoid
B. Frontal
C. Sphenoid
D. Parietal
E. Occipital

3. Bile traverses the cystic and common bile ducts and then is emptied into
which part of the duodenum?
A. Ascending
B. Descending
C. Upper (superior)
D. Horizontal (inferior)
E. none of the above
4. Of the following, which supplies MOST of the blood to the parathyroid
glands?
A. Thyroidea ima a
B. Inferior thyroid a
C. Superior laryngeal a
D. Ascending cervical a
E. Ascending pharyngeal a
5. Which of the following muscles form lateral boundaries of the isthmus of
the fauces?
A. Palatoglossus and palatopharyngeus
B. Palatoglossus and tensor veli palatini
C. Palatoglossus and levator veli palatini
D. Palatopharyngeus and medial pterygoid
E. none of the above
6. Which lymphatic organ has efferent and afferent lymphatic vessels?
A. Spleen
B. Thymus
C. Lymph node
D. Palatine tonsil
E. Pharyngeal tonsil
7 Cellular cementum differs from acellular cementum in that there are
A. cementoids in cellular cementum
B. no Sharpey's fibers in cellular cementum
C. cementocytes embedded in the matrix in cellular cementum
D. completely different intercellular substances in the two
E. there is no difference
8. The pituitary is characterized by
A. being ectodermal in origin, and by having its pars tuberalis affect growth of the
individual
B. having its adenohypophysis portion appear glandular and by having the pars
intermedia affect temperature regulation
C. yielding a hormone called LH that concerns the ovary, and producing ACTH which
affects the adrenal medulla
D. possessing a rich vascular supply, and by having its pars anterior derived from
Rathke's pouch
E. all of the above
9. Ependymal cells constitute the tissue that
A. lines ventricles of the brain
B. lines ventricles of the heart
C. forms a part of the peripheral neuroglia

D. covers the nerve cell body in the ganglion


E. lines the stomach and small intestine
10. Histologic histochemical and physiologic evidence indicates that compact
bone functions as a
A. supportive tissue of the body
B. supportive tissue and part of the hematopoietic tissue of the body
C. supportive tissue, part of the hematopoietic tissue, and part of the fat depot of the
body
D. supportive tissue, part of the hematopoietic tissue, part of the fat depot, and the
mineral depot of the
body
E. none of the above
11. The efferent lymph vessels leave the lymph node at the
A. hilum
B. stroma
C. capsule
D. germinal center
E. trabeculae
12. The mediastinum is basically the region between the two pleural cavities
which contains
A. the heart and its pericardium
B. the vessels proceeding to and from the heart
C. the trachea
D. structures in transit from the neck to abdomen, i.e., esophagus, phrenic and vagus
nerves, and thoracic duct
E. all of the above
13. Cell bodies of preganglionic parasympathetic neurons to the duodenum are
in the
A. nucleus ambiguus
B. submucosal and myenteric plexuses
C. sacral spinal cord lateral gray column
D. thoracic spinal cord lateral gray column
E. dorsal motor nucleus of the vagus nerve
14. The motor fibers of spinal nerves are derived from cells located in the
A. dorsal root ganglion
B. gray matter of the cord
C. white matter of the cord
D. lower part of the brain stem
E. none of the above
15. Layers of the gastrointestinal tract in order from the lumen side
outwardly are:
A. Mucosa, submucosa, tunica muscularis, tunica adventitia
B. Submucosa, tunica adventitia, mucosa, tunica muscularis
C. Tunica adventitia, tunica muscularis, mucosa, submucosa
D. Tunica muscularis, submucosa, tunica adventitia, mucosa
E. None of the above
16. Thin myofilaments and thick myofilaments are observed in skeletal muscle
in E/M preparations. The thick myofilaments
A. consist of actin mainly

B. are present only in the A disk


C. are present in both the A and I disks
D. are traversed or crossed by the Z band
E. all of the above
17. Serous cells making up a demilune secrete into the
A. bloodstream
B. intercellular canaliculi
C. lumen of the acinus directly
D. tissue fluid, thence to the duct system
E. any of the above
18. Transitional epithelium from the empty urinary bladder can be
distinguished from stratified squamous epithelium because
A. transitional epithelium does not show obvious stages of cell life cycle as is seen in
stratified squamous
epithelium
B. stratified squamous epithelium is derived mainly from ectoderm whereas transitional
epithelium is derived
entirely from endoderm
C. the surface cells of stratified squamous epithelium are rounded whereas the
comparable cells in
transitional epithelium are pear-shaped
D. the mitotic rate of transitional epithelium is greater than that of stratified squamous
epithelium
E. the mitotic rate of transitional epithelium is less than that of stratified squamous
epithelium
19. The mucosa of the gingiva has
A. an epithelium which is usually unkeratinized
B. a lamina propria with tall papillae
C. an identifiable submucosa
D. all of the above
E. none of the above
20. The greater splanchnic nerve is composed of
A. special visceral efferent fibers
B. sympathetic fibers from the vagus nerve
C. parasympathetic fibers from the vagus nerve
D. sympathetic fibers from vertebral ganglia T5-T9
E. postganglionic fibers from the celiac ganglion
21. The infrahyoid muscles receive their motor innervation from
A. the pharyngeal plexus
B. branches of the cervical plexus
C. the vagus nerve
D. supraclavicular nerves
E. none of the above
22. In the human, a renal papilla projects directly into the
A. ureter
B. minor calyx
C. major calyx
D. renal pelvis
E. none of the above

23. Which of the following muscles is NOT in contraction during a forced


expiration?
A. Diaphragm
B. Rectus abdominis
C. Transversus thoracis
D. Transversus abdominis
E. Abdominal external oblique
24. Which of the following statements does NOT apply to mitochondria?
A. They contain lipid droplets.
B. They are found in all nucleated cells.
C. They can be stained in living cells.
D. They show cristae in electron micrographs.
E. They vary in shape, number and location in different cells.
25. On the basis of definitions for the various body fluids, which of the
following is CORRECT?
A. Blood serums is the fluid part of the blood after blood coagulation has taken place.
B. Lymph is the fluid contained in lymphatic channels and it has less colloid in it than
does tissue fluid.
C. Tissue fluid is the fluid part of the blood that has passed through the endothelial lining
of the arterioles.
D. Blood plasma is the liquid part of the blood after the platelets have caused
agglutination to take place.
E. none of the above
KEY (CORRECT ANSWERS)
1. D
11.
2. C
12.
3. B
13.
4. B
14.
5. A
15.
6. C
7. C
8. D
9. A
10. A
21. B
22. B
23. A
24. A
25. A

16.
17.
18.
19.
20.

TEST 2
DIRECTIONS: Each question or incomplete statement is followed by several suggested
answers or completions. Select the one that BEST answers the question or completes the
statement. PRINT THE LETTER OF THE CORRECT ANSWER IN THE SPACE AT THE RIGHT.
1 The anterior boundary of the superior mediastinum is the
A. first rib

B. manubrium
C. body of the sternum
D. anterior pericardium
E. reflection of the pericardium off the great vessels
2 The end of the conus medullaris of the spinal cord is situated at the upper
border of the
A. 2nd lumbar vertebra
B. 2nd sacral vertebra
C. 2nd thoracic vertebra
D. 4th lumbar vertebra
E. 4th sacral vertebra
3. The hepatic portal vein is usually formed by the union of the
A. superior mesenteric vein and the splenic vein
B. left renal vein and the inferior mesenteric vein
C. inferior vena cava and the superior mesenteric vein
D. superior mesenteric vein and the inferior mesenteric vein
E. right and left gastroepiploic veins and the superior mesenteric vein
4. Long bones of the skeleton increase in length because of
A. mitotic division of osteocytes
B. mitotic division of osteoblasts
C. resorption of primary bone by the osteoclasts
D. appositional growth on the cartilaginous epiphyseal plate
E. interstitial growth in the cartilaginous epiphyseal plate
5. The prime muscle in retracting and elevating the mandible is the
A. masseter
B. digastric
C. mylohyoid
D. temporalis
E. lateral pterygoid
6. In proceeding from the bronchus to the respiratory bronchiole, there is a(n)
A. decrease in cartilage and an increase in elastic fibers
B. decrease in collagenous fibers and an increase in cilia
C. decrease in cilia and an increase in cartilage
D. increase in cilia and a decrease in elastic fibers
E. none of the above
7. The endocrine gland characterized by colloid-filled follicles is the
A. testis
B. thyroid
C. parathyroids
D. adrenal cortex
E. anterior lobe of pituitary
8. The cell that aids in immunological defense of the body is known as a
A. mast cell
B. giant cell
C. macrophage
D. plasma cell
E. none of the above
9. Cartilage, such as that in human trachea, is characterized by

A. containing chondrocytes in lacunae. The cells in one lacuna communicate with those
in adjacent lacunae by means of cytoplasmic processes in canaliculi
B. possessing matrix in which the few collagen fibers have a network pattern. These
fibers are embedded in
mucopolysaccharide that is nonsulfated
C. having chondrocytes receive nutrients by process of diffusion through cartilage
matrix. Vascular channels
in perichondrium are the source of the nutrients
D. having chondrocytes reside in nests, with nests surrounded by capsule. This capsule
consists of an
envelope of loose connective tissue which lacks collagen fibers
E. none of the above
10. Thyroid gland of adult human is characterized by
A. being located anatomically in the anterior inferior position of the neck.
Developmentally, the gland is
derived from ectoderm as a downgrowth in the midline of the neck from the apex of the
V-shaped row of
vallate papillae of tongue
B. consisting of bilateral lobes connected across the midline of the neck by the isthmus.
In contrast to
the lobes which possess follicles containing colloid, the isthmus is lacking in follicular
components
C. possessing a rich vascular supply. Of the vessels supplying the gland, the superior
thyroid artery is
a branch of the internal carotid artery whereas the inferior thyroid artery is a branch of
the external
carotid artery
D. having a true capsule of the gland consisting essentiall of loose ordinary (areolar)
connective tissue.
Internally, the follicles are supported by stroma whose reticular fibers are the main
fibrous component
E. all of the above
11. The pituitary is derived from
A. endoderm of foregut
B. ectoderm of stomodeum
C. ectoderm of forebrain
D. both A and B above
E. both B and C above

12. In the fetus, the ductus venosus conveys blood from the
A. pulmonary vein to the aorta
B. right atrium to the left atrium
C. pulmonary artery to the pulmonary vein
D. umbilical vein to the inferior vena cava
E. none of the above
13. The epithelium lining the paranasal sinuses is properly classified as
A. ciliated, pseudostratified
B. stratified squamous
C. ciliated, stratified columnar
D. ciliated, simple cuboidal
E. none of the above
14. Osteocytes communicate with the haversian canals via
A. Volkmann's canals
B. Howship's lacunae
C. interstitial lamellae
D. eanaliculi and lacunae
E. all of the above
15. In a lymph nodule, the lighter staining central area is known as the
A. pulp
B. cortex
C. lymph sinus
D. germinal center
E. all of the above
16. What structure separates the stomodeum from the foregut?
A. Rathke's pouch
B. Maxillary process
C. Meckel's cartilage
D. Buccopharyngeal membrane
E. none of the above
17. Meissner's corpuscles are often located in a
A. dental papilla
B. dermal papilla
C. vallate papilla
D. filiform papilla
E. cardiac papilla
18. The inguinal ligament runs between two palpable bony landmarks. These
are the
A. public tubercle and the iliac tubercle
B. symphysis pubis and inferior iliac spine
C. anterior and posterior superior iliac spines
D. anterior superior iliac spine and the public tubercl
E. none of the above
19. The crescents or demilunes of the mucous alveoli of the sublingual gland
are composed of
A. serous cells
B. neural cells
C. striated cells
D. myo-epithelial cells
E. any of the above

20. The inferior mesenteric artery supplies a portion of the


A. small intestine and pancreas
B. small intestine only
C. colon and rectum
D. ascending colon
E. small intestine and rectum
21. After a skull fracture, blood appears at the pharyngeal opening of the
auditory tube. This may be the result of bleeding from the
A. middle ear
B. maxillary sinus
C. sphenoidal sinus
D. posterior cranial fossa
E. nasal passage
22. The lamina propria of attached gingiva is primarily
A. alveolar bone
B. elastic fibers
C. reticular fibers
D. coarse collagenous fibers
E. none of the above
23. The epithelium of the oral pharynx is
A. simple columnar
B. stratified squamous
C. pseudostratified
D. stratified columnar
E. neostratified
24. The maxillary sinus opens into the
A. middle meatus
B. inferior meatus
C. superior meatus
D. superior concha
E. sphenoethmoidal recess
25. The right subclavian and right common carotid arteries arise from the
A. thoracic aorta
B. ascending aorta
C. pulmonary artery
D. arch of the aorta
E. brachiocephalic artery
KEY (CORRECT ANSWERS)
1. B
2. A
3. A
4. E
5. D
6. A
7. B
8. D
9. C
10. D
11. E
12. D
13. A
14. D
15. D
16. D
17. B
18. D
19. A
20. C
21. A
22. D
23. B
24. A
25. E

EXAMINATION SECTION

TEST 1
DIRECTIONS: Each question or incomplete statement is followed by several suggested
answers or completions. Select the one that BEST answers the question or completes the
statement. PRINT THE LETTER OF THE CORRECT ANSWER IN THE SPACE AT THE RIGHT.
1. According to psychoanalytic theory, the part of the personality which is in
closest contact with reality is the
A. id
B. superego
C. libido
D. ego
2. An individual who gives socially acceptable reasons for his behavior, either
verbally, by thought, or conduct, is adjusting through the use of
A. rationalization
B. sublimation
C. retrogression
D. displacement
3. Of the following statements, the MOST nearly correct one
rate of mental growth is that
A. there is a deceleration of the rate of growth with age
B. there is an increase in the rate of growth with age
C. mental growth is constant throughout the period of childhood
D. mental growth is constant during adulthood

regarding the

4. In early childhood, the individual tends to pattern himself on or to identify


himself MOST generally with
A. glamorous or romantic figures
B. age contemporaries
C. characters in movies or on TV
D. parents or parent substitutes
5. With respect to physical growth, superior children as compared with
children of average intelligence are
A. slightly inferior
B. above average C. slightly superior
D. markedly
inferior
6. A candidate in an examination says, "I passed the written and the performance
tests, but they failed me in the interview." The mechanism of personality defense
which he is employing is
A. compensation
B. sublimation
C. identification
D. projection
7. The psychological forces or needs that influence human behavior are
labeled
A. extrasensory
B. generalized
C. intrinsic
D. extrinsic
8. Reactions of nail biting, grimacing, clawing, spitting, etc. in a fourth-grade
child are usually considered
symptomatic evidence of
A. low intelligence
B. psychological conflict
C. hypothyroidism
D.
cretinism
9. The greatest "social distance" in boy-girl relationships has been found to be
during the ages

A. 13 to 17 years
D. 2 to 5 years

B. 9 to 13 years

C. 5 to 9 years

10. According to available findings, the effect of deprivation of affection on


intellectual development is MOST likely to appear in a curtailment of the
A. speed of learning even when the task is rather simple
B. ability to memorize new material
C. ability to retain material, once it has been learned
D. ability to conceptualize
11. Of the following, the one which would MOST likely indicate faulty
emotional development in a girl of six is
A. striving for perfection in all her work
B. stronger liking for music than other school work
C. little interest in doll play
D. reluctance to engage in competitive sports
12. When a six-year-old child violates the standards of conduct of the group,
the teacher should
A. criticize him in the presence of the group
B. accept his behavior as the expression of a deep need
C. support him in order to alleviate his guilt feelings
D. discourage his behavior by showing why it is wrong
13. A shy child is MOST likely to be fairly well adjusted if he has
A. clearly defined interests
B. marked intellectual ability
C. obvious physical handicaps
D. outstanding artistic talent
14. A kindergarten child shows habitual reluctance to undertake a new
activity. This is BEST interpreted as evidence of
A. an inability to cope with adult authority
B. a general attitude of insecurity
C. a specific fear conditioned in infancy
D. a persevering and independent attitude
15. For a four-year-old child, the events of the present are
A. less vivid than those of the past
B. less vivid than those of the future
C. more vivid than those of the past or future
D. as vivid as those of the past or future
16. The "ideal self" represents what an individual
A. believes he ought to be
B. believes others ought to be
C. knows he can be
D. knows he cannot be
17. Girls generally prefer groups of girls and boys prefer groups of boys
during
A. early childhood
B. latency
C. pre-adolescence
D. adolescence

18. The rate and pattern of early motor development are largely determined
by
A. experience
B. learning
C. maturation
D. training
19. Which of the following books describes how parents and other adults can
help youngsters overcome problems of an urban environment?
A. YOUR ADOLESCENT, L. K. and Mary Frank
B. STUDIES IN ADOLESCENCE, Robert E. Grinder
C. IN DEFENSE OF YOUTH, by Earl C. Kelley
D. THE AMERICAN TEENAGER, by H.H. Remmers and D.H. Radler
20. When a person has gained some insight into his own emotional behavior,
usually following resolution of an acute conflict, we often describe him as
having increased his range of
A. emotional repression
B. affective mobility
C. understanding
D. clinical
synapses
21. Sibling rivalry is the term used to describe the competitive feeling
between two or more individuals who
A. are in the same school grade
B. are children of the same parents
C. have similar goals of achievement
D. are in the same chronological age group
22. The mental mechanism of minimizing one's own faults and deficiencies by
criticizing and blaming others is known as
A. compensation
B.
rationalization
C. transference
D. projection
23. Etiology is concerned primarily with
A. symptomology
B. racial origin

C. causation

D. language facility

24. Syndrome is BEST defined as a


A. form of obsession in which the subject sees himself as someone else
B. form of neurosis in which the subject constantly compares himself with others
C. cardiac condition which has no apparent organic basis
D. constellation of symptoms which characteristically occur together in a specific ailment
25. Girls tend to be superior to boys of the same age in
A. linguistic fluency
B. speed of reaction time
C. arithmetical reasoning
D. most forms of perception
26. Of the following, a major recreational activity common
15-year-old boys and girls is
A. going to the movies
B. riding a bicycle
C. watching athletic sports
D. social dancing

to both 10- and

27. In general, juvenile fiction comprises the major part of the reading choices
of
A. girls between 9 and 13
B. boys of all ages
C. girls of all ages
D. boys between 12 and 16
28. The behavior of the typical adolescent is BEST described as
characteristically
A. stubborn and willful, showing disregard for strictures of family and society
B. inconsistent, alternating between childish and adult reactions
C. irresponsible, exhibiting lack of judgment and poor taste
D. individualistic, reflecting indifference to approval from parents and peers

29. Of the following observed behavior symptoms, the one which may BEST be
described as "regression" is the pupil's
A. use of infantile speech and verbal expressions
B. thrusting aside of present desires in order to avoid conflict in a direct solution of a
problem
C. attempt to dominate every situation in which he finds himself
D. evasion of possible failure by selecting an easier goal
30. UNDERSTANDING GROUP BEHAVIOR OF BOYS AND GIRLS was written by
A. Helen H. Jennings
B. Ruth Cunningham
C. Jane Waters
D.
Alice V. Crow
31. When we compare young children and adolescents with respect to the
relative effectiveness of distributed and concentrated practice as a learning
technique, we find that
A. young children learn better by distributed practice; adolescents by concentrated
practice
B. young children learn better by concentrated practice; adolescents by distributed
practice
C. both young children and adolescents learn better by concentrated practice
D. both young children and adolescents learn better by distributed practice
32. Research has demonstrated that there is an increase in racial prejudice
during adolescence. Of the following, the factor that contributes MOST
significantly to this increase is
A. dislike for deviants from norms of social groups
B. influence of parental opinion
C. segregation of groups in school and community
D. fear of economic pressure from minority groups
33. Of the following teachers, the one MOST liked by the largest number of
junior high school pupils is the one who
A. sets easily attainable standards

B. demonstrates a high level of intellectual competence


C. maintains an impersonal, objective attitude
D. is sympathetic
34. Of the following, the behavior which would be considered MOST indicative
of potential or actual maladjustment in a junior high school boy is
A. treating his classmates to sodas in an attempt to buy their votes in a school election
B. spending his entire allowance each week on science fiction paperbacks
C. finding fault with the work of his classmates
D. failing to take care of school property
35. The proportion of TREATED juvenile delinquents who exhibit subsequent
histories of failure to adjust to society is about
A. 5%
B. 25%
C. 45%
D. 65%

36. Current evidence and thinking on the causative factors in juvenile


delinquency support the view that
A. social factors are more basic than psychological factors
B. psychological factors are more basic than social factors
C. psychological factors and social factors are of about equal importance
D. physiological factors are more important than either social or psychological factors
37. Studies involving the relative mental abilities of delinquent and nondelinquent children have generally
A. shown that there are no significant differences between them
B. shown that delinquent children are slightly but significantly brighter than nondelinquents
C. shown that non-delinquent children are somewhat brighter than delinquent children
D. been about evenly divided -- some finding the delinquent children brighter, others
finding mental
superiority for non-delinquents
38. Separation of the infant from his mother can be a traumatic experience.
The amount of emotional damage to the infant and the consequent effects on
his personality depend MAINLY on the
A. quality and consistency of the substitute mothering he receives
B. reasons for and duration of the separation
C. kind of preparation for separation the infant receives
D. degree of the mother's acceptance of the placement
39. Research studies of language development in young children have shown
that
A. the multiple mothering of children in a large family retards language development

B. language retardation in otherwise normal children is usually related to inadequate


language stimulation
C. language retardation is always associated with slow motor development
D. children are usually slow in learning to talk when more than one language is spoken in
the home
40. The two MOST important influences on the cultural development of a
seven-year-old child are the
A. home and peer group B. school and peer group C. home and school
D. home
and church
41. In our culture, a child gains his sense of identity MAINLY from
A. knowledge about and experience with his parents and extended family
B. association with members of his own ethnic group
C. a study of the historical and ethnic factors in this culture
D. association with his peers
42. Of the following, the MOST important influence on the
development of a child during the first year is the
A. family as a whole
B. mother
C. way his siblings react to him
D. relationship between the parents

personality

43. Of the following, the term which is generally applied to the situation in which an
infant in foster care has
insufficient interaction with a substitute mother is
A. maternal rejection
B. mothering complex
C. maternal deprivation D.
interaction deficiency

44. The normal four-year-old child should be expected to


A. cut her meat with a knife
B. bathe herself without help
C. care for herself at the toilet
D. tell time to the nearest quarter hour
45. It is usually not a good idea to take a child under the age of five to a
movie that may frighten him MAINLY because young children cannot
A. appreciate a cultural experience
B. behave themselves in a movie theater
C. distinguish clearly between real life and make believe
D. see movies without acting out what they see
46. The average five-year-old child spends the MAJOR part of his play time
A. playing by himself
B. watching other children play
C. playing cooperatively with other children
D. playing competitive games involving teams

47. A children's counselor faced with a question about sex from a six-year-old
child in her group should
A. tell the child she is too young to understand such things
B. give the child as honest and simple an answer as possible
C. realize that an older child must have told the six- year-old to ask that question
D. answer the question in such a way as to discourage the child from asking any more
questions about sex
48. Most studies of children's fears indicate that fears of the supernatural are
MOST common among the
A. pre-schoolers
B. latency-age group
C. pre-adolescents
D. adolescents
49. The boy who trips on the leg of a chair and then accuses the chair is using
the mechanism of
A. rationalization
B. regression C. daydreaming
D. projection
50. Mild amounts of emotion, such as anxiety, irritation, and apprehension,
tend to have which of the following influences on learning and performance?
A. Integrative
B. Mildly disintegrative
C. Disruptive
D.
Relatively little influence
KEY (CORRECT ANSWERS)
1. D
11. A
21.B
31. D
41. A
2. A
12. D
22.D
32. A
42. B
3. A
13. A
23.C
33. D
43. C
4. D
14. B
24.D
34. C
44. C
5. C
15. C
25.A
35. B
45. C
6. D
7. C
8. B
9. B
10. D

16. A
17. B
18. C
19. C
20. B

26.A
27.A
28.B
29.A
30.B

37. C
39. B
40. C

36. B
38. A

47. B
49. D
50. A

46. C
48. B

TEST 2
DIRECTIONS: Each question or incomplete statement is followed by several
suggested answers or completions. Select the one that BEST answers the
question or completes the statement. PRINT THE LETTER OF THE CORRECT
ANSWER IN THE SPACE AT THE RIGHT.
1. The process by which children take to themselves the values, the thinking,
and the social behavior of their parents is known as
A. projection
B. identification
C. denigration
D. sublimation
E.
imitation
2. An understanding of the family relationships of a youngster who presents a
problem of under-achievement
A. is worthwhile but not essential
B. is important but not within the province of the teacher
C. may reveal factors that have an important bearing on his problem
D. is unlikely to be related to the difficulties the young person has with his school work
E. is important but not within the province of the guidance counselor

3. Which of the following is the MOST correct statement concerning puberty


and physical maturity?
A. Boys and girls who experience early puberty will achieve physical maturity and cease
growing later
than will the late maturers.
B. Boys and girls who experience early puberty will achieve physical maturity and cease
growing sooner
than will the late maturers.
C. Boys and girls who experience early puberty will achieve physical maturity and cease
growing at
approximately the same time as the late maturers.
D. Boys and girls who experience early puberty will achieve physical maturity and cease
growing in any
standard pattern, together with the late maturers.
E. None of the above.
4. The MOST prominent difficulties of the middle years of
around
A. relations with peer groups
B. parent-child relationships
C. schooling and the ability to learn
D. physical development
E. emotional and spiritual development

childhood revolve

5. The MOST accurate statement concerning anxiety, of the following, is that


anxiety is
A. needed for the socialization process
B. not needed for the socialization process
C. less produced by "mental" punishment than by physical punishment
D. of negligible effect in producing neurosis
E. neutralized by feelings of guilt and inadequacy
6. Most studies of children's fears indicate that fears of the supernatural are
MOST common among the
A. early childhood group
B. latency-age group
C. pre-teen group
D. adolescents
E. early childhood group and adolescents as contrasted with the pre-teen and latencyage groups
7. The boy who trips on the leg of a chair and then accuses the chair is using
the mechanism of
A. rationalization
B. regression
C. sublimation
D. projection
E.
delusion

8. Mild amounts of emotion, such as anxiety, irritation, and apprehension, tend


to have which of the following influences on learning and performance?
A. Integrative
B. Mildly disintegrative
C. Disruptive
D. Integrative and segregative
E. Disruptive and disintegrative
9. The "ideal self" represents what an individual
A. believes he ought to be
B. believes others ought to be
C. knows he can be
D. knows others can be
E. believes he ought to be and can be
10. Girls generally prefer groups of girls AND boys prefer
during
A. early childhood
B. latency
C. pre-adolescence
D. adolescence
E. early childhood and pre-adolescence

groups of boys

11. The rate and pattern of early motor development are largely determined
by
A. experience
B. learning
C. maturation
D. training
E.
practice
12 Changes in cognitive behavior between childhood and
in the direction of greater
A. understanding of abstractions
B. reliance on concrete realities
C. dependence on vicarious experiences
D. emphasis on intuition
E. collection of scattered peripheral details

adolescence are

13. Gesell refers to the child's development between the ages of six and ten as
A. an ever-widening spiral
B. a task phase
C. a latency period
D. a transitional interval
E. an imitative stage
14. A defense mechanism defined as the adoption of an attitude opposite to
one that precedes anxiety is
A. sophistry
B. reaction formation
C. fantasy
D. verification
E. rationalization
15. Experiments with the reactions of varying age groups to
that, in general, the greatest fear was exhibited by

snakes indicated

A. infants
E. adults

B. primary pupils

C. latency-age children

D. adolescents

16. As children get older, the differences in ability between the bright and the
dull tend to
A. become smaller
B. become larger
C. remain about the same
D. vary in no set pattern
E. become larger, level off, and then become smaller
17. Which one of the following descriptions is characteristic of the activelyrejected home?
A. Parents show highly emotional attitude, warmth without understanding.
B. Parents tend to use severe punishment only if irritated.
C. Parents restrict child's independence with many rules and requirements.
D. Child is expected to make his own decisions, although advice is unavailable.
E. Parents are jealous of, and in active competition with, their children.
18. Homesickness is an example of which type of adjustment mechanism?
A. Guilt
B. Identification
C. Reaction
D. Sublimation
E. Regression
19. Children below the age of 10 when asked to write on "The person I would like
to be" most frequently want to vie with
A. peers
B. older siblings
C. parents
D. real or imaginary heroes
E. teachers
20. When a child strikes out at a person who did not provoke the anger, the
child is exhibiting
A. depression
B. retrogression
C. compensation
D. displaced aggression
E. projection
21. Which of the following statements describes the effect upon intellectually
gifted children of early school entrance and acceleration?
A. It serves to retard their social development.
B. It produces underachievers.
C. It creates emotional problems.
D. It leads to favorable and valuable results.
E. It substantially affects the development of motor skills.
22. The psychological forces or needs that influence human behavior are
labeled
A. extrasensory
B. extrinsic C. intrinsic
D. generalized
E.
contingent
23. Reactions of nail biting, grimacing, clawing, spitting, etc. in a fourth-grade
child are usually considered
symptomatic evidence of
A. cretinism
B. dementia
C. low intelligence
D. hypothyroidism E.
psychological conflict

24. An individual who gives socially acceptable reasons for his behavior, either
verbally, by thought, or conduct, is adjusting through the use of
A. rationalization
B. displacement
C. sublimation
D. retrogression
E.
projection
25. Of the following statements, the MOST NEARLY correct one regarding the
rate of mental growth is that
A. there is a deceleration of the rate of growth with age
B. mental growth is constant during adulthood
C. mental growth is constant throughout the period of childhood
D. there is an increase in the rate of growth with age
E. the rate of mental growth increases and decreases in a standard pattern
26. In early childhood, the individual tends to pattern himself on or to identify
himself MOST generally with
A. glamorous or romantic figures
B. age contemporaries
C. characters in movies or on TV
D. parents or parent substitutes
E. older siblings
27. With respect to physical growth, mentally superior children as compared
with children of average intelligence are
A. markedly inferior
B. slightly inferior
C. slightly superior
D. markedly superior
E. about average

28. A candidate in an examination says,


"I passed the written and the performance tests, but they failed me in the interview."
The mechanism of personality defense which he is employing is
A. compensation
B. sublimation
C. identification
D. projection
E.
rationalization
29 The psychologist whose name is MOST often associated with the theory
that the experience of birth has a profound influence on personality
development and that an individual who has a slow, prolonged birth is likely to
have a personality which fights, struggles and plunges is
A. Horney
B. Freud
C. Sullivan
D. Rank
E.
McDougall
30 Two different studies have suggested that children who will probably
have lower I.Q.'s are those who have been reared in
A. institutions
B. broken homes
C. foster homes
D. upper class homes
E. lower class homes

31. Studies of twins reared together indicate that the correlation coefficients
of intelligence tests scores
for identical twins lie in the range of
A. .40-.55
B. .65-.74
C. .75-.79
D. .80-.90
E. .90-1.00

32. As a means of changing the current behavior pattern of an adolescent,


which of the following forces will generally prove to be MOST potent?
Disapproval of the behavior pattern by
A. the adolescent's parents
B. an adult he admires
C. a group of his peers
D. his classroom teacher
E. older siblings
33. If the results of studies of boys ' clubs are applicable to the school
situation, one may expect the greatest amount of aggressive behavior to be
noted in classes where the classroom climate may be described as
A. permissive
B. laissez-fair
C. democratic
D. autocratic
E.
unstructured
34. Of the following, the LEAST effective way of dealing with children's fears is
A. explaining and reassuring
B. helping the child to face the feared situation
C. simply ignoring the child's fears
D. setting examples of fearlessness
E. removing the cause of fear from the child's environment
35. The age at which individuals cease to grow in intellectual ability is
A. 13 years
B. 16 years C. 21 years
D. 35 years E. probably none of
these
36. The theory that physical compensation for a feeling of physical or social
inferiority is responsible for the development of a psychoneurosis is attributed
to
A. Adler
B. Horney
C. Freud
D. Sullivan
E. Jung
37. Which of the following terms refers to the maintenance of stability in the
physiological functioning of the organism?
A. functional autonomy
B. canalization
C. homeostasis
D. maturation
E. physiological integration
38. Which of the following authors would you be LEAST likely
information about child care?

A. Sidonie Gruenberg
E. Arnold Gesell

B. Jean Piaget

C. Ernest Harms

to recommend for

D. Benjamin Spock

39. All of the following statements are generally TRUE of children of


elementary school age EXCEPT
A. girls mature approximately one year earlier than do boys
B. girls have poorer health than boys
C. girls excel in body balance and fine hand coordination
D. girls excel in school achievement
E. girls tend to get their second set of teeth earlier than boys

40. The teacher of a sixth-grade class is likely to find all of the following
characteristics among children of this growth level EXCEPT that they
A. are influenced very little by what their peers do
B. are beginning to rebel against adult domination
C. are at a receptive stage for indoctrination of all sorts
D. enjoy giving assistance to younger children in the lower grades
E. are beginning to show more discrimination in the selection of possessions and in the
care of them

KEY (CORRECT ANSWERS)


1.
2.
3.
4.
5.

B
C
B
C
A

6. B
7. D
8. A
9. A
10. B

11.C
12.A
13.A
14.B
15.E

21.
22.
23.
24.
25.

D
C
E
A
A

31.
32.
33.
34.
35.

D
C
D
C
E

16.B
17.C
18.E
19.C
20.D

26.
27.
28.
29.
30.

D
C
D
D
A

36.
37.
38.
39.
40.

A
C
B
B
A

EXAMINATION SECTION
DIRECTIONS:
Each question or incomplete statement is followed by several suggested answers or
completions. Select the one that BEST answers the question or completes the statement.
PRINT THE LETTER OF THE CORRECT
ANSWER IN THE SPACE AT THE RIGHT.

Questions 1-4.
DIRECTIONS:

Questions 1 through 4 are based on the data in the table


below. Select the CORRECT answer for each question from
the following key:
1. True statement, supported by the data directly or by inference
2. Insufficient data furnished to come to this conclusion
3. False statement, contradicted by the data directly or by inference

Relative Importance for Employee


Ranked by
Morale
Employer
Credit for work done
Interesting work
Fair pay
Understanding and appreciation
Counsel on personal problems
Promotion on merit
Good physical working condition
Job security

As Ranked by

As

Employee
1
2
3
4
5
6
7
8

7
3
1
5
8
4
6
2

1. On the whole, employers have very good insight into the motivation of their
employees.
The CORRECT answer is:
A. 1
B. 2
C. 3
2. Workers tend to make unreasonable demands. The CORRECT answer is:
A. 1
B. 2
C. 3
3. The motive of economic fear is stronger in the worker than the motive of
social recognition.

The CORRECT answer is:


C. 3

A. 1

B. 2

4. Workers tend to favor promotion on merit while employers tend to favor promotion on
the basis of seniority. The CORRECT answer is:
A. 1
B.
2
C. 3
5. Whenever possible psychologists prefer to make quantitative classifications
rather than qualitative classifications. This is GENERALLY true for the reason
that
A. there is too much overlapping of categories when classes are qualitative
B. qualitative information is only descriptive and cannot be used for effective control
C. mathematical relationships must be determined before an acceptable theory can be
established
D. more powerful techniques are available for handling quantitative data than qualitative

Questions 6-10.
DIRECTIONS: In each of questions 6 through 10, select the one item, among
those listed, that does not belong; i.e., it is not in the same class, or it is the
only one relevant (or irrelevant), or it is the only one right (or wrong), etc. The one that does NOT belong is:
6.

A. Reaction formation
B. Displacement
C. Projection
D. Hostility

7.

A. Conditioned response
B. Motor skill
C. Insight
D. Maze behavior

8.

A. A gull's warning to her chicks


B. Language of the bees
C. Courtship pattern of the stickleback
A .dog's begging

9.

A. Salivation at the sight of food


B. EVQ-Watering with grit in eye
C. Sweating in warm room
D. Contraction of eye pupils in bright light

10.

A. Theorizing based on behavior


B. Inference from brain anatomy
C. Observing one's own consciousness
D. Conclusions from brain stimulation
Questions 11-25

DIRECTIONS: Each of questions 11 through 25 may have more than one correct
alternative. That is, for a given question, from zero to all five alternatives may be
correct. You must identify ALL the correct alternatives or items in order to achieve a
correct answer. Select the letter of the CORRECT answer.
11. Which of the following are stimulus factors used (or measured) in
conformity experiments?
I. Size of the group
II. Early experiences of the stooges
III. Personality structure of the critical subject
IV. Personality structure of the group
V. Presence or absence of a group of stooges
The CORRECT combination is:
A. I only
B. II,III
C. IV,V
D. V only
E. none of
these

12. Skinner's experiments with animals


I. suggest that pigeons cannot easily be taught such complex tasks as bowling or playing
ping pong
II. have little relevance to the understanding or control of human behavior
III. show that pigeons can be made "superstitious" if the training conditions are right
IV. suggest that reinforcement should be given immediately if one wishes to control
behavior
V. suggest that the principle of successive approximation is of little value in training
animals such as pigeons
The CORRECT combination is:
A. II, IV
B. I, III
C. III,IV
D. I, V
E. none of
these
13. Giving a person an adequate paycheck at the end of the month is an
example of
I. primary positive reinforcement
II. secondary negative reinforcement
III. immediate primary reinforcement
IV. delayed primary reinforcement V. delayed negative secondary reinforcement
The CORRECT combination is:
A. IV, V
B. I, III
C. I, IV
D. III, V
E. none of these
14. Pavlov's findings concerning "transmarginal excitation" suggest that
I. in the equilibration state, the animal reacts to weak stimuli the same as to strong
stimuli
II. in the ultra-paradoxical state, the animal over-reacts to average stimuli, but does not
react to weak stimuli
III. in the paradoxical state, the animal reacts to weak stimuli but not to strong stimuli
IV. in the paradoxical state, the animal reacts to average stimuli but not to strong stimuli

V. the cycle of events following the "transmarginal excitation" is more difficult to elicit
the second time
than the first
The CORRECT combination is:
A. I, II, III
B. I, II
C. II only
D. IV, V
E. none of
these
15 According to various studies:
I. When the activity in one's nervous system (that is, the neural firing) becomes too
intense, old patterns of
firing (habits) become interrupted and do not operate for a period of time.
II. Immediately following over-excitation of the nervous system, the organism enters a
brief plastic pattern in
which it can learn new habits more rapidly than is normally the case.
III. Children can learn a foreign language easier than adults, at least in part, because the
children have fewer interfering habit patterns than do the adults.
IV. There is a characteristic body posture in humans associated with over-excitation of
the nervous system or "transmarginal excitation."
V. Laboratory studies tell us very little about what really goes on in a human's nervous
system when that human experiences a true religious "conversion."
The CORRECT combination is:
A. I, IV
B. I, II, III
C. II, III, IV, V
these

D. I, II, III, IV, V

E. none of

16. Various experiments on "group pressures towards conformity" suggest that


I. the more the material that the subject must judge deals with concrete objects in the
real world (as opposed to material which merely calls for an opinion about an
unobservable quantity), the more likely it is that the subject will be influenced by the
group
II. judgments of fact are easier to shift than are attitudes
III. it is easier to shift personal preferences than it is to shift judgments of vague facts
IV. the easier the task set before the subject, the more likely it is the group will be able to
influence the subject's behavior
V. if the subject is asked to make judgments of material from memory, he yields less
readily than if he is allowed to judge on the basis of immediate perception of the
material
The CORRECT combination is:

A. I only
these

B. II, III

C. I, II, III, IV

D. all of these

E. none of

17. Various experiments on "group pressures towards conformity" suggest that


I. the vaguer the instructions the subject is given, the more likely it is that he will
conform to group opinion
II. in general, the more information the subject has concerning the group, the more
influence the group will have on him
III. most experimenters have found that maximum conformity is induced when the group
has at least seven or eight members
IV. It is quite important that all members of the reference group be unanimous (or nearly
so) in their judgments if pressure to conform is to be induced in the subject
V. up to a point, the larger the discrepancy between the subject's personal judgment and
that of the group, the less the subject is influenced by the group
The CORRECT combination is:
A. I only
B. I, II, III

C. II, III, IV

D. I, II, IV

E. II, IV, V

18. Various experiments on "group pressures towards conformity" suggest that


I. the subject is more likely to yield to group pressures if the members of the group are
physically present than if the subject merely hears their voices
II. the more prestige or competence members of the group are seen as having, the more
powerful agents they become in influencing the subject
III. subjects typically yield more to groups composed of strangers than to groups
composed of people they are acquainted with
IV. subjects typically yield more to hostile groups than to friendly groups
V. subjects typically yield more to groups towards which they (the subjects) are hostile than
to groups they like
The CORRECT combination is:
A. I only
B. II only
C. III only

D. IV only

E. III, IV

19. Various experiments on "group pressures towards conformity" suggest that


I. the more "out in the open" the subject is forced to be in making his judgments, the
more likely it is he will
resist group pressures
II. the subject will be more likely to yield if he is told that the whole group must come to
a unanimous decision on the matter at hand
III. in general, the more important the judgments that the subject must make are said to
be, the less likely it is that the subject will take the rest of the group into account
IV. conforming behavior is difficult to predict even in experimental situations
V. situational or "background" factors are of little importance in producing conformity
behavior

The CORRECT combination is:


A. I, II
B. III only

C. II only

D. II, IV

E. IV, V

20. In order to encourage a subject to resist group pressures


I.
one should reward him for non-conforming behavior
II.
make him think himself an expert in the task at hand
III.
give him one or two brief failure experiences in the task at hand prior to inducing
group pressures
IV.
make him aware of the penalties for anti-social behavior
V.
make him believe that the group is actually quite hostile towards him and
then let him make his judgments in private
The CORRECT combination is:
A. I only
B. I, V
C. II, III, IV
D. I, III
E. I, II, V
21. The pattern for most "religious" or other "conversion" experiences
USUALLY includes
I.
the climax of some great emotional experience
II.
the "great experience," or the "conversion-inducing" experience
III.
some sort of sensory deficit (such as hysterical blindness)
IV.
a plastic period immediately following the above during which indoctrination
typically takes place
V.
the occurrence of a true behavioral conversion, if all of the above happen
The CORRECT combination is:
A. I, II, III
B. I, III
C. I, IV, V
D. I, II
E. I, II, III, IV, V
22. Electro-convulsive shock
I. has been shown to be quite an effective form of therapy for most types of mental
illness
II. is seldom used by mental hospitals today
III. induces "retrograde amnesia" for events occurring just prior to the shock
IV. causes little physical damage to the subject's brain even if used extensively
V. can snap a severely depressed patient out time of his depression for at least a limited
period of
The CORRECT combination is:
A. III, V
B. III, IV, V
C. I only
D . II only
E. none of these

23. Experiments by Pavlov indicated that


I. what he called "transmarginal excitation" in dogs had many of the characteristics of
the
"great experience" in humans
II. "experimental neurosis" could be induced in dogs even if no real punishment was
given to the animals
III. dogs cannot be made into "masochists"
IV. following " transmarginal excitation" dogs tended to lose most of their conditioned
responses for a period
of time

V. the "transmarginal excitation" could later be re-induced in his animals even if only
some small part of the original exciting conditions were presented to the animals
The CORRECT combination is:
A. I, II
B. II,IV
C. IV, V
D. I, II, IV, V E. none of these
24. In classical conditioning, the
I.
CS always follows the UCS
II.
animal does not need to be trained to make the CR to the CS
III.
animal does not need to be trained to make the UCR to the UCS
IV.
CS is typically said to be "neutral" at the beginning of the experiment
V.
stronger or more intense the CS is, in general, the faster the conditioning takes
place
The CORRECT combination is:
A. I, II
B. I, IV
C. III, IV, V
D. I, IV, V
E. none of these
25. Which of the following are residual factors used (or measured) in
conformity experiments?
I.
Personality structure of the group
II.
Personality structure of the critical subject
III.
Presence or absence of a group of stooges
IV.
Difficulty of judgment of task
V.
Concreteness of material to be judged
The CORRECT combination is:
A. I only
B. II only
C. III only
D. IV only
E. V only
Questions 26-50
DIRECTIONS: Each question or incomplete statement is followed by several suggested
answers or completions. Select the one that BEST answers the question or completes the
statement.
26. Which of the following is NOT a criterion for distinguishing deviant from
normal behavior?
A. Personal distress
B. Disabling behavior tendencies
C. Poor reality contact
D. Statistical infrequency
E. All of these
27. A middle-aged man wears tennis shoes consistently to church on Sunday.
This behavior would BEST be described as
A. neurotic
B. eccentric
C. crazy
D. psychotic
E. pathological

28. The behavior defining psychosis in the implicit sense (loss of reality
contact) may be exhibited in which of the following?
A. Sensory-perceptual anomalies
B. Disorientation as to time, person or place

C. Thought disorder
D. Motor anomalies
E. All of the above
29. What model does society currently used as a basis for the terms and
concepts to be applied to deviant behavior?
A. Statistical
B. Moral
C. Medical
D. Behavioral E. Dynamic
30. Patterns of bodily symptoms which tend to occur together are known as
A. syncreases
B. syndromes C. profiles
D. pathological formations
E. anagrams
31. Which of the following difficulties are encountered with the dynamic model
for the study and treatment of deviant behavior?
A. Failure to comprehend the distinction between inference and observation
B. Language of this model is largely figurative
C. Absence of quantification
D. All of the above
E. Both A and C only
32. Reduction of the rate, speed, or magnitude of a conditioned response by
eliminating the reinforcement related to the response is known as
A. counterconditioning
B. spontaneous recovery
C. learning
D. operant
conditioning
E. extinction
Questions 33-36.
DIRECTIONS: With reference to the numbered parts in the following experimental
situation, select the number
1. If it is
a conditioned
stimulus
2. If it is
an unconditioned
response
3. If it is
an unconditioned
stimulus
4. If it is
a conditioned
response
A human subject is placed in a chair. A puff of air (33) is directed toward one eye for
one second and
the subject blinks (34). Contiguous with the air puff a light, of itself not able to elicit a
blink (35), is switched on in view of the subject. Eventually, the subject comes to
blink to the light (36) when it is unpaired with
the air puff.
33. The CORRECT answer is:
A. 1
B. 2
C. 3
D. 4
34. The CORRECT answer is:
A. 1
B. 2
C. 3
D. 4
35. The CORRECT answer is:
A. 1
B. 2
C. 3
D. 4
36. The CORRECT answer is:
A. 1
B. 2
C. 3
D. 4
37. The experimental situation described in questions 33 through 36 is an
example of which of the following forms of conditioning?
A. Classical conditioning
B. Semantic conditioning
C. Operant conditioning
D. Counter-conditioning
E. All of these

38. Questions 38 refers to the following graph:


A STIMULUS GENERALIZATION GRADIENT

300

Responses 200

100

500

520

540

560

580

600

Wavelength (my-)
In the original conditioning session, a subject was conditioned to respond to a
single wavelength of light. Assuming that the above graph represents the
strength of response of that subject to test stimuli of various wavelengths,
what is the MOST likely wavelength value of the single stimulus used in the
original conditioning?
A. 510
B. 540
C. 550
D. 560
E.
570
39. Under which of the following conditions will retention of the same learned
material, as measured by a recall test, be BEST?
A. When the material supports an idea held by the learner
B. When the material is at variance with an idea held by the learner
C. When the material is neutral to the learner (neither supporting nor contradicting his
ideas)
D. All of the above
E. None of the above
40. Which of the following is evidence for the notion that it is possible for
human learning to take place without the subject's being able to verbalize
what has been learned (learning without awareness)?
A. Eyeblink conditioning in infants
B. The "Greenspoon effect"
C. Conditioning of the pupillary response
D. All of the above
E. Only A and C

41. Which of the following curves BEST represents the motivation-efficiency relationship?

A.

B.

C.

D.

42. Subject A is offered $1 for each red paper square he can pick up from a
moving conveyor belt. Subject B is merely told to do the same task as
accurately and quickly as he can. At the end of the experiment, Subject A does
not recall the colors of the paper squares other than the red ones; Subject B
does remember and can report the other colors. The principle that BEST
describes this situation is:
A. The higher the motivation, the more selectively the individual attends to relevant
stimuli only
B. The principle of perceptual defense
C. The principle of rigidity
D. Decline in efficiency as an accompaniment to extreme motivation or unusually high
arousal

43. A little old lady Sunday school teacher is reading a novel and registers the
sentence,
"Goddam lights!" as "Guard, dim the lights!" The principle that BEST describes this
situation is:
A. The higher the motivation, the more slectively the individual attends to relevant
stimuli only
B. The principle of perceptual defense
C. The principle of rigidity
D. Decline in efficiency as an accompaniment to extreme motivation or unusually high
arousal
44. A 13-year-old boy goes to the candy machine he frequents and drops in his
dime, although the prices have risen since he last used the machine to 15 and
the sign on the machine says so. The boy keeps pulling on the knob, getting
angrier and angrier, till, finally, a clerk points out the rather obvious sign on
the machine that prices are now a nickel more.
- The principle that BEST describes this situation is:

A. The higher the motivation, the more selectively the individual attends to relevant
stimuli only
B. The principle of perceptual defense
C. The principle of rigidity
D. Decline in efficiency as an accompaniment to extreme motivation or unusually high
arousal

45. "Our boy" is on the line with two foul shots to make. The score is tied and
there will be only two seconds left to play in the game after his second shot.
He has to make at least one basket to insure our winning the game. He feels
the crowd is tense and he is so anxious to sink one that he misses both.
The principle that BEST describes this situation is:
A. The higher the motivation, the more selectively an individual attends to relevant
stimuli only
B. The principle of perceptual defense
C. The principle of rigidity
D. Decline in efficiency as an accompaniment to extreme motivation or unusually high
arousal
46. Although the image on your retina of a stop sign a block away is much
smaller than that of the same sign at a distance of five feet, you do not judge
the sign to actually grow larger as you approach it. This is an example of the
phenomenon of
A. retinal disparity
B. psychophysical equality
C. size constancy
D. visual compensation
E. audio compensation
47. Exposure to prolonged stress may bring about irreversible changes in the
functioning of certain bodily systems. This statement is
A. true
B. false
C. sometimes true, sometimes false
D. not determinable from
the information given
48. Organic disorders produced by psychological stressors
A. psychosomatic disorders
B. hallucinogenic disorders
C. parasympathetic disorders
D. carcinogenic disorders
E. none of these

are known as

49. Research on the effects of adrenalin and noradrenalin in relation to


emotion gives evidence for which of the following?
A. There are hormonal differences in the two states of emotion, anger and fear
B. There are individual differences in the typical hormonal reaction to threat and,
therefore, in the accompanying emotion
C. The emotional states of anger and fear are hormonally indiscriminable
D. Both A and B

E. Both A and C
50. Effects of the parasympathetic and sympathetic nervous system functions are with
respect to the
responses of visceral organs (responses such as rates of function, secretions, etc.).
A. the same
B. opposite
C. unknown
D. variable
E. unrelated
KEY (CORRECT ANSWERS)
1. C
11. E
21. E
31. D
41. D

2. B
12. C
22. A
32. E
42. A

3. A
13. E
23. D
33. C
43. B

4. B
14. A
24. C
34. B
44. C

5. D
15. D
25. B
35. A
45. D

6. D
16. E
26. D
36. D
46. C

7. C
17. D
27. B
37. A
47. A

8. D
18. B
28. E
38. C
48. A

9. A
19. C
29. C
39. A
49. D

10.
20.
30.
40.
50.

C
E
B
D
B

BASIC FUNDAMENTALS of the PRESCRIPTION


CONTENTS
SECTION I : PRESCRIPTION LANGUAGE-PHARMACEUTICAL LATIN
1. Latin in pharmacy and medicine
2. Commonly used Latin abbreviations,words,and translations
3. Common names of drugs and the Latin counterpart
SECTION II : THE PRESCRIPTION
4. Superscription
5. Inscription
6. Subscription
7. Signa (signatura)
8. Abbreviation of names of drugs
9. Abbreviation in subscription and
signa
10. Prescription examples
11. Importance of pharmaceutical
calculations
12. Fractions
13. Rules applying to all fractions
14. Working with fractions
15. Lowest terms
16. Lowest common denomincator
(LCD)
17. Adding fractions
18. Subtracting fractions
19. Multiplying fractions
20. Dividing fractions
21. Decimals
22. Adding decimals
23. Subtracting decimals
24. Multiplication of decimals

25. Division of decimals


26. Roman numerals
27. Weights and measures
28. Metric system
29. Apothecary system
30. Avoirdupois system
31. Relationship and approximate
equivalents
32. Ratio and proportion
33. Percentage preparations
34. Ratio preparations
35. Specific gravity
36. Specific gravity of liquids
37. Specific gravity of solids
38. Application of specific gravity to
pharmaceutical problems
39. Specific volume
40. Density
41. Temperature
42. Temperature calculation
43. Temperature conversion
44. Dosage
45. Concentration and dilution
46. Alligation

BASIC FUNDAMENTALS OF THE


PRESCRIPTION
Section I. PRESCRIPTION LANGUAGE-PHARMACEUTICAL LATIN

1.

Latin in pharmacy and medicine. Latin in prescription writing is centuries old.


Although more and more of the prescription is being written in English today, most
prescriptions are in part written in Latin or use the Latin abbreviations. As long as
physicians continue to use Latin and Latin abbreviations in prescription writing, the
pharmacist will have to be able to read and fully understand the terms that are used.
Latin used in medicine and prescription writing may be intended to conceal the nature
of the medication from the patient, to reduce the possibility of a patient's tampering
with a prescription, or to make the prescription universally legible to pharmacists
regardless of their national language. Probably the most outstanding reason for the use
of Latin in medicine and pharmacy is through force of habit. It has been done for so long
that it is hard to break away from the trend. Medical and pharmacy colleges teach the
physician to write prescriptions in Latin and the pharmacist to be able to interpret them.

2.

Commonly used Latin abbreviations, words, and translations. Since it would be


extremely time consuming and of doubtful value to present an entire course in
Pharmaceutical Latin here, it is strongly suggested that you read, learn, and memorize
the words, abbreviations, and meanings on the right. This will familiarize you with the
more common terms and phrases used in prescription writing and enable you to
understand the physician's or other prescriber's orders. Later, when you have time and
a desire to improve your command of Latin, as pertinent to pharmacy, procure a copy of
an authoritative pharmacy text. With the aid of such a text, the average individual may
become familiar with Latin as it applies to the professions of medicine and pharmacy.

Latin

PHARMACEUTICAL

LATIN

Abbreviation

English Translation

A.

ad
up to
ad libitum
pleasure
adde, addendus
let them be added

ad

to;

ad lib.

at

add.

add,

agitata ante usum


Shake before using
albus
white
alternus horisalt.
hours

agit. ant. us.


alb.
hor.

alternate

amplus
ana
each

amplus
aa.

large
of

ante
ante cibos; ante cibum
before food
cibus
ante meridiem
noon

a.
a.c.

before
before meals;

cib.; c.
A.M.

food

applicandus
applied

applicand.

aqua
aqua bullions
aqua destillata
water
aqua fervens
aqua frigida
aqua forte

aq.
aq. bull.
aq. dest.

before
to be
water
boiling water
distilled

aq. ferv.
aq. f rig.
aq. fort.

aromaticus
argentum
aurio

hot water
cold water
nitric acid

arum.
arg.

aromatic
silver

aur.

ear

B.
bene
bibe

bis in die
bolus

ben.

well

bib.

drink

b.i.d.
bol.

twice a day
a large pill

C.
capeat
capsula

cap.

charta
(powder)
charta cerati
waxed paper

chart.

cibus

cib.; c.

cap.

let him take


a capsule
paper

chart. cerat.
food

cochleare amplum
spoonful
cochleare magnum
spoonful
coehleare maximum
spoonful
cochleare plenum
spoonful
cochleare medium
spoonful
cochleare modicum
spoonful
coehleare infans
spoonful
cochleare minimum
spoonful
cochleare parvum
spoonful
cochleare paulus
spoonful

coch. amp.
coch. mag.

Table

coch. max.

Table

coch. plen.

Table

coch. med.

Dessert

coch. mod.

Dessert

coch. inf.

Tea

coch. min.

Tea

coch. parr.

Tea

coch. paul.

cola, colatus
strained
collu narium
douche
collu torium
mouthwash

Table

Tea

col.

strain,

coll un.

nasal

coll ut.

collyrium
collyr.
compositus
comp.
, compounded
congius
cong.
continuentur remedia
cont. rem.
medication
creta
cret.
CUM
c.; C

da
decem

d.
decem

eye lotion
compound
gallon
continue the
chalk
with

give
ten

dentur
give
dentur tales doses
dose
dexter
dies

dent.; d.

let be given;

d.t.d.

give of such

dext.
d.

right
day

diebus alternis
days
diebus secundis
day
diebus tertiis

dieb. alt.

on alternate

dieb. secund.

every second

diluo; dilutus
dispensa; dispensator
be dispensed

dil.
disp.

dieb. tert.

dividatur
div.
dividatur in partes aequales
into equal parts
dosis
drachma
dura

every three days


dilute
dispense; let
divide

div. in par. aeq.

divide

dos.; d.
dur.

a dose
a drachm
hard

e
et
e lact.
e.m.p.

out of; in
and
in milk
in the manner

ft.

make; let be

E
e
et
e lacte
ex modo praescripto
prescribed

F
fac; fiat; fiant
made
ferrum
filtra
flavus
folium; folia
fortis; fortior
stronger

ferr.

iron
filtra
flay.
fol.
fort.

filter
yellow
leaf; leaves
strong;

garg.
grad.

a gargle
gradually

G
gargarisma
gradatim

grossus
gramma
granum
gutta; guttae

gros.

large
Gm.
gr.
gtt.

gram
grain
drop; drops

hor.; h
h.s.
hydrarg.

an hour
at bedtime
mercury

in aur. sinist.
in d.
ind.
in ocul. laev; O.L.
inter
in cib.
int. noct.

in the left ear


in a day
daily
in the left eye
between
between meals
during the

in vit.

in glass

lac
lev.
lb.
liq.
lot.
lut.

milk
light
pound
liquid; solution
lotion
yellow

magnus
mane
massa
milligramma
minimum
misce
mistura
mitte

mag.
man.
mass.
mg.; mgm.
sk { LATIN LETTER}
M.
mist.
mitt.

large
morning, in the
a mass
a milligram
a minim
Mix
mixture
send

mollis

moll.

soft

nulls
nebula
niger
nocte

n.
nebul.
nig.
noct.

nostril
a spray
black
at night

hora
hora somni
hydrargyrum

I
in aurem sinistram
in die
in dies
in oculo laevo
inter
inter cibos
inter noctem
night
in vitro

L
lac
levis
libra
liquor
lotio
luteus

modo praescripto prescribed


manner

mod. praes.

in the

nocte maneque
morning
non
non repetatur

noct. maneq.; n. et m.
non
non rep.

octarius
oculus
oculo dextro
in the
oculo sinistro
the
oculo laevo
the
oculo utro

night and

not
do not repeat

0.; oct.

a pint

ocul.
0.D.; ocul. dext.

the eye
the right eye,

0.S.; ocul. sinist.

the left eye, in

0.L.; ocul. laev.

the left eye, in

O.U.; ocul. utro

in each eye

oleum

ol.

oil

omnis
omni alters hora
hour
omni hora
omni mane
omni quarta hors
hours

omn.
omn. alt. hor.

every
every alternate

omn. hor.
omn. man.
omn. 4 hor.

every hour
every morning
every four

parvus
per
per
per os
phiala
phiala fusca
phiala prius agitata
being shaken
placebo
plumbum
ponderosus

parv.

small
by means of
by mouth
a bottle
a brown bottle
the bottle first

placebo
plumb.
pond.

I please
lead
heavy

post aurum
post cibum; post cibos
after meals
post meridiem

post aur.
p.c.

behind the ear


after food;

praecipitatus
pro capillis

per os
phial.; p.
phial. fuse.
p.p.a.

P.M.

ppt.
pro capil.

afternoon
precipitated
for the hair

pro recto
pro re nata
arises
pro usa externo
pulvis

pro rect.
p.r.n.
pro us. ext.
pulv.

rectal
as occasion
for external use
powder

Q
quantitatim sufficientum
quantity
quaque
qq.
quaque die
quaque hora
quater in diem
day

repetatur
recipe
ruber

q.s.

a sufficient

q.d.
q.h.
q.i.d.

each, every
every day
every hour
four times a

Rx; 1
rub.

let it be repeated
take thou
red

s.a.

scruple
according to

rep.

S
scrupulus
secundum artem
the art
secundum legem
law
semi
semissem
sesqui

3
s.l.

according to

sem.
ss
sesqui

one-half
one-half
one and a half

signa
simul
sine
sine aqua
si opus sit
solve

sig.; S.
simul
s
sin. q.; s. aq.
s.o.s.
solv.

write

spiritus vini rectificatus


spiritus vini tenuis

S.V.R.
S.V.T.

alcohol
diluted alcohol

spiritus vini vitis

sp. vin. vit.

at one time
without
without water
if there is need
dissolve

brandy

spiritus frumenti

sp. frum.

statim
succus
syrupus

stat.
suc.
syr.

whiskey
immediately
juice
syrup

T
tabella
talis
ter in die
tree; trium
tussis

tab.
tal.; t.
t.i.d.
tres; trium
tuns.

uncia
unguentum
ut dictum

5
ung.; ungt.
ut diet.

viridis

3. Common

vir.

tablet
of such
three times a day
three
cough
ounce
ointment
as directed
green

names of drugs and the Latin counterpart

Common Name

Latin

Acid
Alcohol
Rectificatus (SVR)

Acidum
Spiritus Vini

Belladonna Leaf
Belladonna Root
Bitter

Belladonna Folium
Belladonna Radix
Amari

Cascara
Castor Oil

Rhamnus Purshiana
Oleum Ricini

Charcoal
Wood Charcoal

Carbo
Carbo Ligni

Clove Oil

Oleum Caryophylli

Coal Tar
Coal Tar Solution
Detergens;

Pix Carbonis
Liquor Carbonis
Liquor Picis Carbonis

Cod Liver Oil


Corn Oil
Cottonseed Oil
Gossypie Seminis

Oleum Morrhuae
Oleum Maydis
Oleum

Earth

Terra

Hard Soap

Sapo Duris

Juice

Succus

Lard
Lime
Linseed Oil
Medicinal Soft Soap
Medicinalis

Adeps
Calx
Oleum Lini
Sapo Mollis

Oil
Ointment
Orange

Oleum
Unguentum
Aurantium

Peppermint
Piperita
Peppermint Oil
Piperitae
Purified Cotton
Purificatum

Mentha
Oleum Menthal
Gossypium

Rosin

Resina

Seed
Sherry Wine
Solution
Spearmint Oil
Viridis
Spermaceti
Starch
Sucrose
Saccharum
Sweet
Syrup

Semen
Vinum Xericum
Liquor
Oleum Menthae

Turpentine
Wax
Wild Cherry
Whiskey

Terebinthinae
Cera
Prunus Virginians
Spiritus Frumenti

Cetaceum
Amylum
Surcrosum;
Dulcis
Syrupus

White Ointment
Alba
White Wax
Wool Fat
Yellow Ointment
Flavum
Yellow Wax

Ungunentum
Cera Alba
Adeps Lanae
Unguentum
Cera Flava

You might also like